MBE (Torts, Con law, Crim law/Crim Pro

Ace your homework & exams now with Quizwiz!

A woman entered into a contract with a painter, by the terms of which the painter was to paint the woman's office for $1,000 and was required to do all of the work over the following weekend so as to avoid disruption of the woman's business. Assume the following facts. If the painter had started to paint on the following Saturday morning, he could have finished before Sunday evening. However, he stayed home that Saturday morning to watch the final game of the World Series on TV and did not start to paint until Saturday afternoon. By late Saturday afternoon, the painter realized that he had underestimated the time it would take to finish the work over the weekend unless he hired a helper. He also stated that to do so would require an additional charge of $200 for the work. The woman told the painter that she apparently had no choice but to pay "whatever it takes" to get the work done as scheduled. The painter hired a friend to help finish the painting and paid the friend $200. The woman has offered to pay the painter $1,000. The painter is demanding $1,200. How much is the painter likely to recover? A: $1,000 only, because the woman received no consideration for her promise to pay the additional sum. B: $1,000 only, because the woman's promise to pay "whatever it takes" is too uncertain to be enforceable. C: $1,200, in order to prevent the woman's unjust enrichment. D: $1,200, because the impossibility of the painter's completing the work alone discharged the original contract and a new contract was formed.

A is correct. A unilateral modification of a contract (such as the woman's promise to pay an additional $200 for the same work) lacks consideration because of the preexisting duty rule. However, some unilateral modifications may be enforceable notwithstanding the lack of additional consideration if the modification was done in good faith in light of circumstances that the parties did not anticipate at the time the contract was formed. Because the painter should have anticipated the circumstances that required his hiring an extra worker, the modification is not enforceable and the woman is not bound by her promise to pay the extra $200. B is incorrect. The promise viewed in the context of the painter's demand was sufficiently definite. C is incorrect. Unjust enrichment is not applicable where the benefit has been conferred pursuant to a binding contract. D is incorrect. The facts do not show that performance was impossible.

A defendant was prosecuted for sexually abusing his 13-year-old stepdaughter. The stepdaughter testified to the defendant's conduct. On cross-examination, defense counsel asks the stepdaughter, "Isn't it true that shortly before you complained that your stepfather abused you, he punished you for maliciously ruining some of his phonograph records?" The question is A: proper, because it relates to a possible motive for the stepdaughter to falsely accuse the defendant. B: proper, because the stepdaughter's misconduct is relevant to her character for veracity. C: improper, because the incident had nothing to do with the stepdaughter's truthfulness. D: improper, because it falls outside the scope of direct examination.

A is correct. A witness may be properly questioned about any bias or motive to lie the witness may have. The defendant's counsel is permitted to inquire into whether the stepdaughter may have a bias or motive to lie about the abuse since the defendant had punished her shortly before the complaint of the abuse.

The defendant and his friend were fooling around with a pistol in the friend's den. The defendant aimed the pistol in the friend's direction and fired three shots slightly to the friend's right. One shot ricocheted off the wall and struck his friend in the back, killing him instantly. The most serious crime of which the defendant can be convicted is A: murder. B: voluntary manslaughter. C: involuntary manslaughter. D: assault with a dangerous weapon

A is correct. At common law, murder is defined as the unlawful killing of another human with malice aforethought. By firing multiple shots from a pistol extremely close to his friend, the defendant was acting with reckless indifference to an unjustifiably high risk to human life. This reckless indifference would qualify as malice aforethought, which would support a murder conviction.

A police officer stopped the defendant for speeding late one night. Noting that the defendant was nervous, he ordered him from the car and placed him under arrest for speeding. By state law, the officer was empowered to arrest the defendant and take him to the nearest police station for booking. He searched the defendant's person and discovered a package of heroin in his jacket pocket. The defendant is charged with possession of heroin. At trial, the defendant's motion to prevent introduction of the heroin into evidence, on the ground that the search violated his federal constitutional rights, will most probably be A: denied, because the search was incident to a valid custodial arrest. B: denied, because the officer acted under a reasonable suspicion and legitimate concern for his own personal safety. C: granted, because there was no reasonable or proper basis upon which to justify conducting the search. D: granted because the officer was not in fear and had no suspicion that the defendant was transporting narcotics.

A is correct. Authorities may conduct a warrantless search incident to a lawful arrest as long as the scope of the search is constrained to the defendant's wingspan and performed contemporaneously with the arrest. Here, the officer was empowered by state law to arrest the defendant for speeding, the search was within the defendant's wingspan, and the search was contemporaneous with a lawful arrest. Therefore, the search was valid and the defendant's motion should be denied

A man owed his friend $1,000, plus interest at 8% until paid, on a long-overdue promissory note, collection of which would become barred by the statute of limitations on June 30. On the preceding April 1, the man and his friend both signed a writing in which the man promised to pay the note in full on the following December 31, plus interest at 8% until that date, and the friend promised not to sue on the note in the meantime. The friend, having received some advice from his nonlawyer brother-in-law, became concerned about the legal effect of the April 1 agreement. On May 1, acting pro se as permitted by the rules of the local small claims court, he filed suit to collect the note. Assuming that there is no controlling statute, is the April 1 agreement an effective defense for the man? A: Yes, because the man's promise to pay interest until December 31 was consideration for the friend's promise not to sue. B: Yes, because the law creates a presumption that the man relied on his friend's promise not to sue. C: No, because there was no consideration for the friend's promise not to sue, in that the man was already obligated to pay $1,000 plus interest at 8% until the payment date. D: No, because the man's April 1 promise is enforceable with or without consideration

A is correct. Because the man was already obligated to pay the friend $1,000 at 8% interest, normally a promise to do the same would not be sufficient consideration to support the friend's promise to refrain from suit. However, the man offered to extend interest payments until December 31 in consideration for the friend's promise not to sue during the remaining time under the statute of limitations. Therefore, there was adequate consideration for the April 1 agreement, and it will provide an effective defense against the friend's claim. B is incorrect. The man does not need to invoke detrimental reliance because there was adequate consideration for the April 1 agreement. C is incorrect. The payment of interest for six additional months is adequate consideration. D is incorrect. The April 1 agreement would provide an adequate defense against the friend's claim.

A woman decided to steal a necklace that belonged to her neighbor. She knew where the neighbor kept the necklace because she had been in the neighbor's house on many occasions when the neighbor had taken off the necklace and put it away in a jewelry box in the bathroom. One night, the woman went to the neighbor's house. The neighbor was away, and the house was dark. The woman opened the bathroom window, saw the jewelry box on the counter, and started to climb inside. As her leg cleared the window sill, the neighbor's dog began to bark loudly. Terrified, the woman jumped back outside and fled. The crimes below are listed in descending order of seriousness. What is the most serious crime, if any, committed by the woman? A: Burglary. B: Attempted burglary. C: Attempted larceny. D: No crime.

A is correct. Burglary requires: (i) a breaking; (ii) and entry; (iii) of the dwelling; (iv) of another; (v) at nighttime; (vi) with the intent of committing a felony. Here, the woman is guilty of burglary because she unlawfully entered the neighbor's house at night with the intent to commit a felony (larceny). Her actions constituted the requisite entry of the neighbor's house because it is sufficient if any part of the actor's person intruded, even momentarily, into the structure. It has been held that the intrusion of a part of a hand in opening a window, or the momentary intrusion of part of a foot clearing the window sill, constituted the requisite entry.

Congress enacted a statute providing grants of federal funds for the restoration and preservation of courthouses that were built before 1900 and are still in use. The statute contains an inseverable condition requiring that any courthouse restored with the aid of such a grant must be equipped with ramps and other facilities necessary to accommodate physically handicapped people. A law of a particular state requires public buildings in the state to have ramps and other facilities for handicapped people. It exempts from those requirements any building that is more than 70 years old if the State Board of Architects finds that the installation of such facilities would destroy the architectural integrity of the building. The county courthouse in the state was built in 1895 and is still in use. It does not contain ramps or other special facilities for handicapped people. The State Board of Architects has determined that the installation of those facilities would destroy the architectural integrity of the building. The county board applies for a federal grant to restore and preserve that county's courthouse. If the county board restores the courthouse with the aid of a federal restoration and preservation grant, is the board bound to install ramps and other facilities for handicapped people in that building? A: Yes, because Congress may impose reasonable conditions related to the public welfare on grants of federal funds to public bodies when the public bodies are free to accept or reject the grants. B: Yes, because the rights of handicapped and disabled people are fundamental rights that take precedence, as a constitutional matter, over considerations of architectural integrity. C: No, because the Constitution does not authorize the federal government to direct the actions of the states or any of their political subdivisions with respect to matters affecting their own governmental buildings. D: No, because any acceptance of this condition by the county board of supervisors would, as a matter of law, be considered to be under duress.

A is correct. Congress can condition the grant of federal funds to public bodies on compliance with measures related to the public welfare as long as the public body is free to accept or reject the grants.

Current national statistics show a dramatic increase in the number of elementary and secondary school students bringing controlled substances (drugs) to school for personal use or distribution to others. In response, Congress enacted a statute requiring each state legislature to enact a state law that makes it a state crime for any person to possess, use, or distribute, within 1,000 feet of any elementary or secondary school, any controlled substance that has previously been transported in interstate commerce and that is not possessed, used, or distributed pursuant to a proper physician's prescription. This federal statute is A: unconstitutional, because Congress has no authority to require a state legislature to enact any specified legislation. B: unconstitutional, because the possession, use, or distribution, in close proximity to a school, of a controlled substance that has previously been transported in interstate commerce does not have a sufficiently close nexus to such commerce to justify its regulation by Congress. C: constitutional, because it contains a jurisdictional provision that will ensure, on a case-by-case basis, that any particular controlled substance subject to the terms of this statute will, in fact, affect interstate commerce. D: constitutional, because Congress possesses broad authority under both the General Welfare Clause and the Commerce Clause to regulate any activities affecting education that also have, in inseverable aggregates, a substantial effect on interstate commerce.

A is correct. Congress does not have the authority to require a state legislature to enact any specified legislation except when it is acting under its spending power. This is not a spending provision, so Congress cannot force states to enact these laws.

At a defendant's murder trial, the defendant calls his first witness to testify that the defendant has a reputation in their community as a peaceable and truthful person. The prosecutor objects on the ground that the witness's testimony would constitute improper character evidence. The court should A: admit the testimony as to peaceableness, but exclude the testimony as to truthfulness. B: admit the testimony as to truthfulness, but exclude the testimony as to peaceableness. C: admit the testimony as to both character traits. D: exclude the testimony as to both character traits.

A is correct. Evidence of the defendant's reputation for peaceableness is relevant to the murder charge, a crime of violence, while his reputation for truthfulness is irrelevant to such a charge and no other basis exists to admit it.

A homeowner contracted in writing with a kitchen contractor to renovate her kitchen for $25,000, "subject to the homeowner's complete personal satisfaction." The contractor replaced the cabinets, flooring, and countertops and then sought payment from the homeowner. The homeowner paid the contractor only $20,000, truthfully saying that she did not like the finish on the cabinets and was therefore not satisfied. If the contractor sues the homeowner for the balance of the contract price, will the contractor be likely to prevail? A: No, because a condition to the homeowner's obligation to pay was not satisfied. B: No, because the contractor breached his duty of good faith and fair dealing by supplying unsatisfactory materials. C: Yes, because the homeowner breached the covenant of good faith and fair dealing by rejecting the cabinets without justification. D: Yes, because the homeowner was the first party to breach the contract.

A is correct. If a party's promise is subject to a condition, there can be no breach of contract by that party until the condition has been fulfilled. Here, the homeowner's promise to pay is subject to her complete personal satisfaction with the contractor's work. The condition of her honest satisfaction with the contractor's work has not been fulfilled, and thus there can be no breach. B is incorrect. The contractor will not prevail because a condition to the homeowner's obligation to pay was not satisfied. C is incorrect. As stated in the facts, the homeowner was truthful in her dissatisfaction with the finish of the cabinets. This would not amount to a breach of the covenant of good faith and fair dealing which would deny the contractor the benefit of the contract. D is incorrect. As discussed above, there can be no breach of contract until a condition has been fulfilled. Because the condition of the homeowner's satisfaction had not been fulfilled, there is no breach of contract.

A landowner owned Greenacre, a tract of land, in fee simple. The landowner entered into a valid written agreement with a purchaser under which the landowner agreed to sell and the purchaser agreed to buy Greenacre by installment purchase. The contract stipulated that the landowner would deliver to the purchaser, upon the payment of the last installment due, "a warranty deed sufficient to convey the fee simple." The contract contained no other provision that could be construed as referring to title. The purchaser entered into possession of Greenacre. After making 10 of the 300 installment payments obligated under the contract, the purchaser discovered that there was outstanding a valid and enforceable mortgage on Greenacre, securing the payment of debt in the amount of 25% of the purchase price the purchaser had agreed to pay. There was no evidence that the landowner had ever been late in payments due under the mortgage and there was no evidence of any danger of insolvency of the landowner. The value of Greenacre now is four times the amount on the debt secured by the mortgage. The purchaser quit possession of Greenacre and demanded that the landowner repay the amounts the purchaser had paid under the contract. After the landowner refused the demand, the purchaser brought an appropriate action against the landowner to recover damages for the landowner's alleged breach of the contract. In such action, should damages be awarded to the purchaser? A: No, because the time for the landowner to deliver marketable title has not arrived. B: No, because the purchaser assumed the risk by taking possession. C: Yes, because in the absence of a contrary express agreement, an obligation to convey marketable title is implied. D: Yes, because the risk of loss assumed by the purchaser in taking possession relates only to physical loss.

A is correct. In general, real estate purchases involve a period of several months between the time the contract is signed and the closing date. A seller is not required to provide marketable title until that closing date arrives. Under an installment contract, although the period between contracting and closing may be several years, the same rule regarding title still applies. In this case, the facts indicate that the parties have entered into an installment contract and that upon the 300th payment, the deed would be delivered. Therefore, the landowner is not required to provide marketable title until the time arrives for the purchaser to make the 300th payment. Since there are still 290 payments to be made, the landowner's time for performance has not yet arrived. B is incorrect. The buyer is at no more risk for having taken possession than she would have been without doing so. C is incorrect. While technically correct, is irrelevant to these facts since the time to convey title has not yet arrived. D is incorrect. As explained above, risk of loss is not at issue for this question because the buyer is not any more at risk for taking possession of the property.

A contractor filed a diversity action against a homeowner in federal court in State A. The contractor alleged that the homeowner failed to pay $150,000 due on a valid service contract to install a kitchen in the homeowner's home. At trial, however, the homeowner disputed the contract's validity, testifying before the jury that the contractor installed the kitchen as a gift and exchanged emails referring to a "contract" that had been in jest. The homeowner's testimony also made clear that both parties were citizens of State A. Shortly into the homeowner's presentation of evidence, the contractor's attorney became concerned that the jury was overly sympathetic to the homeowner. Is the contractor's attorney able to prevent the case from being submitted to the jury? A: Yes, because the contractor's attorney may file a motion to dismiss for lack of subject-matter jurisdiction. B: Yes, because the contractor's attorney may file a motion for summary judgment. C: No, because the contractor is not entitled to judgment as a matter of law. D: No, because the contractor's attorney has no option but to continue with the trial

A is correct. Motions to dismiss for lack of subject-matter jurisdiction are always available, no matter what stage the action is at. During the course of this trial, it came out that both the plaintiff and the defendant are domiciled in State A, meaning that there is not complete diversity between the parties. The other way the court could have had jurisdiction was through federal question jurisdiction. However, as the facts state, this is a contract issue, and no federal question came up. Without complete diversity or a federal question, there is no valid basis for subject-matter jurisdiction, and the case cannot stay in federal court.

Police officers had probable cause to believe that drug dealing was routinely taking place in a particular room at a local motel. The motel manager authorized the officers to enter the room and provided them with a passkey. Without obtaining a warrant, the officers knocked on the room's door, announced their presence, and told the occupants that they would like to speak with them. The officers then heard yelling and repeated flushing of the toilet. They then used the passkey and entered the room, where they saw the occupants dumping drugs into the toilet. The occupants of the room were charged with drug dealing and have moved to suppress the drugs. Should the court grant the motion to suppress? A: No, because exigent circumstances justified the officers' entry. B: No, because the motel manager consented to the officers' entry. C: Yes, because exigent circumstances cannot excuse the lack of a warrant. D: Yes, because the officers cannot benefit from exigent circumstances that they created.

A is correct. Police officers may conduct a warrantless search and seizure of evidence that is likely to disappear before a warrant can be obtained. These types of circumstances are called "exigent circumstances." Here, the police officers reasonably believed that evidence of a crime was being disposed of, based on the actions of the occupants in the motel room, and acted accordingly to prevent the evidence from disappearing. Therefore, exigent circumstances justified the officers' warrantless entry.

A large privately owned and operated shopping mall is open to the public and includes small shops, major department stores, and restaurants that are located around a pedestrian area. It also has several movie theaters, an ice-skating rink, a small amusement park, and a branch of the local public library. The mall is advertised as "a small town with a big-town feel." During shopping hours, a group of 10 protesters gathered in the pedestrian area near the entrance to a department store to denounce the sale of animal fur products in that store. The protesters were peaceful and did not interfere with traffic into or out of the store, but they carried signs and vocally communicated their message to people walking in the area. Mall management quickly broke up the protest and required the protesters to leave the mall. The protesters have sued the mall, claiming that their right to freedom of speech guaranteed by the First and Fourteenth Amendments was violated. Should the protesters prevail? A: No, because the mall is private property, and there was no state action to which the freedom of speech guarantees of the First and Fourteenth Amendments apply. B: No, because the prohibition of protests adjacent to the entrance of a department store during shopping hours is a constitutionally proper limitation on the time, place, and manner of speech. C: Yes, because the mall is functionally equivalent to a town and, therefore, its actions are subject to the Constitution's guarantees of freedom of speech and assembly. D: Yes, because the mall's restriction on the protesters' speech was broader than necessary to ensure proper access to the department store.

A is correct. The First Amendment itself does not require owners of private property that is open to the public to allow access for purposes of the public exercise of free speech or protesting. Here, the mall is privately-owned. Constitutional rights are triggered only when the government tries to limit individual freedoms. The shopping mall, rather than the government, broke up the protest. Therefore, the First and Fourteenth Amendments do not apply. B is incorrect. As stated above, the First and Fourteenth Amendments do not apply. The shopping mall is not a government entity; it is privately-owned. Therefore, an analysis of time, place, and manner is unnecessary. C is incorrect. Even if the mall is advertised as "a small town with a big-town feel," it is still a privately-owned shopping center. It is not subject to the Constitution's guarantees of freedom of speech and assembly. D is incorrect. As stated above, the mall is privately-owned and can, therefore, restrict the protesters' speech as it sees fit.

An electrician from State A filed a diversity action in State A federal court against an actress who resided in State B. The electrician alleged that the actress failed to pay the electrician $150,000 due on a valid service contract to install a kitchen in the actress's home. The actress disputed the contract's validity, testifying before the jury that the electrician installed the kitchen as a gift and exchanged emails referring to a "contract" that had been in jest. The electrician's attorney became increasingly concerned that the jury did not like his client during the trial. What is the electrician's attorney's best course of action after the close of evidence? A: File a motion for judgment as a matter of law. B: File a motion for a new trial on the ground of juror misconduct. C: There is nothing that the electrician's attorney may do at this point. D: File a motion for summary judgment.

A is correct. The attorney should file a motion for judgment as a matter of law and argue that a reasonable jury would not have a legally sufficient basis to find that there had been no contract.

In 1970, a cattle company bought a 150-acre tract of agricultural land well suited for a cattle feedlot. The tract was 10 miles from the city and five miles from the nearest home. By 2006, the city limits extended to the cattle company's feedlot. About 10,000 people lived within three miles of the cattle-feeding operation. The cattle company land is outside the city limits and no zoning ordinance applies. The cattle company uses the best and most sanitary feed lot procedures, including chemical sprays, to keep down flies and odors and frequently removes manure. Despite these measures, residents of the city complain of flies and odors. An action has been filed by five individual homeowners who live within half a mile of the cattle company feedlot. Flies in the area are five to 10 times more numerous than in other parts of the city, and extremely obnoxious odors are frequently carried by the wind to the plaintiffs' homes. The odors do not affect any other part of the city. The flies and odors are a substantial health hazard. If plaintiffs assert a claim based on public nuisance, plaintiffs will A: prevail, because plaintiffs sustained harm different from that suffered by the public at large. B: prevail, because the cattle company's acts interfered with a person's enjoyment of his property. C: not prevail, because only the state may bring an action based on public nuisance. D: not prevail, because plaintiffs came to the nuisance.

A is correct. The cattle lot qualifies as a public nuisance because it unreasonably interferes with the health and property rights of the community. Although public nuisances generally must be prosecuted by public authorities, recovery for a public nuisance is possible if a private party suffered some unique damage not suffered by the general public. Because the plaintiffs suffered different harm than the public at large, they will prevail

In a medical malpractice suit by a patient against his doctor, the patient seeks to introduce a properly authenticated photocopy of the patient's hospital chart. The chart contained a notation made by a medical resident that an aortic clamp had broken during the plaintiff's surgery. The resident made the notation in the regular course of practice, but had no personal knowledge of the operation, and cannot remember which of the operating physicians gave him the information. The document is A: admissible as a record of regularly conducted activity. B: admissible as recorded recollection. C: inadmissible as a violation of the best evidence rule. D: inadmissible, because it is hearsay within hearsay.

A is correct. The chart is admissible as a "business record" because it is a record of acts, events, conditions, opinions, or diagnoses, made at or near the time by, or from information transmitted by, a person with knowledge, kept in the course of regularly conducted business, and it was the regular practice to make the record

On December 1, a broker contracted with a collector to sell her one of a certain type of rare coin for $12,000, delivery and payment to occur on the next March 1. To fulfill that contract, and without the collector's knowledge, the broker contracted on January 1 to purchase for $10,000 a specimen of that type of coin from a hoarder, delivery and payment to occur on February 1. The market price of such coins had unexpectedly fallen to $8,000 by February 1, when the hoarder tendered the coin and the broker repudiated. On February 25, the market in such coins suddenly reversed and had stabilized at $12,000 on March 1. The broker, however, had failed to obtain a specimen of the coin and repudiated his agreement with the collector when she tendered the $12,000 agreed price on March 1. Later that day, after learning by chance of the broker's dealing with the collector, the hoarder telephoned the collector and said: "Listen, the broker probably owes me at least $2,000 in damages for refusing wrongfully to buy my coin for $10,000 on February 1 when the market was down to $8,000. But I'm in good shape in view of the market's recovery since then, and I think you ought to get after the so-and-so." If the collector immediately sues the broker for his breach of the broker-hoarder contract, which of the following will the court probably decide? A: The broker wins, because the collector, if a beneficiary at all of the broker-hoarder contract, was only an incidental beneficiary. B: The broker wins, because as of March 1 neither the hoarder nor the collector had sustained any damage from the broker's repudiation of both contracts. C: The collector wins, because she was an intended beneficiary of the broker-hoarder contract, under which damages for the broker's repudiation became fixed on February 1. D: The collector wins, because she took an effective assignment of the hoarder's claim for damages against the broker when the hoarder suggested that the collector "get after the so-and-so."

A is correct. The contract between the hoarder and the broker had nothing to do with or anticipate the collector's interest in the deal, and any future benefit he would have received was only incidental, preventing him from having any contractual rights. B is incorrect. The broker will win because the collector was merely an incidental third-party beneficiary. Moreover, under the UCC, damages would be calculated at the time the broker repudiated (February 1), not on March 1, as indicated in this answer choice. C is incorrect. An intended beneficiary is one who is either given rights or duties under a contract or a party transfers rights or duties to them. Here, the collector was neither included in the original contract nor transferred any rights, and is thus only an incidental beneficiary without standing. D is incorrect. The language "I think you ought to get after the so-and-so" does not manifest intent to completely and immediately transfer rights to the collector and is thus insufficient to constitute an assignment.

A clerk lived on the second floor of a small convenience store/gas station that he owned. One night he refused to sell the defendant a six-pack of beer after hours, saying he could not violate the state laws. The defendant became enraged and deliberately drove his car into one of the gasoline pumps, severing it from its base. There was an ensuing explosion causing a ball of fire to go from the underground gasoline tank into the building. As a result, the building burned to the ground and the clerk was killed. In a common-law jurisdiction, if the defendant is charged with murder and arson, he should be A: convicted of both offenses. B: convicted of involuntary manslaughter and acquitted of arson. C: convicted of arson and involuntary manslaughter. D: acquitted of both offenses.

A is correct. The defendant could be convicted of either murder with malice aforethought or felony murder with arson as the underlying felony. In addition, the defendant's driving his vehicle into a gasoline pump also had the foreseeable consequences of igniting the gasoline, creating a ball of fire, and burning the building the clerk was in. Therefore, the defendant is guilty of both arson and murder.

A plaintiff sued a defendant for dissolution of their year-long partnership. One issue concerned the amount of money the plaintiff had received in cash. It was customary for the defendant to give the plaintiff money from the cash register as the plaintiff needed it for personal expenses. The plaintiff testified that, as he received money, he jotted down the amounts in the partnership ledger. Although the defendant had access to the ledger, he made no changes in it. The defendant seeks to testify to his memory of much larger amounts he had given the plaintiff. The defendant's testimony is A: admissible, because it is based on the defendant's firsthand knowledge. B: admissible, because the ledger entries offered by a party opponent opened the door. C: inadmissible, because the ledger is the best evidence of the amounts the plaintiff received. D: inadmissible, because the defendant's failure to challenge the accuracy of the ledger constituted an adoptive admission.

A is correct. The defendant is seeking to testify that he provided larger amounts to the plaintiff than was indicated in the ledger. That testimony is based on the defendant's firsthand knowledge; he was the one who handed the plaintiff the money. Therefore, his testimony is admissible.

A defendant has been charged with making a false statement to a federally insured financial institution to secure a loan. At trial, the prosecutor calls the defendant's wife as a willing witness to testify that the defendant told her in confidence that he had misrepresented his assets on the loan application. The defendant objects to his wife's testimony. Should the testimony be admitted? A: No, because even though the wife is a willing witness, the defendant has the right to exclude confidential marital communications in federal court. B: No, but only if the law of the state where the defendant and his wife reside recognizes a privilege for confidential marital communications. C: Yes, because in federal court the right not to testify belongs to the testifying spouse, and she is a willing witness. D: Yes, because while the adverse testimonial privilege is recognized in federal court, the marital communications privilege is not.

A is correct. The defendant may invoke the marital privilege because he told his wife the information in confidence, during their marriage, and no other facts are offered to indicate that the privilege would be otherwise inapplicable (such as abuse or a third party's presence).

An undercover police detective told a local drug dealer that she wanted to buy cocaine but that she needed time to raise the necessary funds. The drug dealer said that he needed time to get the cocaine. They agreed to meet again in 10 days. An hour later, without a warrant, other officers forcibly entered the drug dealer's apartment and arrested him for attempted possession of a controlled substance. If the drug dealer is prosecuted in a common law jurisdiction for attempted possession of cocaine, should he be convicted? A: No, because he had not taken sufficient steps toward commission of the crime. B: No, because he was illegally arrested. C: Yes, because by objective standards the undercover detective and the drug dealer had made an agreement that the dealer would obtain cocaine. D: Yes, because he unequivocally expressed his intent to obtain the cocaine.

A is correct. The drug dealer took no action, much less any action that would qualify at common law, toward obtaining the cocaine. Indeed, the drug dealer likely would not be guilty of attempt even under the Model Penal Code's (MPC) broadened standards because there was no substantial step toward commission of the crime. See M.P.C. § 5.01(1)(c).

A defendant has been sued for theft by his former employer. At trial, the employer seeks to call the defendant's estranged daughter to testify. The daughter would testify that her brother told her, "I helped [the defendant] steal money from his employer." The brother is available to testify but would be a hostile witness. The defendant's attorney objects to the daughter's testimony on hearsay grounds. The employer argues that the statement is admissible under the hearsay exception for statements against interest. Should the court permit the daughter to testify about her brother's statement over the defendant's objection? A: No, because the brother is available to testify. B: No, because the hearsay exception for statements against penal interest applies only in criminal cases. C: Yes, because the statement exposes the brother to criminal liability. D: Yes, but only if corroborating circumstances clearly indicate the statement's trustworthiness.

A is correct. The exception to the hearsay rule for statements against interest allows a hearsay statement to be admissible under the theory that the statement is so prejudicial to the person making it that they would not have made the statement unless they believed it was true. However, this exception may be used to admit evidence only after a showing that the declarant is unavailable to testify. Here, the brother is available, so the statement against interest exception may not be used.

Several years ago, a carpenter purchased Goldacre, financing a large part of the purchase price by a loan from a bank that was secured by a mortgage. The carpenter made the installment payments on the mortgage regularly until last year. Then the carpenter persuaded a friend to buy Goldacre, subject to the mortgage to the bank. They expressly agreed that the friend would not assume and agree to pay the carpenter's debt to the bank. The carpenter's mortgage to the bank contained a due-on-sale clause stating, "If Mortgagor transfers his/her interest without the written consent of Mortgagee first obtained, then at Mortgagee's option the entire principal balance of the debt secured by this Mortgage shall become immediately due and payable." However, without seeking the bank's consent, the carpenter conveyed Goldacre to the friend, the deed stating in pertinent part ". . . subject to a mortgage to the bank [giving details and recording data]." The friend took possession of Goldacre and made several mortgage payments, which the bank accepted. Now, however, neither the friend nor the carpenter has made the last three mortgage payments. The bank has brought an appropriate action against the friend for the amount of the delinquent payments. In this action, judgment should be for A: the friend, because she did not assume and agree to pay the carpenter's mortgage debt. B: the friend, because she is not in privity of estate with the bank. C: the bank, because the carpenter's deed to the friend violated the due-on-sale clause. D: the bank, because the friend is in privity of estate with the bank.

A is correct. The express agreement that the friend would NOT assume liability for the mortgage insulated the friend from liability for the payments, even if she voluntarily paid them for a couple of months. Although the bank may end up foreclosing, its actionable claim is against the carpenter for the entire balance of the mortgage, not the friend. B is incorrect. Privity is not at issue in this question because the friend did not assume the mortgage. C is incorrect. The carpenter's violation of the clause simply gives the bank the right to demand immediate payment from the carpenter, not the friend. D is incorrect. Privity is irrelevant because the friend did not assume the mortgage, as stated above.

A buyer contracted in writing with a shareholder, who owned all of XYZ Corporation's outstanding stock, to purchase all of her stock at a specified price per share. At the time this contract was executed, the buyer's contracting officer said to the shareholder, "Of course, our commitment to buy is conditioned on our obtaining approval of the contract from our parent company." The shareholder replied, "Fine. No problem." The shareholder subsequently refused to consummate the sale on the ground that the buyer had neglected to request the parent company's approval of the contract, which was true. The parent company's chief executive officer, however, is prepared to testify that the parent company would have routinely approved the contract if requested to do so. The buyer can also prove that he has made a substantial sale of other assets to finance the stock purchase, although he admittedly had not anticipated any such necessity when he entered into the stock purchase agreement. If the buyer sues the shareholder for breach of contract, is the buyer likely to prevail? A: Yes, because the condition of the parent company's approval of the contract, being designed to protect only the buyer and the parent company, can be and has been waived by those entities. B: Yes, because the buyer detrimentally relied on the shareholder's commitment by selling off other assets to finance the stock purchase. C: No, because the express condition of the parent company's approval had not occurred prior to the lawsuit. D: No, because obtaining the parent company's approval of the contract was an event within the buyer's control and the buyer's failure to obtain it was itself a material breach of contract.

A is correct. The express condition in the buyer-shareholder contract relating to the parent company's approval affects the buyer's obligation, but not the shareholder's. The facts indicate that both the buyer and the parent company are ready and willing to consummate the sale, so it appears that those parties waived the non-occurrence of the condition. B is incorrect. Without any reason to expect the buyer to change position in reliance on the contract, then it would not be considered reasonably foreseeable by the shareholder. C is incorrect. The non-occurrence of the condition was waived by the buyer and the parent company. D is incorrect. The issue presented here is the effect of the non-occurrence of the condition on the shareholder's obligation to sell.

A plaintiff sustained personal injuries in a three-car collision caused by the concurrent negligence of a trucker and a bus driver. In the plaintiff's action for damages against the other two drivers, the jury apportioned the negligence 30% to the plaintiff, 30% to the trucker, and 40% to the bus driver. The plaintiff's total damages were $100,000. A state statute provides for a system of pure comparative negligence, joint and several liability of concurrent tortfeasors, and contribution based upon proportionate fault. If the plaintiff chooses to pursue the claim against the trucker alone, she will be entitled to collect at most A: $70,000 from the trucker, and then the trucker will be entitled to collect $40,000 from the bus driver. B: $30,000 from the trucker, and then the trucker will be entitled to collect $10,000 from the bus driver. C: $30,000 from the trucker, and then the trucker will be entitled to collect nothing from the bus driver. D: nothing from the trucker, because the percentage of fault for the trucker is not greater than that of the plaintiff.

A is correct. The facts indicate that the jury apportioned responsibility and that the statute allows contribution based on proportionate fault between jointly and severally liable tortfeasors in a pure comparative negligence state. Therefore, all this question requires is a little basic math. The plaintiff's damages were for $100,000. She was responsible for $30,000 of her own damages, leaving $70,000 that she could collect from the trucker alone under the joint and several liability statute; the statute makes each co-defendant liable for the entire amount of the award. She would then be unable to collect any other amount from any other defendant. The jury determined that the share of responsibility for the trucker was 30% or $30,000. Under the facts, the state allows contribution by proportionate fault; as a result, the trucker is entitled to contribution from the bus driver in the amount of $40,000.

A man became angry after he was unexpectedly laid off from his longtime job as a factory worker. The next day, he returned to the factory floor and indiscriminately fired shotgun rounds into the air. The man later testified, without contradiction, that he had not intended to kill anyone but had simply sought to exact revenge on the factory's owners by shutting down operations for the day. Unfortunately, one of the bullets ricocheted off the wall and killed the man's best friend. The crimes below are listed in descending order of seriousness. On these facts, what is the most serious offense of which the man properly could be convicted? A: Murder. B: Voluntary manslaughter. C: Involuntary manslaughter. D: Assault.

A is correct. The most serious offense for which the man could properly be convicted of is murder. At common law, murder is defined as the unlawful killing of another human being with malice aforethought. Malice aforethought exists where the defendant acted with one of the following mental states: (i) intent to kill; (ii) intent to inflict great bodily injury; (iii) reckless indifference to an unjustifiably high risk to human life (acting with a "depraved heart"); or (iv) intent to commit a felony (felony murder). The man could properly be convicted of murder, even though he lacked specific intent to kill, because his conduct created such a high risk of death and was so devoid of social utility that he could be found to have acted with a depraved heart.

A chemical company designed and built a large tank on its premises for the purpose of storing highly toxic gas. The tank developed a sudden leak and escaping toxic gas drifted into the adjacent premises where a neighbor lived. The neighbor inhaled the gas and died as a result. In a suit brought by the neighbor's personal representative against the chemical company, which of the following must be established if the claim is to prevail? A: The toxic gas that escaped from the chemical company's premises was the cause of the neighbor's death. B: The tank was built in a defective manner and the toxic gas that escaped from the chemical company's premises was the cause of the neighbor's death. C: The chemical company was negligent in designing the tank and the toxic gas that escaped from the chemical company's premises was the cause of the neighbor's death. D: The chemical company was negligent in designing the tank, the tank was built in a defective manner, and the toxic gas that escaped from the chemical company's premises was the cause of the neighbor's death.

A is correct. The operation of a storage facility that contains highly toxic chemicals is considered an abnormally dangerous activity. Ultra-hazardous or abnormally dangerous activities give rise to strict liability because the inherent danger or peculiar risk is unreasonably high when compared to its social utility, even in the absence of negligence and where all the proper precautions have been taken. Consequently, the neighbor's estate will prevail in a claim for damages against the chemical company as owner of the storage facility as long as it can be shown that the escaped gases caused the death; it is irrelevant whether the facility was defective or the chemical company was negligent in the operation of the property.

A landowner owned and occupied Blackacre, which was a tract of land improved with a one-family house. His friend orally offered him $50,000 for Blackacre, the fair market value, and he accepted. Because they were friends, they saw no need for attorneys or written contracts and shook hands on the deal. The friend paid the landowner $5,000 down in cash and agreed to pay the balance of $45,000 at an agreed closing time and place. Before the closing, the friend inherited another home and asked the landowner to return his $5,000. The landowner refused, and, at the time set for the closing, the landowner tendered a good deed to the friend and declared his intention to vacate Blackacre the next day. The landowner demanded that the friend complete the purchase. The friend refused. The fair market value of Blackacre has remained $50,000. In an appropriate action brought by the landowner against the friend for specific performance, if the landowner loses, the most likely reason will be that A: the agreement was oral. B: keeping the $5,000 is the landowner's exclusive remedy. C: the friend had a valid reason for not closing. D: the landowner remained in possession on the day set for the closing.

A is correct. The question asks you to analyze why the court would rule against the landowner. This is essentially a hypothetical, asking you IF the court did something, WHY did they do it. To tackle a question like this, you have to look at all the answer choices and decide which scenario would make the hypothetical situation stated in the call occur. The Statute of Frauds mandates that any contract for the sale of land must be in writing and signed by the party to be charged. In this case, the friend's reasons for refusing to close and the landowner's justifications for mandating performance are irrelevant because the sale contract was never memorialized in a writing. A land sale contract may never be oral, so this would be the reason the landowner loses. Part performance that unequivocally indicates that the parties have contracted for the sale of land may remove the contract out of the Statute of Frauds. What constitutes sufficient part performance varies among jurisdictions. The majority requires at least two of the following: (i) payment (in whole or in part); (ii) possession; and/or (iii) valuable improvements. In this case, there was a partial payment, but not possession or valuable improvements. Therefore, the Statute of Frauds still applies to this case. B is incorrect. The friend could sue for the entire value of the contract, or specific performance. C is incorrect. Having a valid reason for closing would not be enough to overcome the breach of contract. D is incorrect. The landowner remaining in possession of the property would not be enough to compel specific performance.

A defendant is on trial for theft. At trial, the prosecutor called a husband and wife. They testified that, as they looked out their apartment window, they saw thieves across the street break the window of a jewelry store, take jewelry, and leave in a car. The wife telephoned the police and relayed to them the license number of the thieves' car as her husband looked out the window with binoculars and read it to her. Neither of them has any present memory of the number. The prosecutor offers as evidence a properly authenticated police tape recording of the wife's telephone call with her voice giving the license number, which is independently shown to belong to the defendant's car. The tape recording of the wife stating the license is A: admissible, under the hearsay exception for present sense impressions. B: admissible, as nonhearsay circumstantial evidence. C: inadmissible, because it is hearsay not within any exception. D: inadmissible, because the wife never had firsthand knowledge of the license number.

A is correct. The recording satisfies the present sense impression exception to the hearsay rule because the wife was relaying the information in real-time, as reported by the husband while he was observing the event live.

A man rented a car from a car rental agency. Unbeknownst to the rental agency, the car had a bomb hidden in it at the time of the rental. The bomb exploded an hour later, injuring the man. Immediately prior to renting the car to the man, the rental agency had carefully inspected the car to be sure it was in sound operating condition. The rental agency did not inspect for hidden explosive devices, but such an inspection would have revealed the bomb. There had been no previous incidents of persons hiding bombs in rental cars. In a negligence action by the man against the car rental agency, is the man likely to prevail? A: No, because the rental agency could not have reasonably foreseen the likelihood of someone placing a bomb in the car it was about to rent to the man. B: No, because the rental agency did not hide the bomb in the car. C: Yes, because an inspection for explosive devices would have revealed the bomb. D: Yes, because the bomb made the car abnormally dangerous.

A is correct. The standard to be applied in a negligence action is whether the defendant acted with ordinary care. The presence of a bomb in a rental car is sufficiently unlikely that a reasonable rental agency would not routinely inspect for such a device. In the absence of evidence that the agency should have foreseen that there might be a bomb hidden in the car, the man cannot prove a negligence claim

A child was the illegitimate, unacknowledged child of the decedent. The decedent died intestate, leaving neither spouse nor any children other than the illegitimate child. The state's law of intestate succession provides that an unacknowledged illegitimate child may not inherit his father's property. The spouse, all other blood relations, and the state are preferred as heirs over the unacknowledged illegitimate child. The illegitimate child filed suit in an appropriate court alleging that the state statute barring an illegitimate child from sharing in a parent's estate is invalid and that he should be declared a lawful heir to his father's estate. In challenging the validity of the state statute, the illegitimate child's strongest argument would be that A: preferring collateral relatives and even the state over unacknowledged children is not substantially related to an important government interest, and, therefore, the law violates the Equal Protection Clause. B: he has been deprived of property without due process because his fundamental right to inherit has been compromised without a compelling state need. C: it violates the Privileges and Immunities Clause of the Fourteenth Amendment. D: it is a denial of procedural due process because it does not give the unacknowledged illegitimate child an opportunity to prove paternity.

A is correct. The state statute classifies on the basis of legitimacy, so it will be subject to intermediate scrutiny. Thus, the strongest argument is that the classification is not substantially related to an important government interest and therefore violates the Equal Protection Clause of the Fourteenth Amendment

A particular state has a state employee grievance system that requires any state employee who wishes to file a grievance against the state to submit that grievance for final resolution to a panel of three arbitrators chosen by the parties from a statewide board of 13 arbitrators. In any given case, the grievant and the state alternate in exercising the right of each party to eliminate five members of the board, leaving a panel of three members to decide their case. At the present time, the full board is composed of seven male arbitrators and six female arbitrators. A female state employee filed a sexual harassment grievance against her male supervisor and the state. The state's attorney exercised all of her five strikes to eliminate five of the female arbitrators. At the time she did so, the state's attorney stated that she struck the five female arbitrators solely because she believed women, as a group, would necessarily be biased in favor of another woman who was claiming sexual harassment. Counsel for the state employee eliminated four males and one female arbitrator, all solely on the grounds of specific bias or conflicts of interest. As a result, the panel was all male. When the panel ruled against the state employee on the merits of her case, she filed an action in an appropriate state court, challenging the panel selection process as a gender-based denial of equal protection of the laws. In this case, the court should hold that the panel selection process is A: unconstitutional, because the gender classification used by the state's attorney in this case does not satisfy the requirements of intermediate scrutiny. B: unconstitutional, because the gender classification used by the state's attorney in this case denies the grievant the right to a jury made up of her peers. C: constitutional, because the gender classification used by the state's attorney in this case satisfies the requirements of the strict scrutiny test. D: constitutional, because the gender classification used by the state's attorney in this case satisfies the requirements of the rational basis test.

A is correct. The state's attorney in this case intentionally excluded arbitrators based on gender. The Supreme Court has held that peremptory challenges based solely on gender are unconstitutional as a violation of the Equal Protection Clause because they reinforce negative stereotypes against women without furthering the governmental interest of a fair trial. See J.E.B. v. Alabama, 511 U.S. 127 (1994). Similarly, the state's attorney in this question is making a gender-based classification, so the state would need to show that the classification is substantially related to an important government objective. Because the state's attorney's argument was that women would simply be biased toward a female claimant, the attorney failed to demonstrate how the exclusion of female arbitrators would be substantially related to the need for a fair arbitration.

A landlord leased an apartment to a tenant by written lease for two years ending on the last day of a recent month. The lease provided for $700 in monthly rent. The tenant occupied the apartment and paid the rent for the first 15 months of the lease term, until he moved to another city to take a new job. Without consulting the landlord, the tenant moved a friend into the apartment and signed an informal writing transferring to the friend his "lease rights" for the remaining nine months of the lease. The friend made the next four monthly $700 rent payments to the landlord. For the final five months of the lease term, no rent was paid by anyone, and the friend moved out with three months left of the lease term. The landlord was on an extended trip abroad, and did not learn of the default and the vacancy until the end of the lease term. The landlord has sued the tenant and the friend, jointly and severally, for $3,500 for the last five months' rent. What is the likely outcome of the lawsuit? A: Both the tenant and the friend are liable for the full $3,500, because the tenant is liable on privity of contract and the friend is liable on privity of estate as assignee. B: The friend is liable for $1,400 on privity of estate, which lasted only until he vacated, and the tenant is liable for$2,100 on privity of contract and estate for the period after the friend vacated. C: The friend is liable for $3,500 on privity of estate, and the tenant is not liable, because the landlord's failure to object to the friend's payment of rent relieved the tenant of liability. D: The tenant is liable for $3,500 on privity of contract, and the friend is not liable, because a sublessee does not have personal liability to the original landlord.

A is correct. The tenant transferred the premises to the friend for the remainder of the lease term of nine months, which was an effective assignment. However, the tenant was not released by the landlord and thus remains liable on the privity of contract. B is incorrect. The privity of estate remained with the friend until the end of the lease term because that was the friend's assigned interest. The tenant remains liable for the full $3,500 on privity of contract because the landlord never granted a release. C is incorrect. Because the landlord never released the tenant, the tenant remains liable on privity of contract based on the original lease and can be jointly liable for the $3,500 with the friend. D is incorrect. This answer assumes that the friend was a sublessee, which he was not.

In the prosecution of a defendant for murdering a victim, the defendant testified that the killing had occurred in selfdefense when the victim tried to shoot him. In rebuttal, the prosecution seeks to call a witness, the victim's father, to testify that the day before the killing, the victim told her father that she loved the defendant so much she could never hurt him. The witness's testimony is A: admissible within the hearsay exception for statements of the declarant's then existing state of mind. B: admissible, because the victim is unavailable as a witness. C: inadmissible as hearsay not within any exception. D: inadmissible, because the victim's character is not at issue.

A is correct. The witness's testimony is being offered to show the victim's existing emotional and mental state when she made the statement and is thus admissible as an exception to the hearsay rule.

A shopper was riding on an escalator in a department store when the escalator stopped abruptly. The shopper lost her balance and fell down the escalator steps, sustaining injuries. Although the escalator had been regularly maintained by an independent contractor, the store's obligation to provide safe conditions for its invitees was nondelegable. The shopper has brought an action against the store for damages, and the above facts are the only facts in evidence. The store has moved for a directed verdict. Should the court grant the motion? A: No, because the finder of fact could infer that the escalator malfunction was due to negligence. B: No, because the store is strictly liable for the shopper's injuries. C: Yes, because an independent contractor maintained the escalator. D: Yes, because the shopper has not produced evidence of negligence.

A is correct. There is enough evidence here to support an inference of negligence on the part of the store or the contractor. A jury could find that the malfunction was due to the negligent installation, maintenance, or operation of the escalator; the store would be responsible for all these possible causes under the nondelegable duty doctrine.

An eight-year-old child went to the grocery store with her mother. The child pushed the grocery cart while her mother put items into it. The child's mother remained near the child at all times. Another customer in the store noticed the child pushing the cart in a manner that caused the customer no concern. A short time later, the cart the child was pushing struck the customer in the knee, inflicting serious injury. If the customer brings an action, based on negligence, against the child, the child's best argument in defense would be that A: The child exercised care commensurate with her age, intelligence, and experience. B: The child is not subject to tort liability. C: The child was subject to parental supervision. D: The customer assumed the risk that the child might hit the customer with the cart.

A is correct. This choice gives a child-appropriate negligence standard of care. The customer's claim for negligence will be allowed, but the child will only be held to the standard of care expected of "a reasonable child" of the same age, training, maturity, experience, and intelligence.

A man has sued a police officer, alleging that the officer violated the man's civil rights by using excessive force while arresting him. At trial, the officer admits having hit the man in the head with the butt of his gun, but contends that the force was necessary, because the man was resisting arrest. In support of his contention, the officer seeks to introduce evidence that the man had resisted arrest on three prior occasions during the last 10 years. Is this testimony regarding the man's conduct during the three prior arrests admissible? A: No, because evidence of the prior incidents constitutes impermissible character evidence. B: No, because the officer has not shown that the man was convicted in connection with the prior incidents. C: Yes, because the incidents in question are relevant evidence of the man's propensity for violence. D: Yes, because the incidents in question are sufficient to constitute a habit.

A is correct. This is a civil case, where use of prior conduct (the three arrests) is not permissible as evidence that the actor (the man) likely acted in accord with those past actions.

Under the authority of a federal voting rights statute, some states drew congressional districts in a manner calculated to increase the likelihood that members of historically disadvantaged minority racial groups would be elected. The U.S. Supreme Court declared these districts to be unconstitutional, as improper racial gerrymanders. In response to this ruling, Congress passed a new statute that explicitly denies the Supreme Court appellate jurisdiction over all future cases challenging the constitutionality of action taken under the authority of the federal voting rights statute. Which of the following is the most persuasive argument for the constitutionality of the new statute restricting the Supreme Court's appellate jurisdiction? A: Article III of the Constitution explicitly states that the Supreme Court's appellate jurisdiction is subject to such exceptions and regulations as Congress shall make. B: The constitutional principle of separation of powers authorizes Congress to pass statutes calculated to reduce the effects of Supreme Court decisions that interfere with the exercise of powers that have been delegated to the legislative branch. C: The establishment and apportionment of congressional districts directly affect interstate commerce, and the Constitution authorizes Congress to use its plenary authority over such commerce for any purpose it believes will promote the general welfare. D: The Fifteenth Amendment authorizes Congress to enforce the amendment's voting rights provisions by appropriate legislation, and Congress could reasonably determine that this restriction on the Supreme Court's appellate jurisdiction is an appropriate means to that end.

A is correct. This is the most persuasive argument because Congress has the authority to regulate and limit the Court's appellate jurisdiction pursuant to Article III and Ex parte McCardle.

A marksman bought a new rifle and wanted to try it out by doing some target shooting. He went out into the country to an area where he had previously hunted. Much to his surprise, he noticed that the area beyond a clearing contained several newly constructed houses that had not been there before. Between the houses there was a small playground where several children were playing. Nevertheless, the marksman nailed a paper target to a tree and went to a point where the tree was between himself and the playground. He then fired several shots at the target. One of the shots missed the target and the tree and hit and killed one of the children in the playground. The marksman was convicted of murder. He appealed, contending that the evidence was not sufficient to support a conviction of murder. The appellate court should A: affirm the conviction, because the evidence is sufficient to support a conviction of murder. B: reverse the conviction and remand for a new trial, because the evidence is not sufficient for murder but will support a conviction of voluntary manslaughter. C: reverse the conviction and remand for a new trial, because the evidence is not sufficient for murder but will support a conviction of involuntary manslaughter. D: reverse the conviction and order the case dismissed, because the evidence is sufficient only for a finding of negligence and negligence alone cannot support a criminal conviction.

A is correct. When determining the sufficiency of the evidence to sustain a charge, the appellate court considers the evidence in the light most favorable to the state. In this case, the marksman placed his target so that, if he missed, he would be shooting in the direction of the children; he fired numerous shots from a rifle in the direction of an occupied playground and killed a child. Even though he did not have the intent to kill, the evidence is sufficient for a jury to find that the marksman acted with extreme recklessness toward human life, which is sufficient for a finding of malice aforethought for the murder conviction. The appellate court should affirm the conviction of murder.

For 10 years, a vacationer and a neighbor have owned summer vacation homes on adjoining lots. A stream flows through both lots. As a result of a childhood swimming accident, the vacationer is afraid of water and has never gone close to the stream. The neighbor built a dam on her property that has completely stopped the flow of the stream to the vacationer's property. The dam unreasonably interferes with the use and enjoyment of the vacationer's property but was built in conformity with all applicable laws. In a suit by the vacationer against the neighbor, will the vacationer prevail? A: Yes, because the damming unreasonably interferes with the use and enjoyment of the vacationer's property. B: Yes, because the neighbor intended to affect the vacationer's property. C: No, because the vacationer made no use of the stream. D: No, because the dam was built in conformity with all applicable laws.

A is correct. While the call of the question does not provide a specific claim, the choices are aspects of nuisance. A landowner who causes a substantial, unreasonable interference with a neighbor's use or enjoyment of his property without a valid defense is liable for private nuisance. This rule also applies to flowing water, so an upstream owner may not stop the flow of water to a downstream property if it would substantially interfere with the use and enjoyment of the downstream property.

A woman sued her former employer in state court, asserting age and sex discrimination claims under both state and federal law. The woman's attorney had recently been embarrassed in court by the judge to whom the case was assigned. Wishing to avoid difficulties with the judge, the woman's attorney promptly removed the case to federal court on the basis of federal-question jurisdiction. The employer's attorney has timely moved to remand. How is the federal court likely to proceed? A: Remand the entire case. B: Remand the state claims but keep the federal claims. C: Retain the case to avoid the risk of bias and impropriety in having it proceed before a judge who has shown clear hostility toward the woman's attorney. D: Retain the case, because it was timely removed and the woman alleges federal claims.

A is the correct answer. A defendant can remove an action that could have originally been brought by the plaintiff in the federal courts. Only defendants can exercise the right of removal. In this case, the woman is the plaintiff. Therefore, her attorney may not file for removal. The case should be remanded for a defect in the removal.

1. At Defendant's murder trial, Defendant calls Witness as his first witness to testify that Defendant has a reputation in their community as a peaceable and truthful person. The prosecutor objects on the ground that Witness's testimony would constitute improper character evidence. The court should A) Admit the testimony as to peaceableness, but exclude the testimony as to truthfulness. B) Admit the testimony as to truthfulness, but exclude the testimony as to peaceableness. C) Admit the testimony as to both character traits. D) Exclude the testimony as to both character traits.

A) Admit the testimony as to peaceableness, but exclude the testimony as to truthfulness.

1. Plaintiff sued Defendant for breach of a commercial contract in which Defendant had agreed to sell Plaintiff all of Plaintiff's requirements for widgets. Plaintiff called Expert Witness to testify as to damages. Defendant seeks to show that Expert Witness had provided false testimony as a witness in his own divorce proceedings. This evidence should be A) Admitted only if elicited from Expert Witness on cross-examination. B) Admitted only if the false testimony is established by clear and convincing extrinsic evidence. C) Excluded, because it is impeachment on a collateral issue. D) Excluded, because it is improper character evidence.

A) Admitted only if elicited from Expert Witness on cross-examination.

1. A man was arrested and charged with involuntary manslaughter. The victim died after a head-on collision with the man's car. The prosecution claimed that the man fell asleep while operating his vehicle and drove into oncoming traffic, killing the victim. At trial, the prosecution called a witness to testify that she was visiting the man on the morning of the accident, and he told her, "I didn't get any sleep last night because of a terrible migraine headache." The man's attorney objects to the witness's testimony. How should the trial court rule on the testimony? A) It is admissible as non-hearsay B) It is admissible as a statement of declarant's mental or physical condition. C) It is admissible as a declaration against interest. D) It is inadmissible as hearsay not within any exception.

A) It is admissible as non-hearsay

1. A victim was found dead, apparently from a gunshot wound. A friend of the murder victim was charged with first-degree murder. At trial, the suspect offered an alibi defense during his case-in-chief. On cross-examination, the prosecutor asked the suspect if it was true that he had been convicted of misdemeanor for check-fraud two years earlier. The suspect's lawyer objected. How should the judge rule? A) She should overrule the objection, because the suspect's alleged conviction is admissible to show his tendency to make false statements. B) She should overrule the objection, because the probative value of the evidence outweighs the possibility of unfair prejudice against the suspect. C) She should sustain the objection, because the suspect's conviction was for a misdemeanor, not a felony. D) She should sustain the objection, because the conviction was for a non-violent crime.

A) She should overrule the objection, because the suspect's alleged conviction is admissible to show his tendency to make false statements.

1. A brother and a sister purchased land under a deed that conveyed title to them as joint tenants with right of survivorship. Common law joint tenancy is unmodified by statute in the jurisdiction. The purchase price was $50,000, of which the sister paid $10,000 and the brother paid $40,000. The sister later mortgaged her interest in the land. The brother then died testate, leaving his entire estate to a cousin. The sister later paid off her mortgage debt, and the mortgage was released. At the present time, who owns the land? A) The answer depends on whether the jurisdiction follows the lien theory or the title theory of mortgages. B) Title is entirely in the sister as the surviving joint tenant. C) Title is in the sister and the cousin as the equal tenants in common. D) Title is in the sister and the cousin as tenants in common, with the sister having a 20% interest and the cousin having an 80% interest.

A) The answer depends on whether the jurisdiction follows the lien theory or the title theory of mortgages.

1. A woman decided to steal a necklace that belonged to her neighbor. She knew where the neighbor kept the necklace because she had been in the neighbor's house on many occasions when the neighbor has taken off the necklace and put it away in a jewelry box in the bathroom. One night, the woman went to the neighbor's house. The neighbor was away and the house was dark. The woman opened the bathroom window, saw the jewelry box on the counter, and started to climb inside. As her leg cleared the window sill, the neighbor's cat let out a loud screech. Terrified, the woman bolted back outside and fled. The crimes below are listed in descending order of seriousness. What is the most serious crime committed by the woman? A- Burglary B- Attempted burglary C- Attempted larceny D- No crime

A- Burglary

1. Martha's high school teacher told her that she was going to receive a failing grade in history which would prevent her from graduating. Furious, she reported to the principal that the teacher had fondled her, and the teacher was fired. A year later, still unable to get work because of the scandal, the teacher committed suicide. Martha, remorseful, confessed that her accusation had been false. If Martha is charged with manslaughter, her best defense would be that she A- Committed no act that proximately caused the teacher's death. B- Did not intend to cause the teacher's death. C- Did not act with Malice. D- Acted under extreme emotional distress.

A- Committed no act that proximately caused the teacher's death.

1- A statute of the state of Orrington provides that assessments of real property for tax purposes must represent the "actual value" of the property. The Blue County Tax Commission, in making its assessments, has uniformly and consistently determined the "actual value" of real property solely by reference to the price at which the particular property was last sold. In recent years, the market values of real property in Blue County have been rising at the rate of 15% per year. Owner is required to pay real estate taxes on her home in Blue County that are 200% to 300% higher than those paid by many other owners of similar homes in similar neighborhoods in that county, even though the current market values of their respective homes and Owner's home are nearly identical. The reason the taxes on Owner's home are higher than those imposed on the other similar homes in similar neighborhoods is that she bought her home much more recently than the other owners and, therefore, it is assessed at a much higher "actual value" than their homes. Persistent efforts by Owner to have her assessment reduced or the assessments of the others raised by the Blue County Tax Commissions have failed. Owner has now filed suit against the Blue County Tax Commission, charging only that the tax assessment on her property is unconstitutional. The strongest constitutional argument to support Owner's claim is that the comparative overvaluation of the Owner's property by the Blue County Tax Commission in making tax assessments over time. A- Deprives Owner of the equal protection of the laws. B- Deprives Owner of a privilege or immunity of national citizenship. C- Constitutes a taking of private property for public use without just compensation. D- Constitutes an ex post facto law.

A- Deprives Owner of the equal protection of the laws.

1. Jack, a bank teller, was fired by Morgan, the president of the bank. Jack decided to take revenge against Morgan, but decided against attempting it personally, because he knew Morgan was protected around the clock by bank security guards. Jack knew that Chip had a violent temper and was very jealous. Jack falsely told Chip that Chip's wife, Elsie, was having an affair with Morgan. Enraged, Chip said, "What am I going to do?" Jack said, "If it were my wife, I'd just march into his office and rushed to the bank." Chip grabbed a revolver and rushed to the bank. He walked into the bank, carrying a gun in his hand. One of the security guards, believing a holdup was about to occur, shot and killed Chip. If charged with murder of Chip, Jack should be found A- Guilty, based upon extreme recklessness. B- Guilty, based upon transferred intent. C- Not guilty, because he did not intend for Chip to be shot by the security guard. D- Not guilty, because he did not shoot Chip and he was not acting in concert with the security guard.

A- Guilty, based upon extreme recklessness.

1. Jack, a bank teller, was fired by Morgan, the president of the bank. Jack decided to take revenge against Morgan, but decided against attempting it personally, because he knew Morgan was protected around the clock by bank security guards. Jack knew that Chip had a violent temper and was very jealous. Jack falsely told Chip that Chip's wife, Elsie, was having an affair with Morgan. Enraged, Chip said, "What am I going to do?" Jack said, "If it were my wife, I'd just march into his office and rushed to the bank." Chip grabbed a revolver and rushed to the bank. He walked into the bank, carrying a gun in his hand. One of the security guards, believing a holdup was about to occur, shot and killed Chip. If charged with attempted murder of Morgan, Jack should be found A- Guilty, because he intended to kill Morgan and used Chip to carry out his plan. B- Guilty, because he was extremely reckless as to Morgan. C- Not guilty, because Morgan was never in imminent danger of being killed. D- Not guilty, because Chip, if successful, would be guilty of no more than manslaughter and an accessory cannot be guilty of a higher crime than the principal.

A- Guilty, because he intended to kill Morgan and used Chip to carry out his plan.

1. Joe and Marty were coworkers. Joe admired Marty's wristwatch and frequently said how much he wished he had one like it. Marty decided to give Joe the watch for his birthday the following week. On the weekend before Joe's birthday, Joe and Marty attended a company picnic. Marty took his watch off and left it on a blanket when he went off to join in a touch football game. Joe strolled by, saw the watch on the blanket, and decided to steal it. He bent over and picked up the watch. Before he could pocket it, however, Marty returned. When he saw Joe holding the watch, he said, "Joe, I know how much you like that watch. I was planning to give it to you for your birthday. Go ahead and take it now." Joe kept the watch. Joe has committed A- Larceny B- Attempted larceny C- Embezzlement D- No crime

A- Larceny

1- A group of students at a state university's law school wished to debate the future of affirmative action in that state and at law school. For this debate they requested the use of a meeting room in the law school that is available on a first-come, first-served basis for extracurricular student use. Speakers presenting all sides of the issue were scheduled to participate. The law school administration refused to allow the use of any of its meeting rooms for this purpose solely because it believed that "such a debate, even if balanced, would have a negative effect on the morale of the law school community and might cause friction among the students that would disrupt the institution's educational mission." Is the refusal of the law school administration to allow the use of its meeting room for this purpose constitutional? A- No, because the law school administration cannot demonstrate that its action was necessary to vindicate a compelling state interest. B- No, because the law school administration cannot demonstrate that its action was rationally related to a legitimate state interest. C- Yes, because the law school administration's only concern was the adverse effect of such a discussion of affirmative action on the immediate audience and the mission of the institution. D- Yes, because the law students do not have a right to use a state-owned law school facility for a meeting that is not organized and sponsored by the law school itself.

A- No, because the law school administration cannot demonstrate that its action was necessary to vindicate a compelling state interest.

1. A man has four German shepherd dogs that he has trained for guard duty and that he holds for breeding purposes. The man has "Beware of Dogs" sigs clearly posted around a fenced-in yard where he keeps the dogs. The man's next-door neighbor frequently walks past the man's house and knows about the dogs' ferocity. One summer day, the neighbor entered the man's fenced-in yard to retrieve a snow shovel that the man had borrowed during the past winter. The neighbor was attacked by one of the dogs and was severely injured. In a suit against the man, is the neighbor likely to prevail? A- No, because the neighbor knew that the man had dangerous dogs in the yard. B- No, because the neighbor was trespassing when he entered the man's property. C- Yes, because the neighbor was an invitee for the purpose of retrieving the shovel. D- Yes, because the man was engaged in an abnormally dangerous activity.

A- No, because the neighbor knew that the man had dangerous dogs in the yard. talking about assumption of the risk

1) A homeowner hired an arsonist to set fire to the homeowner's house so that the homeowner could collect the insurance proceeds from the fire. After pouring gasoline around the house, the arsonist lit the fire with his cigarette lighter and then put the lighter in his pocket. As the arsonist was standing back admiring his work, the lighter exploded in his pocket. The arsonist suffered severe burns to his leg. After finding out that the explosion was caused by a manufacturing defect in the lighter, the arsonist brought an action against the manufacturer of the lighter based on strict product liability. Under applicable law, the rules of pure comparative fault apply in such actions. Will the arsonist prevail? A) Yes, because the lighter exploded as the result of a defect caused by manufacturing error. B) Yes, because the lighter was the proximate cause of the arsonist's injury. C) No, because the lighter was not being used for an intended or reasonably foreseeable purpose. D) No, because the arsonist was injured in the course of committing a felony by the device used to perpetrate the felony.

A. A) Yes, because the lighter exploded as the result of a defect caused by manufacturing error. Strict liability

The owner of Blackacre is an elderly, unmarried man with no children and no surviving relatives. Blackacre is a large property bordered by forests on the east side, a small stream on the west side, a large hill to the south, and a meadow to the north. There are neighbors to the east, west, and south. The owner is terminally ill and would like to leave Blackacre to the neighbor over the hill before he dies. The owner executed a deed conveying Blackacre "to my neighbor." The owner signed the deed, but the neighbor did not. Is the deed effective? A: No, because the grantee did not sign. B: No, because the grantee is not identified. C: Yes, because the grantor did sign. D: Yes, because the grantor and the grantee are identifiable.

B is correct. A deed must properly identify the grantor and grantee to be effective. A deed that does not identify the grantee is ineffective until the grantee's name can be added or determined. In this case, the conveyance "to my neighbor" is not specific enough. The owner has neighbors to the east, west, and south. Although the owner intended the grantee to be his neighbor to the south, the deed is ineffective. A is incorrect. A grantee does not have to sign a deed for it to be effective. Only a grantor is required to sign a deed. The deed is ineffective because the grantee is not properly identified. C is incorrect. The grantor's signature only is not sufficient for the deed to be effective. The deed must still include all the other requirements: the identity of the grantor and grantee, words of transfer, a description of the property interest being transferred, and the grantor's signature. D is incorrect. The deed is ineffective because there is no way of determining from the deed which neighbor is the intended grantee.

A proposed federal statute would prohibit all types of discrimination against Black persons on the basis of their race in every business transaction executed anywhere in the United States by any person or entity, governmental or private. Is this proposed federal statute likely to be constitutional? A: Yes, because it could reasonably be viewed as an exercise of Congress's authority to enact laws for the general welfare. B: Yes, because it could reasonably be viewed as a means of enforcing the provisions of the Thirteenth Amendment. C: No, because it would regulate purely local transactions that are not in interstate commerce. D: No, because it would invade the powers reserved to the states by the Tenth Amendment.

B is correct. A federal statute prohibiting discrimination on the basis of race in every business transaction could be justified as a means of enforcing the Thirteenth Amendment; it would be rational for Congress to conclude that such legislation serves to eradicate the badges and incidents of slavery.

A hockey fan had a season ticket for her home team's hockey games at the local arena (Section B, Row 12, Seat 16). During the intermission between the first and second periods of a game, the hockey fan solicited signatures for a petition urging that the coach of the hockey team be fired. The local arena and hockey team are owned by a privately owned entity. As evidenced by many prominently displayed signs, this entity prohibits all solicitations anywhere within the arena at any time and in any manner. The privately owned entity notified the hockey fan to cease her solicitation of signatures. The hockey fan continued to seek signatures on her petition during the hockey team's next three home games at the arena. Each time, the entity notified the hockey fan to cease such solicitation. The hockey fan announced her intention to seek signatures on her petition again during the hockey team's next home game at the arena. The entity wrote a letter informing her that her season ticket was canceled and tendered a refund for the unused portion. The hockey fan refused the tender and brought an appropriate action to establish the right to attend all home games. In this action, the court will decide for A: the privately owned entity, because it has a right and obligation to control activities on realty it owns and has invited the public to visit. B: the privately owned entity, because the hockey fan's ticket to hockey games created only a license. C: the hockey fan, because, having paid value for the ticket, her right to be present cannot be revoked. D: the hockey fan, because she was not committing a nuisance by her activities.

B is correct. A ticket is a mere license rather than a property interest. It gives the ticket holder a contractual right to use some portion of the issuer's property for a fixed period of time, but does not afford that ticket holder any interest in the property. While the license may be revoked by the issuer, that issuer will be liable for damages if it does so wrongfully. Further, because the ticket is a contractual right, a ticket holder's remedy for wrongful revocation of a ticket will be in contract, rather than property law. In this case, the hockey fan's ticket was a mere license issued by the privately owned entity. A is incorrect. The privately owned entity's right to revoke the specific license it issued to the hockey fan, rather than a general right to police its facility, is the basis for overcoming the hockey fan's claim. C is incorrect. The hockey fan did not, however, possess an irrevocable license. D is incorrect. The privately owned entity may revoke the hockey fan's license regardless of whether she is committing a nuisance.

In an automobile negligence action by a plaintiff against a defendant, a bystander testified for the plaintiff. The defendant later called a witness, who testified that the bystander's reputation for truthfulness was bad. On cross-examination of the witness, the plaintiff's counsel asks, "Isn't it a fact that when you bought your new car last year, you made a false affidavit to escape paying the sales tax?" This question is A: proper, because it will indicate the witness's standard of judgment as to reputation for truthfulness. B: proper, because it bears on the witness's credibility. C: improper, because character cannot be proved by specific instances of conduct. D: improper, because one cannot impeach an impeaching witness.

B is correct. As a general rule, character evidence used as propensity evidence is not admissible in civil cases unless character is directly at issue in the case. Character evidence that deals specifically with a witness's credibility may be admissible for impeachment. On cross-examination, a witness may be asked about prior bad acts of specific instances of conduct if they are probative of truthfulness. Here, the witness's creation of a false affidavit is probative of his character for truthfulness and bears on his credibility. The plaintiff's counsel's questioning of the witness is proper.

The plaintiff took a diamond ring to a pawnshop and borrowed $20 on it. It was agreed that the loan was to be repaid within 60 days and if it was not, the pawnshop owner, the defendant, could sell the ring. A week before expiration of the 60 days, the defendant had an opportunity to sell the ring to a customer for $125. He did so, thinking it unlikely that the plaintiff would repay the loan and if he did, the defendant would be able to handle him somehow, even by paying him for the ring if necessary. Two days later, the plaintiff came in with the money to reclaim his ring. The defendant told him that it had been stolen when his shop was burglarized one night and that therefore he was not responsible for its loss: Larceny, embezzlement, and false pretenses are separate crimes in the jurisdiction. It is most likely that the defendant has committed which of the following crimes? A: Larceny. B: Embezzlement. C: Larceny by trick. D: Obtaining by false pretenses.

B is correct. At common law, larceny is the taking possession and carrying away of the personal property of another, without the owner's consent and with the intent to permanently deprive the owner of said property. At common law, larceny by trick is the obtaining of possession, with the owner's consent, of the property of another by fraud or misrepresentation, with the intent to permanently deprive the owner of said property. At common law, false pretenses is obtaining the possession and title of property of another through fraud or misrepresentation, with the intent to permanently deprive the owner of the property. At common law, embezzlement is the fraudulent conversion of another person's property by someone who had lawful possession of said property.

A worker from State A sued a farmer from State B in a state court in State B, alleging that the farmer violated the worker's rights under Title VII of the Civil Rights Act of 1964. The farmer filed a notice of removal in the federal court of State B. The worker then filed a motion for remand to state court. Should the court grant the worker's motion to remand? A: No, because the federal claim was mistakenly filed in a state court. B: No, because the court has subject-matter jurisdiction based on a federal question. C: Yes, because the notice of removal should have been filed with the state court. D: Yes, because the farmer is a citizen of State B, the state where the action was brought

B is correct. Because the complaint alleges a violation of Title VII of the Civil Rights Act of 1964, a federal law, the farmer can remove to federal court in State B.

A tennis player from State A filed a diversity action in State B federal district court against a reporter from State B. The tennis player alleged that the reporter failed to pay a debt due on a valid contract. The reporter filed a motion to dismiss for improper venue, which the court denied. The reporter then filed an answer raising two affirmative defenses: (i) the court lacks personal jurisdiction; and (ii) the action is barred by the statute of limitations. What is the tennis player's strongest response to the reporter's answer? A: File a motion to strike both the lack of personal jurisdiction and the statute of limitations affirmative defenses, on the ground that they were waived. B: File a motion to strike only the reporter's affirmative defense for lack of personal jurisdiction, on the ground that it has been waived. C: File a motion to strike only the statute of limitations affirmative defense, on the ground that it has been waived. D: Continue with the case without filing a motion to strike any of the reporter's affirmative defenses.

B is correct. By filing the initial motion to dismiss without raising the lack of personal jurisdiction defense, the reporter waived the defense and consented to the personal jurisdiction. In contrast, a statute of limitations defense is not among the defenses listed in FRCP 12 that are waived if not made in a pre-answer motion.

A surfer from State A brought a diversity action against a hairdresser from State B in State B federal court. The surfer's single-count complaint sought damages for injuries he suffered from a car accident in State C when he was a passenger in the hairdresser's automobile. State C's guest statute provides a valid defense to the surfer's claim but neither State A nor State B has a guest statute. Which choice-of-law rules must the court use to determine whether State C's guest statute applies? A: State A's choice-of-law rules, because the surfer is from State A. B: State B's choice-of-law rules, because the federal court is located in State B. C: State C's choice-of-law rules, because the accident occurred in State C. D: Federal choice-of-law rules, because the suit is in federal court.

B is correct. Choice-of-law rules are considered substantive for Erie purposes, so a federal court sitting a diversity case must follow the choice-of-law rules of the state in which it sits. See Klaxon v. Stentor Electric Manufacturing Co., 313 U.S. 487 (1941). Here, the federal court is situated in State B, so it must use State B choice-of-law rules to determine whether State C's guest statute applies.

The Sports Championship Revenue Enhancement Act is a federal statute that was enacted as part of a comprehensive program to eliminate the federal budget deficit. That act imposed, for a period of five years, a 50% excise tax on the price of tickets to championship sporting events. Such events included the World Series, the Super Bowl, major college bowl games, and similar championship sports events. This federal tax is probably A: constitutional, because the compelling national interest in reducing the federal budget deficit justifies this tax as a temporary emergency measure. B: constitutional, because an act of Congress that appears to be a revenue raising measure on its face is not rendered invalid because it may have adverse economic consequences for the activity taxed. C: unconstitutional, because a 50% tax is likely to reduce attendance at championship sporting events and, therefore, is not rationally related to the legitimate interest of Congress in eliminating the budget deficit. D: unconstitutional, because Congress violates the equal protection component of the Fifth Amendment by singling out championship sporting events for this tax while failing to tax other major sporting events to which tickets are sold

B is correct. Even though the 50% tax is high, it is a proper use of congressional power to raise revenue through uniform taxation, despite any adverse economic effects that may result.

A defendant was charged with attempted murder of a victim in a sniping incident in which the defendant allegedly shot at the victim from a bush as the victim drove his car along an expressway. The prosecutor offers evidence that seven years earlier the defendant had fired a shotgun into a woman's house and that the defendant had once pointed a handgun at another driver while driving on the street. This evidence should be A: excluded, because such evidence can be elicited only during cross-examination. B: excluded, because it is improper character evidence. C: admitted as evidence of the defendant's propensity toward violence. D: admitted as relevant evidence of the defendant's identity, plan, or motive.

B is correct. Evidence of "prior bad acts" is inadmissible against a defendant when offered to prove that he is more likely to have committed the instant crime based on criminal propensity.

While walking on a public sidewalk, a pedestrian was struck by a piece of lumber that fell from the roof of a homeowner's house. The homeowner had hired a repairman to make repairs to his roof, and the lumber fell through due to negligence on the repairman's part. Assume that the homeowner exercised reasonable care in hiring the repairman, that the repairman was an independent contractor, and that public policy made a homeowner's duty to keep the sidewalk safe for pedestrian a nondelegable duty. If the pedestrian brings an action against the homeowner to recover damages for the injury caused to him by the repairman's negligence, will the pedestrian prevail? A: Yes, under the res ipsa loquitur doctrine. B: Yes, because the repairman's act was a breach of a nondelegable duty owed by the homeowner to the pedestrian. C: No, because the repairman was an independent contractor rather than the homeowner's servant. D: No, because the homeowner exercised reasonable care in hiring the repairman to do the repair.

B is correct. In general, an independent contractor is liable for his own torts. However, an exception exists where the contractor is carrying out an inherently dangerous activity or where there is a public policy consideration that makes the duty nondelegable. The facts indicate that the duty to make the sidewalk safe for pedestrians was a nondelegable duty.

Police officers received a tip that illegal drugs were being sold at a certain ground-floor apartment. They decided to stake out the apartment. The stakeout revealed that a significant number of people visited the apartment for short periods of time and then left. A man exited the apartment and started to walk briskly away. The officers grabbed the man and, when he struggled, wrestled him to the ground. They searched him and found a bag of heroin in one of his pockets. After discovering the heroin on the man, the officers decided to enter the apartment. They knocked on the door, which was opened by the woman who lived there. The officers asked if they could come inside, and the woman gave them permission to do so. Once inside, the officers observed several bags of heroin on the living room table. The woman has been charged with possession of the heroin found on the living room table. She has filed a pretrial motion to suppress the heroin on the ground that it was obtained by an illegal search and seizure. Should the woman's motion be granted? A: No, because the tip together with the heroin found in the man's pocket provided probable cause for the search. B: No, because the woman consented to the officers' entry. C: Yes, because the officers' decision to enter the apartment was the fruit of an illegal search of the man. D: Yes, because the officers did not inform the woman that she could refuse consent.

B is correct. In the absence of an exception, the Fourth Amendment requires that police have both probable cause and a search warrant before they are able to enter a private dwelling. In this case, no such warrant existed. However, police may conduct a valid warrantless search if they have voluntary and intelligent consent to do so. Here, the facts do not indicate that the police improperly obtained the woman's consent. The woman's consent justified the officers' entry. Once inside, the police properly seized the heroin because it was in plain view. Police may make a warrantless seizure when they are legitimately on the premises and discover evidence that is in plain view that they have probable cause to believe is evidence or a fruit or instrumentality of crime. The woman's consent gave the police legitimate authority to be inside the building, they discovered the heroin on a living room table, and the criminal nature of the heroin was readily apparent.

A man who had become very drunk left a bar and started to walk home. Another patron of the bar, who had observed the man's condition, followed him. The patron saw the man stumble and fall to the ground near an alley. The patron then began to pull out a gun but saw that the man had passed out in the gutter. The patron reached into the man's pocket, grabbed his wallet, and started to walk away. When the patron heard police officers approaching, he dropped the wallet and ran off. The crimes below are listed in descending order of seriousness. What is the most serious crime of which the patron properly could be convicted? A: Robbery. B: Larceny. C: Attempted robbery. D: Attempted larceny.

B is correct. Larceny requires a trespassory taking and carrying away of the personal property of another with intent to steal it. All the elements were satisfied here. The asportation (carrying away) element requires movement of only a slight distance, so it was satisfied here even though the patron ultimately discarded the wallet.

A plaintiff suffered from a serious, though not immediately life-threatening impairment of his circulatory system. The plaintiff's cardiologist recommended a cardiac bypass operation and referred the plaintiff to a surgeon. The surgeon did not inform the plaintiff of the 2% risk of death associated with this operation. The surgeon defended his decision not to mention the risk statistics to the plaintiff because the plaintiff "was a worrier and it would significantly lessen his chance of survival to be worried about the nonsurvival rate." The surgeon successfully performed the bypass operation and the plaintiff made a good recovery. However, when the plaintiff learned of the 2% risk of death associated with the operation, he was furious that the surgeon had failed to disclose this information to him, saying that he would have refused the operation if he had known of the risk. If the plaintiff asserts a claim against the surgeon based on negligence, will the plaintiff prevail? A: No, because the surgeon used his best personal judgment in shielding the plaintiff from the risk statistic. B: No, because the operation was successful and the plaintiff suffered no harm. C: Yes, because the plaintiff would have refused the operation had he been informed of the risk. D: Yes, because a patient must be told the risk factors associated with a surgical procedure in order to give informed consent.

B is correct. Negligence (including medical malpractice) requires proof of duty, breach, causation, and damages. It is not a dignitary tort. If the plaintiff did not suffer an actual injury, he cannot prevail in an action for negligence against his doctor

A man and a woman, both purse snatchers, independently entered a grocery store, each with the intent to steal purses from unsuspecting shoppers. The man approached a victim, grabbed the strap of the purse that was hanging from her shoulder, and pulled. The victim screamed and tried to hold on to the purse, but the man knocked her down, causing her to lose her grip on the purse. The man then escaped with the purse. During the commotion that arose when the man's victim screamed, the woman approached the shopping cart of a shopper who had turned toward the noise and wandered away from her cart. The woman reached into the nowunattended shopping cart, grabbed the shopper's purse, and quickly walked away, also escaping with a purse. Both purse snatchers were apprehended a short while later. Which of the two purse snatchers could properly be convicted of robbery? A: Both the man and the woman. B: Only the man. C: Only the woman. D: Neither the man nor the woman.

B is correct. Only the man can be convicted of robbery because only he used force to steal the purse.

When a man entered a bank and presented a check for payment, the bank teller recognized the signature on the check as a forgery because the check was drawn on the account of a customer whose handwriting she knew. The bank teller called the police. Before the police arrived, the man picked up the check from the counter and left. The man was charged with attempting to cash a forged check. At trial, the prosecutor has called the bank teller to testify that the signature on the check was forged. Is the bank teller's testimony admissible? A: Yes, because a bank teller is by occupation an expert on handwriting. B: Yes, because it is rationally based on the bank teller's perception and is helpful to the jury. C: No, because the bank teller was at fault in allowing loss of the original by failing to secure the check. D: No, because it is not possible for either the jury or an expert to compare the signature on the missing check with a signature established as genuine.

B is correct. Rule 701 of the Federal Rules of Evidence (FRE) allows lay opinion testimony when it is rationally based on the perception of the witness and is helpful to the jury. Here the teller knew the signature of the bank customer on whose account the check was drawn. This knowledge made it possible for her to recognize the signature on the check as a forgery. Her testimony that the signature was a forgery is evidence that the signature on the check presented was different from the signature of the owner of the account (a signature she knows). Obviously the owner of the account would be a stronger prosecution witness than the teller in establishing that the signature was forged, but this doesn't mean that the teller would not be allowed to testify.

Congress passes an act requiring that all owners of bicycles in the United States register them with a federal bicycle registry. The purpose of the law is to provide reliable evidence of ownership to reduce bicycle theft. No fee is charged for the registration. Although most stolen bicycles are kept or resold by the thieves in the same cities in which the bicycles were stolen, an increasing number of bicycles are being taken to cities in other states for resale. Is this act of Congress constitutional? A: Yes, because Congress has the power to regulate property for the general welfare. B: Yes, because Congress could determine that, in inseverable aggregates, bicycle thefts affect interstate commerce. C: No, because most stolen bicycles remain within the state in which they were stolen. D: No, because the registration of vehicles is a matter reserved to the states by the Tenth Amendment.

B is correct. The Commerce Clause empowers Congress to regulate economic or commercial intrastate activity that, in the aggregate, has a substantial effect on interstate commerce. Because the cumulative resale of stolen bikes could rationally affect interstate commerce, the act is constitutional.

The defendant visited a fellow college student in the student's dormitory room. They drank some beer. The student produced a box containing marijuana cigarettes and asked if the defendant wanted one. The defendant, afraid of being caught, declined and urged the student to get rid of the marijuana. The student refused. Shortly thereafter, both went out to get more beer, leaving the door to the student's room standing open. Making an excuse about having dropped his pen, the defendant went back into the student's room. Still apprehensive about their being caught with the marijuana cigarettes, he took the cigarettes and flushed them down the toilet. He was sure the student was too drunk to notice that the cigarettes were missing. The defendant is charged with larceny and burglary (defined in the jurisdiction as breaking and entering the dwelling of another with intent to commit any felony or theft). He should be found guilty of A: burglary only. B: larceny only. C: both burglary and larceny D: neither burglary nor larceny.

B is correct. The defendant took the property of another (the student's marijuana) without consent and with the intent to permanently deprive the owner of the property. It is irrelevant that the owner's possession of the property was likely illegal. Thus, the defendant should be found guilty of larceny. However, the defendant is not guilty of burglary. He had the student's consent to be in the dorm room, which was not limited in any way. The door was open when they left, so no actual physical breaking occurred, nor was there a constructive breaking because the consent was not limited or taken away at any point. Even though the defendant made up an excuse to go back into the room, he was still operating under the consent previously given by the student. Because there was consent, and thus no trespass, no breaking occurred and he is not guilty of burglary.

A defendant was charged with battery for allegedly attacking a man after the two of them left a local bar together. No one else witnessed the incident. At trial, each testified that he had acted only in self-defense. The defendant has called his next-door neighbor as a witness to testify to the defendant's reputation both for truthfulness and for peacefulness. The government has objected to the testimony in its entirety. How should the court proceed? A: Admit the evidence in its entirety. B: Admit the evidence regarding the defendant's reputation for peacefulness, but exclude the evidence regarding his truthfulness. C: Exclude the evidence regarding the defendant's reputation for peacefulness, but admit the evidence regarding his truthfulness. D: Exclude the evidence in its entirety.

B is correct. The defendant's character for peacefulness is a pertinent trait to the battery charge (a crime of violence), and he may offer evidence in the form of his reputation for peacefulness. The evidence of the defendant's truthfulness must be excluded, however, because there is no indication that the defendant's character for truthfulness while testifying has been questioned, nor is it relevant to guilt or innocence.

At a party, the defendant and the victim agreed to play a game they called "spin the barrel." The victim took an unloaded revolver, placed one bullet in the barrel, and spun the barrel. The victim then pointed the gun at the defendant's head and pulled the trigger once. The gun did not fire. The defendant then took the gun, pointed it at the victim, spun the barrel, and pulled the trigger once. The gun fired, and the victim fell over dead. A statute in the jurisdiction defines murder in the first degree as an intentional and premeditated killing or one occurring during the commission of a common-law felony, and murder in the second degree as all other murder at common law. Manslaughter is defined as a killing in the heat of passion upon an adequate legal provocation or a killing caused by gross negligence. The most serious crime for which the defendant can properly be convicted is A: murder in the first degree, because the killing was intentional and premeditated and, in any event, occurred during commission of the felony of assault with a deadly weapon. B: murder in the second degree, because the defendant's act posed a great threat of serious bodily harm. C: manslaughter, because the defendant's act was grossly negligent and reckless. D: no crime, because the victim and the defendant voluntarily agreed to play a game and each assumed the risk of death.

B is correct. The defendant's pointing a loaded weapon at the victim and pulling the trigger posed a great threat of serious bodily injury to the victim, and, because it resulted in the victim's death, the defendant should be convicted of murder in the second degree. Murder in the second degree is murder with malice aforethought, done without premeditation and deliberation. The defendant's actions were done with reckless indifference to an unjustifiably high risk to human life and will support a finding of malice aforethought.

A defendant was prosecuted for the murder of a victim, whose body was found one morning in the street near the defendant's house. The state calls a witness, a neighbor, to testify that during the night before the body was found he heard the defendant's wife scream, "You killed him! You killed him!" The witness's testimony is A: admissible, as a report of a statement of belief. B: admissible, as a report of an excited utterance. C: inadmissible, because it reports a privileged spousal communication. D: inadmissible on spousal immunity grounds, but only if the wife objects.

B is correct. The defendant's wife's statement is an out-of-court statement that is being offered for the truth of the matter asserted - that the defendant killed the victim - so it is hearsay. However, the statement related to a startling event or condition made while the wife was under the stress of excitement caused by the event or condition, and is thus admissible as an excited utterance. Therefore, the witness's testimony is admissible.

An owner owned 80 acres of land, fronting on a town road. Two years ago, the owner sold to a buyer the back 40 acres. The 40 acres sold to the buyer did not adjoin any public road. The owner's deed to the buyer expressly granted a right-of-way over a specified strip of the owner's retained 40 acres, so the buyer could reach the town road. The deed was promptly and properly recorded. Last year, the buyer conveyed the back 40 acres to a doctor. They had discussed the right-of-way over the owner's land to the road, but the buyer's deed to the doctor made no mention of it. The doctor began to use the right-of-way as the buyer had, but the owner sued to enjoin such use by the doctor. The court should decide for A: the doctor, because he has an easement by implication. B: the doctor, because the easement appurtenant passed to him as a result of the buyer's deed to him. C: the owner, because the buyer's easement in gross was not transferable. D: the owner, because the buyer's deed failed expressly to transfer the right-of-way to the doctor.

B is correct. The easement described in the facts is an easement appurtenant. A is incorrect. While this answer is legally correct, it is not the best choice. The easement appurtenant would have passed to the buyer even if he did not have an easement by implication. C is incorrect. The buyer did not have an easement in gross. D is incorrect. An easement appurtenant does not need to be identified in any deeds accompanying later conveyances

A woman owned Woodsedge, a tract used for commercial purposes, in fee simple and thereafter mortgaged it to a bank. She signed a promissory note secured by a duly executed and recorded mortgage. There was no "due on sale" clause, that is, no provision that, upon sale, the whole balance then owing would become due and owing. The woman conveyed Woodsedge to a friend "subject to a mortgage to the bank, which the grantee assumes and agrees to pay." The friend conveyed Woodsedge to his son "subject to an existing mortgage to the bank." A copy of the note and the mortgage that secured it had been exhibited to each grantee. After the son made three timely payments, no further payments were made by any party. In fact, the real estate had depreciated to a point where it was worth less than the debt. There is no applicable statute or regulation. In an appropriate foreclosure action, the bank joined the woman, her friend, and the son as defendants. At the foreclosure sale, although the fair market value for Woodsedge in its depreciated state was obtained, a deficiency resulted. The bank is entitled to collect a deficiency judgment against A: the woman only. B: the woman and the friend only. C: the friend and his son only. D: the woman, the friend, and the son.

B is correct. The friend assumed the responsibilities for the mortgage so she is personally liable. The woman did not receive a novation from the bank releasing her from liability, so she is still personally liable on the mortgage, even though she has transferred it to the friend. A is incorrect. While the woman is still liable because she did not receive a novation, this is not a complete answer because the friend is also liable on the mortgage. C is incorrect. The friend is liable for the mortgage but the son is not. The son took the property subject to the mortgage, so he is not personally liable. D is incorrect. The woman and the friend are liable for the mortgage, but as explained above, the son took subject to the mortgage so he is not personally liable.

A builder purchased a large tract of land intending to construct residential housing on it. The builder hired a contractor to build a large in-ground swimming pool on the tract. The contract provided that the contractor would carry out blasting operations that were necessary to create an excavation large enough for the pool. The blasting caused cracks to form in the walls of the plaintiff's home in a nearby residential neighborhood. In the plaintiff's action for damages against the builder, the plaintiff should A: prevail, only if the builder retained the right to direct and control the contractor's construction of the pool. B: prevail, because the blasting that the contractor was hired to perform damaged the plaintiff's home. C: not prevail, if the contractor used reasonable care in conducting the blasting operations. D: not prevail, if the builder used reasonable care to hire a competent contractor.

B is correct. The general rule of an employer's non-liability for torts of his independent contractor does not apply to abnormally dangerous work, which is considered non-delegable and triggers strict liability. As such, the builder will be liable for the damage caused by the contractor's blasting.

A driver negligently ran into a pedestrian who was walking along a road. The pedestrian sustained an injury to his knee, causing it to buckle from time to time. Several months later, the pedestrian sustained an injury to his shoulder when his knee buckled, causing him to fall down a flight of stairs. The pedestrian then brought an action against the driver for the injuries to his knee and shoulder. In his action against the driver, for which of his injuries may the pedestrian recover damages? A: For the injuries to his knee and shoulder, because the driver takes the victim as he finds him. B: For the injuries to his knee and shoulder, if the jury finds that the pedestrian's fall down a flight of stairs was a normal consequence of his original injury. C: For the injury to his knee only, because the injury to the pedestrian's shoulder is separable. D: For the injury to his knee only, if the jury finds that the driver could not have foreseen that his negligent driving would cause the pedestrian to fall down a flight of stairs.

B is correct. The jury's finding that the fall was a normal consequence of the original injury will allow for the analysis to proceed to a determination of the scope of recoverable damages. Negligent defendants are liable for the full extent damages, regardless of foreseeability

A driver negligently ran into a pedestrian who was walking along a road. The pedestrian sustained an injury to his knee, causing it to buckle from time to time. Several months later, the pedestrian sustained an injury to his shoulder when his knee buckled, causing him to fall down a flight of stairs. The pedestrian then brought an action against the driver for the injuries to his knee and shoulder. In his action against the driver, for which of his injuries may the pedestrian recover damages? A: For the injuries to his knee and shoulder, because the driver takes the victim as he finds him. B: For the injuries to his knee and shoulder, if the jury finds that the pedestrian's fall down a flight of stairs was a normal consequence of his original injury. C: For the injury to his knee only, because the injury to the pedestrian's shoulder is separable. D: For the injury to his knee only, if the jury finds that the driver could not have foreseen that his negligent driving would cause the pedestrian to fall down a flight of stairs.

B is correct. The jury's finding that the fall was a normal consequence of the original injury will allow for the analysis to proceed to a determination of the scope of recoverable damages. Negligent defendants are liable for the full extent damages, regardless of foreseeability.

The aged mother of a sister and brother, both adults, wished to employ a live-in companion so that she might continue to live in her own home. Their mother, however, had only enough income to pay one-half of the companion's $2,000 monthly salary. Learning of their mother's plight, the siblings agreed with each other in a signed writing that on the last day of January and each succeeding month during their mother's lifetime, each would give their mother $500. Their mother then hired the companion. The siblings made the agreed payments in January, February, and March. In April, however, the brother refused to make any payment and notified his sister and mother that he would make no further payments. Will their mother succeed in an action for $500 brought against the brother after April 30? A: Yes, because by making his first three payments, the brother confirmed his intent to contract. B: Yes, because the mother is an intended beneficiary of a contract between the siblings. C: No, because a parent cannot sue her child for breach of a promise for support. D: No, because the siblings intended their payment to their mother to be gifts.

B is correct. The mother was not a party to the contract between the sister and the brother, so the only way for her to have contractual rights is if she was the intended beneficiary of the contract. For the mother to be an intended beneficiary, she must be the person to whom performance is to be given. In this case, several factors indicate that the mother is an intended beneficiary: the mother is named in the contract, the mother directly receives the payments, and the mother has a close familial relationship with the parties. Therefore, the mother is an intended beneficiary and will prevail in a suit against the brother for the $500 payment. A is incorrect. Modern contract law is based upon the objective test, which does not consider a party's private intent. The brother's intent is irrelevant. Further, whether a contract was formed is not at issue. The issue is whether the mother has contractual rights that may be asserted. C is incorrect. The mother is simply asserting her rights as an intended beneficiary to the contract; her claim is not solely based on her relationship to the son. Furthermore, there is no rule of contract law which states that a parent may not sue a child. D is incorrect. The promise to pay $500 was made in exchange for a mutual promise to pay the other $500. Because consideration was present, a contract was formed, and the $500 payments will not be considered gifts.

A department store had experienced a growing incidence of shoplifting. At the store's request, the police concealed an undercover detective at a vantage point above the women's apparel fitting rooms where she could see into these rooms, where customers tried on clothes. The detective saw a customer enter a fitting room, stuff a dress into her pocketbook, leave the fitting room, and start for the street door. By prearranged signal, the detective notified another police officer near the door, who detained the customer as the customer started to go out into the street. The customer was placed under arrest, and the dress was retrieved from her purse. The customer is charged with shoplifting. Her motion to prevent the introduction of the dress into evidence will be A: granted, because the police should have secured a search warrant to search her bag. B: granted, because a customer has a reasonable expectation of privacy while using a department store fitting room. C: denied, because the search and seizure were made incident to a valid arrest based on probable cause. D: denied, because the detective could see into the room and thus the customer's activities were legitimately in plain view.

B is correct. The motion to suppress evidence will be granted if the customer can show that she was subject to an illegal search by the state. The Fourth Amendment protects people from unreasonable searches and seizures by the state. The first issue in this Fourth Amendment analysis is to determine if there was a search or seizure. By watching the customer in the fitting room, the detective was conducting a search. The customer had a reasonable expectation of privacy in a department store fitting room, which was private and could only be seen by the detective when she was concealed at a vantage point above the rooms. A person has a reasonable expectation that they will not be watched in a place that is closed from public view such as the fitting room. The customer had an expectation of privacy in the fitting room, and that expectation is one that society has recognized as justified. The second issue to determine is whether the search was conducted by a state actor. The detective was working for the police department which was conducting surveillance. The detective was clearly a state actor. Finally, there is no exception to the warrant requirement that would allow for the search. The fitting room was not in plain view; the detective had to conceal herself above the fitting room to be able to see in, which is not a place visible in plain view. Therefore, the customer's motion to suppress should be granted because her reasonable expectation of privacy was violated by the detective's search

Kelly County, in State A, is located adjacent to the border of State B. The communities located in Kelly County are principally suburbs of Scarletville, a large city located in State B, and therefore there is a large volume of traffic between that city and Kelly County. While most of that traffic is by private passenger automobiles, some of it is by taxicabs and other kinds of commercial vehicles. An ordinance of Kelly County, the stated purpose of which is to reduce traffic congestion, provides that only taxicabs registered in Kelly County may pick up or discharge passengers in the county. The ordinance also provides that only residents of Kelly County may register taxicabs in that county. Which of the following is the proper result in a suit brought by Scarletville taxicab owners challenging the constitutionality of this Kelly County ordinance? A: Judgment for Scarletville taxicab owners, because the fact that private passenger automobiles contribute more to the traffic congestion problem in Kelly County than do taxicabs indicates that the ordinance is not a reasonable means by which to solve that problem. B: Judgment for Scarletville taxicab owners, because the ordinance unduly burdens interstate commerce by insulating Kelly County taxicab owners from out-of-state competition without adequate justification. C: Judgment for Kelly County, because the ordinance forbids taxicabs registered in other counties of State A as well as in states other than State A to operate in Kelly County and, therefore, it does not discriminate against interstate commerce. D: Judgment for Kelly County, because Scarletville taxicab owners do not constitute a suspect class and the ordinance is reasonably related to the legitimate governmental purpose of reducing traffic congestion.

B is correct. The ordinance classifies on the basis of county residency: only residents of Kelly County can register taxicabs there, and only registered taxicabs can pick up or drop off passengers in the county. The ordinance effectively gives Kelly residents a monopoly over the taxicab business in the guise of reducing congestion. This monopoly burdens interstate commerce, and because there is no limit on the number of taxicabs that can be registered, it is difficult to see how the ordinance will reduce congestion. Thus, the ordinance impermissibly burdens interstate commerce without adequate justification.

Six years ago, the owner of Blackacre in fee simple executed and delivered to a widower an instrument in the proper form of a warranty deed, purporting to convey Blackacre to "[the widower] and his heirs." At that time, the widower had one child. Three years ago, the widower executed and delivered to a buyer an instrument in the proper form of a warranty deed, purporting to convey Blackacre to "[the buyer]." The child did not join in the deed. The buyer was and still is unmarried and childless. The only possibly applicable statute in the jurisdiction states that any deed will be construed to convey the grantor's entire estate, unless expressly limited. Last month, the widower died, never having remarried. His child is his only heir. Blackacre is now owned by A: the child, because the widower's death ended the buyer's life estate pur autre vie. B: the buyer in fee simple pursuant to the widower's deed. C: the child and the buyer as tenants in common of equal shares. D: the child and the buyer as joint tenants, because both survived the widower.

B is correct. The owner conveyed Blackacre to the widower in fee simple. The language of the conveyance indicating that it was made to "[the widower] and his heirs" does not create an interest in the child. Because the widower holds the land in fee simple, his ownership will not be terminated by the happening of any subsequent condition, and he is free to transfer the land to whomever he wishes (there are no restraints on alienation). A is incorrect. There is nothing in the facts indicating that the buyer had a life estate. C is incorrect. The widower conveyed the entire interest of Blackacre to the buyer. There is no interest left for the child to share as a tenant in common, because the widower conveyed Blackacre in fee simple to the buyer. D is incorrect. There are no restrictions on the widower's right to re-convey the land, his conveyance to the buyer in fee simple will be upheld, and the child has no rights in the property. The widower held Blackacre in fee simple and was free to convey it as he wished.

A property owner and a landscape architect signed a detailed writing in which the landscape architect agreed to landscape the owner's residential property in accordance with a design prepared by the landscape architect and incorporated in the writing. The owner agreed to pay $10,000 for the work upon its completion. The owner's spouse was not a party to the agreement and had no ownership interest in the premises. Shortly before the agreement was signed, the owner and landscape architect orally agreed that the writing would not become binding on either party unless the owner's spouse should approve the landscaping design. If the owner's spouse disapproves of the design and the owner refuses to allow the landscape architect to proceed with the work, is evidence of the oral agreement admissible in the landscape architect's action against the owner for breach of contract? A: Yes, because the oral agreement required approval by a third party. B: Yes, because the evidence shows that the writing was intended to take effect only if the approval occurred. C: No, because the parol evidence rule bars evidence of a prior oral agreement even if the latter is consistent with the terms of a partial integration. D: No, because the prior oral agreement contradicted the writing by making the parties' duties conditional.

B is correct. The parol evidence rule bars extrinsic evidence of a prior agreement either where the prior agreement contradicts the terms of a final written agreement or where the prior agreement purports to add to a completely integrated agreement (i.e., one that is intended by the parties to be both the final and exclusive manifestation of the parties' understanding). An exception to the parol evidence rule applies where the extrinsic evidence is offered to prove the existence of a condition precedent to the written agreement. Therefore, evidence of the owner and landscape architect's oral agreement is admissible because it demonstrated the existence of a condition precedent to the writing's taking effect. A is incorrect. The oral agreement would not be admissible unless it fits within one of the exceptions to the parol evidence rule. C is incorrect. This parol evidence rule does not bar evidence of a prior oral agreement when that evidence shows a condition precedent to the binding agreement. D is incorrect. The prior oral agreement falls within the exception of the parol evidence rule.

An injured man from State A brought a products liability action against a manufacturer and a retailer in a state court in State A. The man claimed a drug he purchased from the retailer, created by the manufacturer, caused him to suffer $100,000 in damages. The manufacturer is incorporated in State B, with its sole manufacturing plant in State C, and its board of directors meets in State A to direct and control the corporation's activities. The retailer is a limited liability company with its principal place of business in State A and its owner-members are citizens of State D. The manufacturer and the retailer timely removed the case to the federal district court in State A. The injured man timely moved to remand the case back to state court. The district court is extremely busy with criminal cases and civil cases that arise under federal law. Is the district court likely to grant the man's motion to remand? A: Yes, because either the manufacturer or the retailer, or both, are citizens of the state in which the action was brought, and venue is therefore improper. B: Yes, because the man is not diverse from both the manufacturer and the retailer, and the court lacks any other basis for subject-matter jurisdiction. C: No, because the district court may exercise its discretion to retain the case instead of conserving its resources for actions arising under federal law. D: No, because all of the requirements for removal have been satisfied, including personal jurisdiction, subject-matter jurisdiction, and venue.

B is correct. The parties are not completely diverse, which means the federal court does not have subject-matter jurisdiction over the case. A corporation's domicile may be in its principal place of business and its state of incorporation. Here, the manufacturer's board of directors meets in State A to direct and control its activities, which establishes domicile in State A, in addition to State B, where it is incorporated. The injured man is from State A, which means the court does not have subject-matter jurisdiction based on diversity because the manufacturer is also domiciled in State A. Without complete diversity or federal question jurisdiction, the federal court is likely to grant the motion to remand back to state court.

A driver negligently drove his car into a pedestrian, breaking her leg. The pedestrian's leg was put in a cast, and she used crutches to get around. While shopping at her local supermarket, the pedestrian non-negligently placed one of her crutches on a banana peel that had been negligently left on the floor by the manager of the supermarket's produce department. The pedestrian's crutch slipped on the peel, and she fell to the floor, breaking her arm. Had the pedestrian stepped on the banana peel at a time when she did not have to use crutches, she would have regained her balance. The pedestrian sued the driver and the supermarket for her injuries. The pedestrian will be able to recover from A: the driver, for her broken leg only. B: the driver, for both of her injuries. C: the supermarket, for both of her injuries. D: the driver, for her broken leg only, and the supermarket, for her broken arm only.

B is correct. The pedestrian's broken arm is proximately connected to the driver's negligent driving. Courts generally hold that subsequent ordinary negligence actions, which may add to the plaintiff's injuries, are a foreseeable consequence of the original action and will not break the chain of causation. The facts clearly state that the pedestrian would not have lost her balance had she not been on crutches as the result of being hit by the driver. Because it is foreseeable that walking on crutches would make the pedestrian less stable on her feet and more vulnerable to subsequent injury from falling, the driver is the proximate (legal) cause of both injuries (although responsibility for the broken arm alone may be apportioned between the driver and the supermarket as concurrent tortfeasors).

An owner conveyed Blackacre to a buyer by a warranty deed. The buyer recorded the deed four days later. After the conveyance but prior to the buyer's recording of the deed, a lender properly filed a judgment against the owner. The two pertinent statutes in the jurisdiction provide the following: 1) any judgment properly filed shall, for 10 years from filing, be a lien on the real property then owned or subsequently acquired by any person against whom the judgment is rendered; and 2) no conveyance or mortgage of real property shall be good against subsequent purchasers for value and without notice unless the same be recorded according to law. The recording act has no provision for a grace period. The lender joined both the owner and the buyer in an appropriate action to foreclose the judgment lien against Blackacre. If the lender is unsuccessful, it will be because A: the owner's warranty of title to the buyer defeats the lender's claim. B: the lender is not a purchaser for value. C: any deed is superior to a judgment lien. D: four days is not an unreasonable delay in recording a deed.

B is correct. The recording statute identified in the facts is a notice act. In this case, the lender's interest arises from a judgment lien against the owner rather than payment of valuable consideration. Therefore, the lender is not protected by the recording statute. Because the buyer is protected by the statute, the owner no longer holds an interest in Blackacre, and there is no property for the lien to attach to. A is incorrect. Under a notice statute, the lender will prevail over the buyer, assuming the lender is a BFP (a purchaser for value) because the buyer is a grantee who failed to record before the judgment lien was properly filed. C is incorrect. A deed is not always superior to a judgment lien. D is incorrect. This is a correct statement but is irrelevant to why the lender will be unsuccessful.

A plaintiff sued a defendant for illegal discrimination, claiming that the defendant fired him because of his race. At trial, the plaintiff called a witness, expecting him to testify that the defendant had admitted the racial motivation. Instead, the witness testified that the defendant said that he had fired the plaintiff because of his frequent absenteeism. While the witness is still on the stand, the plaintiff offers a properly authenticated secret tape recording he had made at a meeting with the witness in which the witness related the defendant's admissions of racial motivation. The tape recording is A: admissible as evidence of the defendant's racial motivation and to impeach the witness's testimony. B: admissible only to impeach the witness's testimony. C: inadmissible, because it is hearsay not within any exception. D: inadmissible, because a secret recording is an invasion of the witness's right of privacy under the U.S. Constitution.

B is correct. The tape recording is admissible to impeach the witness's testimony that the defendant fired the plaintiff for frequent absenteeism. However, if offered for the truth of the matter asserted - that the defendant fired the plaintiff because of his race - it would be inadmissible hearsay.

A man and his friend were charged with conspiracy to dispose of a stolen diamond necklace. The friend jumped bail and cannot be found. Proceeding to trial against the man alone, the prosecutor calls the friend's girlfriend as a witness to testify that the friend confided to her that "[the man] said I still owe him some of the money from selling that necklace." The witness's testimony is A: admissible, as evidence of a statement by party-opponent. B: admissible, as evidence of a statement against interest by the friend. C: inadmissible, because the friend's statement was not in furtherance of the conspiracy. D: inadmissible, because the friend is not shown to have firsthand knowledge that the necklace was stolen.

B is correct. The testimony contains two separate hearsay statements, both of which must satisfy an exemption or exception for the testimony to be admissible. The statement by the man to the friend is an admission by a partyopponent, and is therefore exempt as non-hearsay. The statement by the friend to the girlfriend constitutes an exception as a statement against interest. Therefore, the testimony is admissible.

A man obtained a bank loan secured by a mortgage on an office building that he owned. After several years, the man conveyed the office building to a woman, who took title subject to the mortgage. The deed to the woman was not recorded. The woman took immediate possession of the building and made the mortgage payments for several years. Subsequently, the woman stopped making payments on the mortgage loan, and the bank eventually commenced foreclosure proceedings in which the man and the woman were both named parties. At the foreclosure sale, a third party purchased the building for less than the outstanding balance on the mortgage loan. The bank then sought to collect the deficiency from the woman. Is the bank entitled to collect the deficiency from the woman? A: No, because the woman did not record the deed from the man. B: No, because the woman is not personally liable on the loan. C: Yes, because the woman took immediate possession of the building when she bought it from the man. D: Yes, because the woman was a party to the foreclosure proceeding.

B is correct. The woman took title to the office building subject to the mortgage but did not assume the mortgage debt. The debt is to be satisfied out of the building. The building is the principal, and the man, as transferor, is the only party liable for any deficiency. This situation can be contrasted with one in which a buyer expressly assumes the mortgage debt. In that case, the buyer would be primarily liable for any deficiency and the seller, absent a release by the mortgagee, would be secondarily liable. A is incorrect. The woman took title to the office building subject to the mortgage debt, which means that the debt was to be satisfied out of the building. The building is the principal, and the man, as transferor, is the only party liable for any deficiency. This situation can be contrasted with one in which a buyer expressly assumes the mortgage debt. In that case, the buyer would be primarily liable for any deficiency and the seller, absent a release by the mortgagee, would be secondarily liable. Recording the deed would give the bank constructive notice of the transfer but would have no effect on the collection of the deficiency. C is incorrect. The woman took title to the building subject to the mortgage. Her title to the building allowed her to take possession of the building, but her possession has no effect on the payment of any deficiency judgment. Taking title to the building subject to the mortgage means that the debt is to be satisfied out of the building. The building is the principal, and the man, as transferor, is the only party liable for any deficiency. This situation can be contrasted with one in which a buyer expressly assumes the mortgage debt. In that case, the buyer would be primarily liable for any deficiency and the seller, absent a release by the mortgagee, would be secondarily liable. D is incorrect. Because the woman took title to the building subject to the mortgage debt, she was a necessary party to the foreclosure proceeding. However, the fact that she took title to the building subject to the mortgage means that the debt is to be satisfied out of the building. The building is the principal, and the man, as transferor, is the only party liable for any deficiency. This situation can be contrasted with one in which a buyer expressly assumes the mortgage debt. In that case, the buyer would be primarily liable for any deficiency and the seller, absent a release by the mortgagee, would be secondarily liable.

A city has had a severe traffic problem on its streets. As a result, it enacted an ordinance prohibiting all sales to the public of food or other items by persons selling directly from trucks, cars, or other vehicles located on city streets. The ordinance included an inseverable grandfather provision exempting from its prohibition vendors who, for 20 years or more, have continuously sold food or other items from such vehicles located on the streets of the city. A retail vendor of ice cream products qualifies for this exemption and is the only food vendor that does. A yogurt retailer has a business similar to the ice cream vendor, but the yogurt vendor has been selling to the public directly from trucks located on the streets of the city only for the past ten years. The yogurt vendor filed suit in an appropriate federal district court to enjoin the enforcement of this ordinance on the ground that it denies the yogurt vendor the equal protection of the laws. In this case, the court will probably rule that the ordinance is A: constitutional, because it is narrowly tailored to implement the city's compelling interest in reducing traffic congestion and, therefore, satisfies the strict scrutiny test applicable to such cases. B: constitutional, because its validity is governed by the rational basis test, and the courts consistently defer to economic choices embodied in such legislation if they are even plausibly justifiable. C: unconstitutional, because the nexus between the legitimate purpose of the ordinance and the conduct it prohibits is so tenuous and its provisions are so underinclusive that the ordinance fails to satisfy the substantial relationship test applicable to such cases. D: unconstitutional, because economic benefits or burdens imposed by legislatures on the basis of grandfather provisions have consistently been declared invalid by courts as per se violations of the Equal Protection Clause of the Fourteenth Amendment.

B is correct. This ordinance regulates who may sell food from vehicles on the streets, which implicates neither a fundamental right nor a suspect class. Therefore, the ordinance would be subject to rational basis review, and the court will defer to any economic choice made by the city as long as it is plausibly justifiable.

An already married man went through a marriage ceremony with a woman and thereby committed bigamy. The man's friend, who did not know of the man's previous marriage, had encouraged the man to marry the woman and was best man at the ceremony. If the friend is charged with being an accessory to bigamy, he should be found A: not guilty, because his encouragement and assistance was not the legal cause of the crime. B: not guilty, because he did not have the mental state required for aiding and abetting. C: guilty, because he encouraged the man, and his mistake as to the existence of a prior marriage is not a defense to a charge of bigamy. D: guilty, because he was present when the crime occurred and is thus a principal in the second degree.

B is correct. To be an accessory to a criminal charge, the defendant must have the intent that the crime be committed and aid and abet in the commission of the crime. The friend, not knowing that the man was committing bigamy, had no intent to aid or assist the man in the commission of bigamy and should be found not guilty.

A woman brought a wrongful death action against a car driver who hit and killed her husband while her husband was crossing a street. In settlement negotiations, the driver told the woman that he was sorry for what had happened and that he had been too drunk to drive when he ran into her husband with his car. The woman and the driver settled the wrongful death action for $1 million. Subsequently, the driver was charged with reckless homicide of the husband. At the driver's criminal trial, the prosecutor offered, as evidence of the driver's guilt, the driver's statement to the woman during settlement negotiations. Is the driver's statement to the woman admissible in the subsequent criminal prosecution? A: No, because it is hearsay not within any exception. B: No, because a statement made in civil settlement negotiations between private parties is not admissible to prove the validity of a claim in a subsequent criminal prosecution. C: Yes, because a statement made in civil settlement negotiations is admissible in a subsequent criminal prosecution. D: Yes, because the statement's probative value outweighs the risk of prejudice and jury confusion.

B is correct. Under Federal Rule of Evidence (FRE) 408(a)(2), statements made during settlement negotiations in a private civil action are inadmissible in a subsequent criminal case that arises out of the same incident. Here, the subsequent criminal case arises out of the same incident that was the subject of the civil case, the car accident that killed the woman's husband. Therefore, the driver's statements made during settlement negotiations for the civil action are inadmissible to prove the driver's guilt in the criminal case.

A generally applicable state statute requires an autopsy by the county coroner in all cases of death that are not obviously of natural causes. The purpose of this law is to ensure the discovery and prosecution of all illegal activity resulting in death. In the 50 years since its enactment, the statute has been consistently enforced. A husband and wife are sincere practicing members of a religion that maintains it is essential for a deceased person's body to be buried promptly and without any invasive procedures, including an autopsy. When the couple's son died of mysterious causes and an autopsy was scheduled, the couple filed an action in state court challenging the constitutionality of the state statute, and seeking an injunction prohibiting the county coroner from performing an autopsy on their son's body. In this action, the couple claimed only that the application of this statute in the circumstances of their son's death would violate their right to the free exercise of religion as guaranteed by the First and Fourteenth Amendments. Assume that no federal statutes are applicable. As applied to the couple's case, the court should rule that the state's autopsy statute is A: constitutional, because a dead individual is not a person protected by the Due Process Clause of the Fourteenth Amendment. B: constitutional, because it is a generally applicable statute and is rationally related to a legitimate state purpose. C: unconstitutional, because it is not necessary to vindicate a compelling state interest. D: unconstitutional, because it is not substantially related to an important state interest.

B is correct. When a neutral law of general applicability impacts a religious practice, the law is subject to rational basis review. There is no inquiry into the extent of the impact or the sincerity of the religious beliefs. The law must merely be rationally related to a conceivable legitimate state interest.

The United States government demonstrated that terrorist attacks involving commercial airliners were perpetrated exclusively by individuals of one particular race. In response, Congress enacted a statute imposing stringent new airport and airline security measures only on individuals of that race seeking to board airplanes in the United States. Which of the following provides the best ground for challenging the constitutionality of this statute? A: The commerce clause of Article I, Section 8. B: The due process clause of the Fifth Amendment. C: The privileges and immunities clause of Article IV. D: The privileges or immunities clause of the Fourteenth Amendment.

B is correct. When the federal government takes action that would violate the Equal Protection Clause of the Fourteenth Amendment had it been conducted by a state government, it amounts to a violation of the Due Process Clause of the Fifth Amendment. The security measure is a presumptive violation of equal protection because it imposes a discriminatory racial classification on one minority group, which triggers strict scrutiny. It would be virtually impossible for the government to prove that the classification is necessary to serve a compelling interest.

The owner of Greenacre owned the land in fee simple. By warranty deed he conveyed Greenacre to a friend for life, "and from and after the death of my friend to my uncle, his heirs and assigns." Subsequently, the uncle died, devising all of his estate to a hospital. The uncle was survived by his daughter, his sole heir-at-law. Shortly thereafter the friend died, survived by the owner, the hospital, and the daughter. Title to Greenacre now is in A: the owner, because the contingent remainder never vested, and the owner's reversion was entitled to possession immediately upon the friend's death. B: the hospital, because the vested remainder in the uncle was transmitted by his will. C: The daughter, because she is the uncle's heir. D: either the owner or the daughter, depending upon whether the destructibility of contingent remainders is recognized in the applicable jurisdiction.

B is correct. When the uncle devised his entire estate to the hospital, this also included Greenacre. There is no interest in Greenacre for his heirs to inherit. A is incorrect. The uncle's heirs had a vested remainder subject to open, which the uncle then transmitted to the hospital. The owner gets nothing because he conveyed away his entire interest to the uncle. C is incorrect. The uncle devised his interest in Greenacre to the hospital, so there is no interest left for his daughter to inherit. D is incorrect. No one held a contingent remainder

An appropriations act passed by Congress over the President's veto directs that one billion dollars "shall be spent" by the federal government for the development of a new military weapons system, which is available only from the Arms Corporation. On the order of the President, the Secretary of Defense refuses to authorize a contract for purchase of the weapons system. The Arms Corporation sues the Secretary of Defense alleging an unlawful withholding of these federal funds. The strongest constitutional argument for the Arms Corporation is that A: passage of an appropriation over a veto makes the spending mandatory. B: Congress' power to appropriate funds includes the power to require that the funds will be spent as directed. C: the President's independent constitutional powers do not specifically refer to spending. D: the President's power to withhold such funds is limited to cases where foreign affairs are directly involved.

B is correct. While not an independent source of power, the Necessary and Proper Clause does give Congress the power to make all laws necessary and proper for carrying out any power granted to any branch of the federal government. Thus, the strongest argument that the Arms Corporation has is that Congress' power to appropriate funds to the executive branch also includes the power to direct how the funds are spent.

1. Perez sued Dawson for damages arising out of an automobile collision. At trial, Perez called Minter, an eyewitness to the collision. Perez expected Minter to testify that she had observed Dawson's automobile for five seconds prior to the collision and estimated Dawson's speed at the time of the collision to have been 50 miles per hour. Instead, Minter testified that she estimated Dawson's speed to have been 25 miles per hour. Without finally excusing Minter as a witness, Perez then called Wallingford, a police officer, to testify that Minter had told him during his investigation at the accident scene that Dawson "was doing at least 50." Wallingford's testimony is A) Admissible as a present sense impression B) Admissible to impeach Minter C) Inadmissible, because Perez may not impeach his own witness D) Inadmissible, because it is hearsay not within any exception

B) Admissible to impeach Minter

1. Defendant is on trial for robbing a bank in State A. She testified that she was in State B at the time of the robbery. Defendant calls her friend, Witness, to testify that two days before the robbery Defendant told him that she was going to spend the next three days in State B. Witness's testimony is A) Admissible, because the statement falls within the present sense impression exception to the hearsay rule. B) Admissible, because a statement of plans falls within the hearsay exception for then-existing state of mind. C) Inadmissible, because it is offered to establish an alibi by Defendant's own statement. D) Inadmissible, because it is hearsay not within any exception.

B) Admissible, because a statement of plans falls within the hearsay exception for then-existing state of mind.

1. Defendant was charged with attempted murder of Victor in a sniping incident in which Defendant allegedly shot at Victor from ambush as Victor drove his car along an expressway. The prosecutor offers evidence that seven years earlier Defendant had fired a shotgun into a woman's house and that Defendant had once pointed a handgun at another driver while driving on the street. This evidence should be A) excluded, because such evidence can be elicited only during cross-examination B) Excluded, because it is improper character evidence. C) Admitted as evidence of Defendant's propensity toward violence. D) Admitted as relevant evidence of Defendant's identity, plan, or motive.

B) Excluded, because it is improper character evidence.

1. The plaintiff is suing a legal journal for libel based on an article that implied he had cheated the IRS by not paying his taxes. At trial, the plaintiff denies any wrongdoing. An accountant is then called by the defense to testify that five years ago the plaintiff had improperly distorted the value of certain investments in an effort to lower his taxable income. Should the accountant's testimony be admitted? A) Yes, to challenge the plaintiff's credibility. [SB1] B) Yes, as substantive evidence of the plaintiff's character. C) No, because it is improper character evidence. D) No, because the testimony must be corroborated. [SB1]Credibility = impeachment

B) Yes, as substantive evidence of the plaintiff's character.

1. A defendant is on trial for possession with the intent to deliver narcotics. The prosecution calls a police officer who testifies that he witnesses several transactions between the defendant and buyers from an undisclosed location approximately 100 feet away from the corner where the defendant was standing. During cross-examination, the defendant's attorney attempts to admit into evidence a photograph that was properly authenticated by the police officer. The photograph is offered to show that the corner upon which the defendant was standing would have been blocked from the police officer's view. Upon objection by the prosecution, is the photograph admissible? A) Yes, because the court can take judicial notice of the layout of the street corner. B) Yes, to impeach the police officer. C) No, because extrinsic evidence cannot be used to impeach on a collateral matter. D) No, because the police officer cannot properly authenticate the photograph.

B) Yes, to impeach the police officer.

1- Modality City has had a severe traffic problem on its streets. As a result, it enacted an ordinance prohibiting all sales to the public of food or other items by persons selling directly from trucks, cars, or other vehicles located on city streets. The ordinance included an inseverable grandfather provision exempting from its prohibition vendors who, for 20 years or more, have continuously sold food or other items from such vehicles located on the streets of Modality City. Northwind Ice Cream, a retail vendor of ice cream products, qualifies for this exemption and is the only food vendor that does. Yuppee Yogurt is a business similar to Northwind, but Yuppee has been selling to the public directly from trucks located on the streets of Modality City only for the past ten years. Yuppee filed suit in an appropriate federal district court to enjoin enforcement of this ordinance on the ground that it denies Yuppee the equal protection of the laws. In this case, the court will probably rule that the ordinance is A- Constitutional, because it is narrowly tailored to implement the city's compelling interest in reducing traffic congestion and, therefore, satisfies the strict scrutiny test applicable to such cases. B- Constitutional, because its validity is governed by the rational basis test, and the courts consistently defer to economic choices embodied in such legislation if they are even plausibly justifiable. C- Unconstitutional, because the nexus between the legitimate purpose of the ordinance and the conduct it prohibits is so tenuous and its provisions are so underinclusive that the ordinance fails to satisfy the substantial relationship test applicable to such cases. D- Unconstitutional, because economic benefits or burdens imposed by legislatures on the basis of grandfather provisions have consistently been declared invalid by courts as per se violations of the equal protection clause of the Fourteenth Amendment.

B- Constitutional, because its validity is governed by the rational basis test, and the courts consistently defer to economic choices embodied in such legislation if they are even plausibly justifiable.

1- Three states, East Winnetka, Midland, and West Hampton, are located next to one another in that order. The states of East Winnetka and West Hampton permit the hunting and trapping of snipe, but the state of Midland strictly forbids it in order to protect snipe, a rare species of animal, from extinction. The state of Midland has a state statute that provides "Possession of snipe traps is prohibited. Any game warden finding a snipe trap within the state shall seize and destroy it." Snipe traps cost about $15 each. Prentis is a resident of West Hampton and an ardent snipe trapper. She drove her car to East Winnetka to purchase a new snipe trap in her car, Prentis stopped in a Midland state park to camp for a few nights. While she was in that park, a Midland game warden saw the trap, which was visible on the front seat of her car. The warden seized the trap and destroyed it in accordance with the Midland statute after Prentis admitted that the seized item was a prohibited snipe trap. No federal statutes or federal administrative regulations apply. For this question only, assume that Prentis demonstrates that common carriers are permitted to transport snipe traps as cargo across Midland for delivery to another state and that in practice the Midland statute is enforced only against private individuals transporting those traps in private vehicles. If Prentis challenges the application of the Midland statute to her on the basis only of a denial of equal protection, the application of the statute will probably be found A- constitutional, because the traps constitute contraband in which Prentis could have no protected property interest. B- Constitutional, because there is a rational basis for differentiating between the possession of snipe traps as interstate cargo by common carriers and the possession of snipe traps by private individuals. C- Unconstitutional, because the state cannot demonstrate a compelling public purpose for making this differentiation between common carriers and such private individuals. Unconstitutional, because interstate travel is a fundamental right that may not be burdened by state law

B- Constitutional, because there is a rational basis for differentiating between the possession of snipe traps as interstate cargo by common carriers and the possession of snipe traps by private individuals.

1- A federal statute prohibits the construction of nuclear energy plants in this country without a license from the Federal Nuclear Plant Siting Commission. The statute provides that the Commission may issue a license authorizing the construction of a proposed nuclear energy plant 30 days after the Commission makes a finding that the plant will comply with specified standards of safety, technological and commercial feasibility, and public convenience. In a severable provision, the Commission's enabling statute also provides that the Congress, by simple majorities in each house, may veto the issuance of a particular license by the Commission if such a veto occurs within 30 days following the required Commission finding. Early last year, the Commission found that Safenuk, Inc., met all statutory requirements and, therefore, voted to issue Safenuke, Inc., a license authorizing it to construct a nuclear energy plant. Because they believed that the issuance of a license to Safenuke, Inc., was not in accord with the applicable statutory criteria, a majority of each of the two houses of Congress voted, within the specified 30-day period, to veto the license. On the basis of that veto, the Commission refused to issue the license. Subsequently, Safenuke, Inc., sued the Commission in an appropriate federal district court, challenging the constitutionality of the Commission's refusal to issue the license. In this suit, the court should hold the congressional veto of the license of Safenuke, Inc., to be A- Invalid, because any determination by Congress that particular agency action does not satisfy statutory criteria violates Article III, Section 1 of the Constitution because it constitutes the performance of a judicial function by the legislative branch. B- Invalid, because Article I, Section 7 of the Constitution has been interpreted to mean that any action of Congress purporting to alter the legal rights of persons outside of the legislative branch must be presented to the President for his signature or veto. C- Valid, because Congress has authority under the commerce clause to regulate the construction of nuclear energy plants. D- Valid, because there is a compelling national interest in the close congressional supervision of nuclear plant siting in light of the grave dangers to the public health and safety that are associated with the operation of such plants.

B- Invalid, because Article I, Section 7 of the Constitution has been interpreted to mean that any action of Congress purporting to alter the legal rights of persons outside of the legislative branch must be presented to the President for his signature or veto.

1. Skakel, a freshman at the University of Michigan in Ann Arbor, was pledging Zeta Eta Mu (ZEM) fraternity. During the "hell week" initiation ritual, each pledge was required to perform an act to demonstrate his courage and manhood. Accompanied by a group of ZEM members, Skakel went to the State Street overpass that extended across Interstate Highway 90. The overpass was elevated approximately 50 feet above the highway. Skakel then pulled out a five-pound bowling ball that he had been carrying in a bag. He lobbed the ball from the overpass onto the highway below. Skakel had hoped that the ball would fall harmlessly to the pavement. However, the ball crashed through the front windshield of a car driven by Mahorn. The Bowling ball struck Mahorn, causing him to lose control of his vehicle. The car swerved across the highway and crashed into a telephone pole. Mahorn was killed in the wreckage. In this jurisdiction, first degree murder is defined as any willful killing of another human being committed with premeditation or deliberation. Second degree murder covers all other unlawful killings with express or implied malice aforethought. Manslaughter is defined as any homicide committed as a result of adequate provocation or caused by defendant's criminal negligent conduct. If Skakel is charged with criminal homicide , he should be found guilty of A- Murder in the first degree B- Murder in the second degree C- Manslaughter D- None of the above.

B- Murder in the second degree

1- Doctor, a resident of the city of Greenville in the state of Green, is a physician licensed to practice in both Green and the neighboring state of Red. Doctor finds that the most convenient place to treat her patients who need hospital care is in the publicly owned and operated Redville Municipal Hospital of the city of Redville in the state of Red, which is located just across the state line from Greenville. For many years Doctor had successfully treated her patients in that hospital. Early this year she was notified that she could no longer treat patients in the Redville hospital because she was not a resident of Red, and a newly adopted rule of Redville Municipal Hospital, which was adopted in conformance with all required procedures, stated that every physician who practices in that hospital must be a resident of Red. Which of the following constitutional provisions would be most helpful to Doctor in an action to challenge her exclusion from the Redville hospital solely on the basis of this hospital rule? A- The bill of attainder clause B- The privileges and immunities clause of Article IV C- The due process clause of the Fourteenth Amendment D- The ex post facto clause

B- The privileges and immunities clause of Article IV

1- Congressional hearings determined that the use of mechanical power hammers is very dangerous to the persons using them and to persons in the vicinity of the persons using them. As a result, Congress enacted a stature prohibiting the use of mechanical power hammers on all construction projects in the United States. Subsequently, a study conducted by a private research firm concluded that nails driven by mechanical power hammers have longer-lasting joining power than hand-driven nails. After learning about this study, the city council of the city of Green enacted an amendment to its building safety code requiring the use of mechanical power hammers in the construction of all buildings intended for human habitation. This amendment to the city of Green's building safety code is A- unconstitutional, because it was enacted subsequent to the federal statute. B- Unconstitutional, because it conflicts with the provisions of the federal statute. C- Constitutional, because the federal statute does not expressly indicate that it supersedes inconsistent state or local laws. D- Constitutional, because the long-term safety of human habitations justifies some additional risk to the people engaged in their construction.

B- Unconstitutional, because it conflicts with the provisions of the federal statute.

1) At a country auction, a plaintiff acquired an antique cabinet that he recognized as an extremely rare and valuable collector's item. Unfortunately, the plaintiff's cabinet had several coats of varnish and paint over the original oil finish. Its potential value could only be realized if these layers could be removed without damaging the original finish. Much of the value of the cabinet depends on the condition of a unique oil finish, the secret of which died with the original inventor. A professional restorer of antique furniture recommended that the plaintiff use a specific paint stripper to remove the paint and varnish from the cabinet. The plaintiff obtained and read a sales brochure published by the company who manufactures the paint stripper, which contained the following statement: "This product will renew all antique furniture. Will not damage original oil finishes." The plaintiff purchased the paint stripper and used it on his cabinet, being very careful to follow the accompanying instructions exactly. Despite the plaintiff's care, the original finish of the cabinet was irreparably damaged. When finally refinished, the cabinet was worth less than 20% of what it would have been worth if the original finish had been preserved. No other removal technique could have preserved the original finish. If the plaintiff sues the manufacturer to recover the loss he has suffered as a result of the destruction of the cabinet's original finish, will the plaintiff prevail? A) Yes, because no other known removal technique would have preserved the original finish. B) Yes, because the loss would not have occurred had the statement in the brochure been true. C) No, because the product was not defective when sold by the manufacturer. D) No, because the product was not dangerous to persons.

B. Yes, because the loss would not have occurred had the statement in the brochure been true. Warranty answer

1. The police had, over time, accumulated reliable information that Jason operated a large cocaine-distribution network, that he and his accomplices often resorted to violence, and that they kept a small arsenal of weapons in his home. One day, the police received reliable information that a large brown suitcase with leather straps containing a supply of cocaine had been delivered to Jason's home and that it would be moved to distribution point the next morning. The police obtained a valid search warrant to search for and seize the brown suitcase and the cocaine and went to Jason's house. The police knocked on Jason's door and called out, "Police. Open up. We have a search warrant." After a few seconds with no response, the police forced the door open and entered. Nearing noises in the basement, the police ran down there and found Jason with a large brown suitcase and put handcuffs on Jason. A search of his person revealed a switchblade knife and a .45-caliber pistol. Jason cursed the police and said, "You never would have caught me with the stuff if it hadn't been for that lousy snitch Harvey!" The police then fanned out through the house, looking in every room and closet. They found no one else, but one officer found an Uzi automatic weapon in a box on a closet shelf in Jason's bedroom. In addition to charges relating to the cocaine in the suitcase, Jason is charged with unlawful possession of weapons. Jason moves pretrial to suppress the use as evidence of the weapons seized by the police and of the statement he made. As to the switchblade knife and the .45-caliber pistol, Jason's motion to suppress should be A- Granted, because the search and seizure were the result of illegal police conduct in executing the search warrant. B- Granted, because the police did not inform Jason that he was under arrest and did not read him his Miranda rights. C- Denied, because the search and seizure were incident to a lawful arrest. D- Denied, because the police had reasonable grounds to believe that there were weapons in the house.

C - Denied, because the search and seizure were incident to a lawful arrest.

1- The state of Orrington wanted to prevent its only major league baseball team, the privately owned and operated Orrington Opossums, from moving to the rival state of Atrium. After a heated political debate in the legislature, Orrington enacted legislation providing for a one-time grant of $10 million in state funds to the Opossums to cover part of the projected income losses the team would suffer during the next five years if it remained in that state. The legislation required that the team remain in the state for at least ten years if it accepted the grant. After accepting the grant, the owners of the Opossums decided to build a new $150 million stadium in Orrington. As plans for the construction of the new stadium proceeded, it became evident that all of the contractors and subcontractors would be white males, and that they had been chosen by the owners of the Opossums without any public bods because these contractors and subcontractors had successfully built the only other new baseball stadium in the region. Several contractors who were females or members of minority racial groups filed suit against the owners of the Opossums in federal district court to compel public solicitation of bids for the construction of its new stadium on an equal opportunity basis, and to enjoin construction of the stadium until compliance was ensured. Their only claim was that the contracting practices of the owners of the Opossums denied them the equal protection of the laws in violation of the Fourteenth Amendment. In this suit, the court will probably rule that A- The nexus between the actions of the owners of the Opossums and the one-time grant of monies to them by the state is sufficiently substantial to subject their actions to the limitations of the Fourteenth Amendment. B- The intense public preoccupation with the activities of major league baseball teams coupled with the fact that baseball is considered to be our national pastime is sufficient to justify application of the Fourteenth Amendment to the activities of major league teams. C- In the absence of additional evidence of state involvement in the operations or decisions of the owners of the Opossums, a onetime grant of state monies to them is insufficient to warrant treating their actions as subject to the limitations of the Fourteenth Amendment. D- The issues presented by this case are nonjusticiable political questions because there is a lack of judicially manageable standards to resolve them and they are likely to be deeply involved in partisan politics.

C - In the absence of additional evidence of state involvement in the operations or decisions of the owners of the Opossums, a onetime grant of state monies to them is insufficient to warrant treating their actions as subject to the limitations of the Fourteenth Amendment.

A man approached an undercover officer and inquired about hiring someone to kill his girlfriend's parents. The man did not realize he was speaking to an undercover officer. The undercover officer pretended to agree to handle the job and secretly taped subsequent conversations with the man concerning plans and payment. A few days before the payment was due, the man changed his mind and called the plan off. Nevertheless, the man was charged with solicitation to commit murder. The man should be A: acquitted, because he withdrew before payment and commission of the act. B: acquitted, because no substantial acts were performed. C: convicted, because the offense was completed before his attempt to withdraw. D: convicted, because the undercover officer agreed to commit the offense.

C is correct. A charge of solicitation is completed once the person invites, requests, commands, hires, or encourages another to commit a particular offense with the intent that the offense be committed. In this case, the man is guilty of solicitation when he requested that the undercover officer kill his girlfriend's parents.

A driver was driving his car near a homeowner's house when the homeowner's child darted into the street in front of the driver's car. As the driver swerved and braked his car to avoid hitting the child, the car skidded up into the homeowner's driveway and stopped just short of the homeowner, who was standing in the driveway and had witnessed the entire incident. The homeowner suffered from serious emotional distress from witnessing the danger to his child and to himself. Neither the homeowner nor his property was physically harmed. If the homeowner asserts a claim for damages against the driver but is unable to establish that the driver was negligent, will the homeowner still be able to prevail? A: Yes, because the driver's entry onto the homeowner's land was unauthorized. B: Yes, because the homeowner suffered serious emotional distress by witnessing the danger to his child and to himself. C: No, because the homeowner failed to show that the driver was negligent. D: No, because the homeowner's child was not exercising reasonable care.

C is correct. A claim for damages based solely on emotional distress requires at least negligence by the defendant in order to prevail. Here, absent any evidence that the driver acted negligently, the subsequent events causing the homeowner's emotional suffering will not give rise to a successful claim.

A student contracted for an expensive cable television service for a period of six months solely to view the televised trial of a defendant, who was on trial for murder in a court of a particular state. In the midst of the trial, the judge prohibited any further televising of the defendant's trial because he concluded that the presence of television cameras was disruptive. The student brought an action in federal district court against the judge in the defendant's case asking only for an injunction that would require the judge to resume the televising of the defendant's trial. The student alleged that the judge's order to stop the televising of the defendant's trial deprived him of property--his investment in cable television service--without due process of law. Before the student's case came to trial, the defendant's criminal trial concluded in a conviction and sentencing. There do not appear to be any obvious errors in the proceeding that led to the result in the defendant's case. After the defendant's conviction and sentencing, the opposing party in the student's case moved to dismiss the suit. The most proper disposition of this motion by the federal court would be to A: defer action on the motion until after any appellate proceedings in the defendant's case have concluded, because the defendant might appeal, his conviction might be set aside, he might be tried again, and television cameras might be barred from the new trial. B: defer action on the motion until after the state Supreme Court expresses a view on its proper disposition, because the state law of mootness governs suits in federal court when the federal case is inexorably intertwined with a state proceeding. C: grant the motion, because the subject matter of the controversy between the student and the defendant has ceased to exist and there is no strong likelihood that it will be revived. D: deny the motion, because the student has raised an important constitutional question: whether his investment in cable service, solely to view the defendant's trial, is property protected by the Due Process Clause of the Fourteenth Amendment.

C is correct. Article III limits the exercise of federal court jurisdiction to cases and controversies, and once the injury that gave rise to the case is incapable of remedy, the case becomes moot and no longer justiciable. Here, once the trial was over, the relief the student sought, (televising the trial), was no longer possible, and his case became moot. Moreover, it is unlikely that the controversy will be revived because the case is not on appeal now, and there were no obvious errors in the proceedings. Therefore, the federal court should grant the motion to dismiss.

While waiting in line to open an account with a bank, a customer read a poster on the bank's wall that said, "New Customers! $25 FOR 5 MINUTES. If you stand in line for more than five minutes, we will pay you $25! We like happy customers!" The customer started timing his wait and just as five minutes was about to pass, the bank manager tore the poster down and announced, "The $25 stand-in-line promotion is over." The customer waited in line for 10 more minutes before being served. Does the customer have a claim against the bank for $25? A: No, because the bank withdrew its offer before the customer completed the requested performance. B: No, because the bank's statement was a nonbinding gift promise. C: Yes, because the bank could not revoke its offer once the customer had commenced performance. D: Yes, because the customer's presence in line served as notice to the bank that he had accepted.

C is correct. By beginning performance on the unilateral contract established by the poster, the bank's offer became an option contract and thus irrevocable. A is incorrect. Completion of performance is what constitutes acceptance, but it is not necessary to render the offer irrevocable. As such, the manager's removal of the poster did not revoke, and the customer's completion of performance makes the bank liable. B is incorrect. This was not a gift promise, but a unilateral contract supported by valid consideration. The bank offered $25 in exchange for the customer standing in line, which was a bargained-for exchange. D is incorrect. The customer's mere presence in line constituted the beginning of performance and created an option. The reason the customer has a claim is that the offer became irrevocable at that point, so tearing down the sign did not absolve the bank of liability for the $25.

Congress provides by statute that any state that fails to prohibit automobile speeds of over 55 miles per hour on highways within the state shall be denied all federal highway construction funding. One of the richest and most highway-oriented states in the country refuses to enact such a statute. The strongest argument that can be made in support of the constitutionality of this federal statute is that A: the states ceded their authority over highways to the national government when the states accepted federal grants to help finance their highways. B: the federal government can regulate the use of the state highways without limitation because the federal government paid for some of their construction costs. C: Congress could reasonably believe that the 55 mile-an-hour speed limit will assure that the federal money spent on highways results in greater benefit than harm to the public. D: a recent public opinion survey demonstrated that 90 percent of the people in this country support a 55 mile-an-hour speed limit.

C is correct. Congress's reasonable belief that the speed limit conditioned on receipt of federal construction funding will result in greater benefit than harm to the public supports the conclusion that this a proper condition on federal spending. It is an incentive (rather than a compulsion) for lowering the speed limit, it is clearly stated, and there is a nexus between safety, the speed limit, and the use of federal funds on the highways.

A newly enacted federal statute appropriates $100 million in federal funds to support basic research by universities located in the United States. The statute provides that "the ten best universities in the United States" will each receive $10 million. It also provides that "the ten best universities" shall be "determined by a poll of the presidents of all the universities in the nation, to be conducted by the United States Department of Education." In responding to that poll, each university president is required to apply the well-recognized and generally accepted standards of academic quality that are specified in the statute. The provisions of the statute are inseverable. Which of the following statements about this statute is correct? A: The statute is unconstitutional, because the reliance by Congress on a poll of individuals who are not federal officials to determine the recipients of its appropriated funds is an unconstitutional delegation of legislative power. B: The statute is unconstitutional, because the limitation on recipients to the ten best universities is arbitrary and capricious and denies other high quality universities the equal protection of the laws. C: The statute is constitutional, because Congress has plenary authority to determine the objects of its spending and the methods used to achieve them, so long as they may reasonably be deemed to serve the general welfare and do not violate any prohibitory language in the Constitution. D: The validity of the statute is nonjusticiable, because the use by Congress of its spending power necessarily involves political considerations that must be resolved finally by those branches of the government that are closest to the political process.

C is correct. Congress's spending power is plenary, and Congress can choose how it spends its money so long as that choice can reasonably be said to serve the general welfare and not violate any prohibition in the Constitution.

A truck driver from State A filed a proper diversity suit against a manufacturer from State B in federal court in State A. Before answering the truck driver's complaint, the manufacturer timely filed a motion to dismiss for lack of personal jurisdiction. The truck driver then filed a response to the manufacturer's pre-answer motion, detailing the manufacturer's minimum contacts in State A due to the manufacturer's selling of goods in State A. The manufacturer then made a second pre-answer motion claiming improper venue. Is the court likely to consider the manufacturer's claim for improper venue? A: Yes, because a party may raise the defense of improper venue at any time. B: Yes, because a party does not need to raise the defense of improper venue in their initial pre-answer motion. C: No, because the manufacturer waived this defense by not raising it in its initial pre-answer motion. D: No, because the truck driver's answer establishes that the manufacturer has minimum contacts in State A, so the court may presume that venue is proper.

C is correct. Federal Rule of Civil Procedure (FRCP) 12(b) governs pre-answer motions. The defendant must raise certain defenses at the time he files a pre-answer motion or his answer, whichever is first, or the defense is waived. These defenses include lack of personal jurisdiction, improper venue, insufficient process, and insufficient service of process. Therefore, the defense of improper venue is waived if not raised in the first motion made to the court.

A defendant operates a bank courier service that uses armored trucks to transport money and securities. One of the defendant's armored trucks was parked illegally, too close to a street intersection. The plaintiff, driving his car at an excessive speed, skidded into the armored truck while trying to make a turn. The truck was not damaged, but the plaintiff was injured. The plaintiff has brought an action against the defendant to recover damages for his loss resulting from the accident. The jury determined that both parties were negligent, but that the defendant was less negligent than the plaintiff. The jurisdiction follows a pure comparative negligence rule. In this action, the plaintiff should recover A: nothing, because the defendant was not an active or efficient cause of the plaintiff's loss. B: nothing, because the defendant was less negligent. C: his entire loss, reduced by a percentage that reflects the negligence attributed to the plaintiff. D: his entire loss, because the defendant's truck suffered no damage.

C is correct. In a pure comparative rule jurisdiction, the plaintiff may recover his full amount of damages, less the portion attributed to his own negligence. The plaintiff is not barred from recovery by his own negligence, but he will have his award reduced, according to the court's determination of the plaintiff's percentage of responsibility for his own injuries due to his excessive speed in driving.

20 years ago, a testator who owned Blackacre, a one-acre tract of land, duly delivered a deed of Blackacre "to the school district so long as it is used for school purposes." The deed was promptly and properly recorded. Five years ago, the testator died leaving his son as his only heir but, by his duly probated will, he left "all my Estate to my friend." Last month, the school district closed its school on Blackacre and for valid consideration duly executed and delivered a quitclaim deed of Blackacre to a developer, who planned to use the land for commercial development. The developer has now brought an appropriate action to quiet title against the testator's son, the friend, and the school district. The only applicable statute is a provision in the jurisdiction's probate code which provides that any property interest which is descendible is devisable. In such action, the court should find that title is now in A: the developer. B: the son. C: the friend. D: the school district.

C is correct. In this case, the school district's interest was a fee simple determinable (meaning it could be terminated), and the testator's interest was a possibility of reverter. According to the facts, the only event that could terminate the school district's interest would be to cease using the land for "school purposes." When that event occurred, the school's interest terminated and the land reverted back to the testator. The testator, however, had died by the time his interest reverted, so that interest went to the friend in accordance with the dictates of the testator's duly probated will. A is incorrect. The school district's interest reverted back to the testator because the school district terminated their interest by selling the land to the developer. B is incorrect. Because the school district terminated their interest by selling Blackacre to the developer, the interest in Blackacre reverted back to the testator, whose heir was the friend. However, if the testator had not bequeathed his estate to the friend, then the interest would have reverted to the testator's son as the testator's only heir. D is incorrect. The school district terminated its own interest in Blackacre.

A breeder bought a two-month-old registered boar at auction from a seller for $800. No express warranty was made. 15 months later, tests by experts proved conclusively that the boar had been born incurably sterile. If this had been known at the time of the sale, the boar would have been worth no more than $100. In an action by the breeder against the seller to avoid the contract and recover the price paid, the parties stipulate that, as both were and had been aware, the minimum age at which the fertility of a boar can be determined is about 12 months. Which of the following will the court probably decide? A: The breeder wins, because the parties were mutually mistaken as to the boar's fertility when they made the agreement. B: The breeder wins, because the seller impliedly warranted that the boar was fit for breeding. C: The seller wins, because the breeder assumed the risk of the boar's sterility. D: The seller wins, because any mistake involved was unilateral, not mutual.

C is correct. Mutual mistake will only discharge duties under a contract when the party seeking to avoid it did not assume the risk of the mistake. The facts state that both parties knew the fertility was unknown due to the boar's young age, which means there was uncertainty, not mistake. The fact that the breeder knew the fertility could not yet be evaluated for 10 months, but contracted for the sale anyway, means that the court should find for the seller. A is incorrect. This was not a mistake, but an element of uncertainty that the breeder knew of and proceeded to enter into the contract anyway. B is incorrect. The facts do not suggest such an implied term since both parties were aware that such a determination was not possible at the time of the contract. D is incorrect. As explained above, this was not a mistake, unilateral or otherwise. It was a situation involving uncertainty by both parties regarding the outcome of the boar's fertility.

The plaintiffs, a retired couple, had lived in their home in a residential neighborhood for 20 years when the defendants, a family of six, moved into the house next door and built a swimming pool in the back yard. The family's four young children frequently played in the pool after school. They often were joined by other neighborhood children. The plaintiffs were in the habit of reading and listening to classical music in the afternoons. Sometimes they took naps. The boisterous sounds of the children playing in the pool disturbed the plaintiffs' customary enjoyment of quiet afternoons. In the plaintiffs' nuisance action for damages against the defendants, the plaintiffs should A: prevail, because the children's noise constituted a substantial interference with the plaintiffs' use and enjoyment of their home. B: prevail, because the the plaintiffs' interest in the quiet enjoyment of their home takes precedence in time over the defendants' interests. C: not prevail, because the noise did not constitute a substantial and unreasonable disturbance to persons of normal sensibilities. D: not prevail, because the children's interest in healthy play has priority over the plaintiffs' interest in peace and quiet

C is correct. Private nuisance is defined as a substantial, unreasonable interference with another private individual's use or enjoyment of their own property. Substantial interference is something that would be offensive, inconvenient or annoying to an average person of the same community. Unreasonable interference is determined by balancing the severity of the injury against the utility of the defendant's conduct.

Plagued by neighborhood youths who had been stealing lawn furniture from his backyard, a homeowner remained awake each night watching for them. One evening the homeowner heard noises in his backyard. He yelled out, warning intruders to leave. Receiving no answer, he fired a shotgun filled with nonlethal buckshot into bushes along his back fence where he believed the intruders might be hiding. A six-year-old child was hiding in the bushes and was struck in the eye by some of the pellets, causing loss of sight. If the homeowner is charged with second-degree assault, which is defined in the jurisdiction as "maliciously causing serious physical injury to another," he is A: not guilty, because the child was trespassing and he was using what he believed was nondeadly force. B: not guilty, because he did not intend to kill or to cause serious physical injury. C: guilty, because he recklessly caused serious physical injury. D: guilty, because there is no privilege to use force against a person who is too young to be criminally responsible

C is correct. Proof of malice does not need to be proof of an actual specific intent to kill or harm another; it can be implied from a defendant's gross recklessness with regard to human life shown. In this case, the homeowner fired a shotgun into bushes where he believed people were hiding. This action would support a finding of malice. The shooting caused the child to lose his sight, which is a serious injury. Therefore, the homeowner is guilty of second degree assault.

A state has a statute providing that an unsuccessful candidate in a primary election for a party's nomination for elected public office may not become a candidate for the same office at the following general election by nominating petition or by write-in votes. A woman sought her party's nomination for governor in the May primary election. After losing in the primary, the woman filed nominating petitions containing the requisite number of signatures to become a candidate for the office of governor in the following general election. The chief elections officer of the state refused to certify the woman's petitions solely because of the above statute. The woman then filed suit in federal district court challenging the constitutionality of this state statute. As a matter of constitutional law, which of the following is the proper burden of persuasion in this suit? A: The woman must demonstrate that the statute is not necessary to achieve a compelling state interest. B: The woman must demonstrate that the statute is not rationally related to a legitimate state interest. C: The state must demonstrate that the statute is the least restrictive means of achieving a compelling state interest. D: The state must demonstrate that the statute is rationally related to a legitimate state interest.

C is correct. Restrictions on the ability of individuals to be candidates must be examined to determine whether they violate the First Amendment freedoms regarding association and expression or the Fourteenth Amendment Equal Protection Clause. Here, First Amendment rights are involved in the action because the statute infringes on the right of political association. As a result, strict scrutiny is the appropriate level of review for the statute.

Insurance is provided in a particular state only by private companies. Although the state insurance commissioner inspects insurance companies for solvency, the state does not regulate their rates or policies. An insurance company charges higher rates for burglary insurance to residents of one part of a county in the state than to residents of another section of the same county because of the different crime rates in those areas. The plaintiff is a resident of the county who was charged the higher rate by the insurance company because of the location of her residence. The plaintiff sues the insurance company, alleging that the differential in insurance rates unconstitutionally denies her the equal protection of the law. Will the plaintiff's suit succeed? A: Yes, because the higher crime rate in the plaintiff's neighborhood demonstrates that the county police are not giving persons who reside there the equal protection of the laws. B: Yes, because the insurance rate differential is inherently discriminatory. C: No, because the constitutional guarantee of equal protection of the law is not applicable to the actions of these insurance companies. D: No, because there is a rational basis for the differential in insurance rates.

C is correct. The Constitution provides for equal protection of the law, which means that it protects individuals from actions by the state (the Fourteenth Amendment) or the federal government (the Fifth Amendment). Equal protection only restricts private action in extremely specific circumstances, none of which are present in this fact pattern.

In a federal investigation of a defendant for tax fraud, the grand jury seeks to obtain a letter written January 15 by the defendant to her attorney in which she stated: "Please prepare a deed giving my ranch to the university but, in order to get around the tax law, I want it back-dated to December 15." The attorney refuses to produce the letter on the ground of privilege. Production of the letter should be A: prohibited, because the statement is protected by the attorney-client privilege. B: prohibited, because the statement is protected by the client's privilege against self-incrimination. C: required, because the statement was in furtherance of crime or fraud. D: required, because the attorney-client privilege belongs to the client and can be claimed only by her.

C is correct. The attorney-client privilege will cover confidential communications made during a legal consultation between an attorney and client. However, if the services of the lawyer were sought or obtained to enable or aid anyone to commit or plan to commit a crime or a fraud, the privilege does not apply. The defendant's letter explicitly states that the deed she is seeking should be back-dated to avoid the tax law. Because the services sought by the defendant from her attorney were for the commission of tax fraud, the communication will not be privileged and the production of the letter should be required.

A gentleman owned Greenacre in fee simple of record on January 10. On that day, a bank loaned the gentleman $50,000 and the gentleman mortgaged Greenacre to the bank as security for the loan. The mortgage was recorded on January 18. The gentleman conveyed Greenacre to a buyer for a valuable consideration on January 11. The bank did not know of this, nor did the buyer know of the mortgage to the bank, until both discovered the facts on January 23, the day on which the buyer recorded the gentleman's deed. The recording act of the jurisdiction provides: "No unrecorded conveyance or mortgage of real property shall be good against subsequent purchasers for value without notice, who shall first record." There is no provision for a grace period, and there is no other relevant statutory provision. The bank sued the buyer to establish that its mortgage was good against Greenacre. The court should decide for A: the buyer, because he paid valuable consideration without notice before the bank recorded its mortgage. B: the buyer, because the bank's delay in recording means that it is estopped from asserting its priority in time. C: the bank, because the buyer did not record his deed before the bank's mortgage was recorded. D: the bank, because after the mortgage to the bank, the gentleman's deed to the buyer was necessarily subject to its mortgage.

C is correct. The bank was the first party to record the deed and also acquired the land without notice of prior conveyances. A is incorrect. The buyer recorded after the bank and will not prevail. B is incorrect. The bank still recorded before the buyer, and thus will prevail under a race-notice act. D is incorrect. The bank will prevail under the recording act even if the buyer's deed is not subject to the bank's mortgage, because it recorded first.

A man was a suspect in a homicide committed during a robbery of a liquor store. A barber was a friend of the suspect. The police telephoned the barber and asked if he would help locate the suspect. The barber agreed and met the police officers at headquarters later that night. After a discussion during which police asked questions about the suspect and the homicide, the barber said he wanted to get something "off his chest" and advised the officers that he was in on the robbery but that the suspect had shot the owner of the store without his permission or prior knowledge. The officers then for the first time gave the barber his Miranda warnings. The barber was indicted for felony murder. He moved to prevent the introduction of his statement into evidence. His motion should be A: granted, because the barber was effectively in custody and entitled to receive Miranda warnings at the beginning of the discussion. B: granted, because the barber's rights to counsel and to due process were violated by the interrogation at police headquarters. C: denied, because his statement was freely and voluntarily given, and he was not entitled to Miranda warnings. D: denied, because by visiting headquarters voluntarily, the barber waived his right to have Miranda warnings at the beginning of the discussion.

C is correct. The barber's statements were made voluntarily and without police coercion. The officers were questioning the barber in an attempt to locate information and the whereabouts of the suspect, not in an attempt to obtain a confession from the barber. The barber was not the suspect, never placed under arrest, and not subject to custodial interrogation, so the police officers were not constitutionally required to give him his Miranda warnings. The barber's motion to suppress his statement should be denied.

A plaintiff, who was 20 years old, purchased a new, high-powered sports car that was marketed with an intended and recognized appeal to youthful drivers. The car was designed with the capability to attain speeds in excess of 100 miles per hour. It was equipped with tires designed and tested only for a maximum safe speed of 85 miles per hour. The owner's manual that came with the car stated that "continuous driving over 90 miles per hour requires highspeed-capability tires," but the manual did not describe the speed capability of the tires sold with the car. The plaintiff took her new car out for a spin on a straight, smooth country road where the posted speed limit was 55 miles per hour. Intending to test the car's power, she drove for a considerable distance at over 100 miles per hour. While she was doing so, the tread separated from the left rear tire, causing the car to leave the road and hit a tree. The plaintiff sustained severe injuries. The plaintiff has brought a strict product liability action in tort against the manufacturer of the car. You should assume that pure comparative fault principles apply to this case. Will the plaintiff prevail? A: No, because the plaintiff's driving at an excessive speed constituted a misuse of the car. B: No, because the car was not defective. C: Yes, because the statement in the manual concerning the tires did not adequately warn of the danger of high-speed driving on the tires mounted on the car. D: No, because the plaintiff's driving at a speed in excess of the posted speed limit was negligence per se that was not excusable.

C is correct. The car manufacturer created a high-powered sports car and then failed to equip it with high-speed capability tires. This would make the car unreasonably dangerous and strict liability may be applied if, as in this case, the manufacturer failed to give a proper warning as to the type of tires needed for the car to be driven at its higher speeds. The duty to warn will create strict liability despite the plaintiff's prolonged use of the car at a high speed because its use at that high speed was a foreseeable use, given its design and marketing. While the car was not defective, the failure to give proper directions and specific warning was.

A man, his brother, and his cousin are charged in a common law jurisdiction with conspiracy to commit larceny. The state introduced evidence that they agreed to go to a neighbor's house to take stock certificates from a safe in the neighbor's bedroom, that they went to the house, and that they were arrested as they entered the neighbor's bedroom. The man testified that he thought the stock certificates belonged to the cousin, that the neighbor was improperly keeping them from the cousin, and that he went along to aid in retrieving the cousin's property. The brother testified that he suspected the man and the cousin of being thieves and joined up with them in order to catch them. He also testified that he made an anonymous telephone call to the police alerting them to the crime and that the call caused the police to be waiting for them when they walked into the neighbor's bedroom. The cousin did not testify. If the jury believes the brother, it should find him A: guilty, because there was an agreement, and the entry into the bedroom is sufficient for the overt act. B: guilty, because he is not a police officer and thus cannot claim any privilege of apprehending criminals. C: not guilty, because he did not intend to steal. D: not guilty, because he prevented the theft from occurring.

C is correct. The charge of conspiracy requires that two parties intentionally enter an agreement to commit a crime with the intent to achieve the objective of the agreement. If the jury believes that the brother was attempting to catch two thieves, then the brother did not have the requisite intent to achieve the objective of the agreement (larceny). Therefore, he should be found not guilty of conspiracy to commit larceny.

An accountant and a bookkeeper, as part of a contract dissolving their accounting business, agreed that each would contribute $100,000 to fund an annuity for a clerk who was a longtime employee of the business. The clerk's position would be terminated at the dissolution, and he did not have a retirement plan. The accountant and the bookkeeper informed the clerk of their plan to fund an annuity for him. The clerk, confident about his financial future because of the promised annuity, purchased a retirement home. The accountant later contributed his $100,000 to fund the annuity, but the bookkeeper stated that he could afford to contribute only $50,000. The accountant agreed in writing that the bookkeeper should contribute only $50,000. Does the clerk have a valid basis for an action against the bookkeeper for the unpaid $50,000? A: No, because the clerk was bound by the modification of the contract made by the accountant and the bookkeeper. B: No, because the clerk was only a donee beneficiary of the contract between the accountant and the bookkeeper and had no vested rights. C: Yes, because the clerk's reliance on the promised annuity prevented the parties from changing the terms of the contract. D: Yes, because the promise to establish the annuity was made binding by consideration from the clerk's many years of employment.

C is correct. The clerk was an intended beneficiary of the contract between the accountant and the bookkeeper because their promise to fund the annuity was clearly intended to benefit the clerk. The clerk materially changed his position in reliance on the promise when he purchased a retirement home. The clerk's material reliance terminated the ability of the accountant and the bookkeeper to modify their respective duties because the clerk's rights have vested. A is incorrect. The clerk materially changed his position in reasonable reliance on the contract, which means his rights have vested and the bookkeeper and accountant cannot modify the annuity unless the clerk consents. B is incorrect. As explained above, the clerk was not a donee beneficiary because the contract was expressly created to benefit the clerk. The clerk's rights vested when he purchased the retirement home in reasonable reliance on the annuity. D is incorrect. Under the bargained-for-exchange test for consideration, acts performed in the past constitute past consideration, which does not amount to the consideration required to enforce a contract. The clerk's many years of employment constituted past consideration and would be insufficient to prevent the accountant and the bookkeeper from modifying their duties regarding the promise to fund the annuity except for the clerk's subsequent reliance on their promise.

A state statute made it a misdemeanor to construct any building of more than five stories without an automatic fire sprinkler system. A local construction company built in the state a ten-story federal office building. It constructed the building according to the precise specifications of a federal contract authorized by federal statutes. Because the building was built without the automatic fire sprinkler system required by state law, the state prosecuted the private contractor. Which of the following is the company's strongest defense to that prosecution? A: The state sprinkler requirement denies the company property or liberty without due process. B: The state sprinkler requirement denies the company equal protection of the laws. C: As applied, the state sprinkler requirement violates the Supremacy Clause. D: As applied, the state sprinkler requirement violates the Obligation of Contracts Clause.

C is correct. The contractor built the building to precise specifications of a federal contract authorized by federal statutes. If the state sprinkler requirement conflicts with the federal statutes, the state sprinkler requirement cannot be enforced under the Supremacy Clause. Therefore, this is the strongest defense.

Congress recently enacted a statute creating a program that made federal loans available to family farmers who had been unable to obtain loans from private lenders. Congress appropriated a fixed sum of money to fund loans made pursuant to the program and gave a designated federal agency discretion to decide which applicants were to receive the loans. Two weeks after the program was established, a family farmer applied to the agency for a loan. Agency officials promptly reviewed her application and summarily denied it. The farmer has sued the agency in federal district court, claiming only that the denial of her application without the opportunity for a hearing violated the due process clause of the Fifth Amendment. The farmer claims that she could have proved at such a hearing that without the federal loan it would be necessary for her to sell her farm. Should the court uphold the agency's decision? A: No, because due process requires federal agencies to provide a hearing before making any factual determination that adversely affects an identified individual on the basis of his or her particular circumstances. B: No, because the denial of a loan may deprive the farmer of an established liberty interest to pursue her chosen occupation. C: Yes, because the applicable statute gives the farmer no legitimate claim of entitlement to receive a loan. D: Yes, because the spending clause of Article I, Section 8, gives Congress plenary power to control the distribution of appropriated funds in any manner it wishes.

C is correct. The court should uphold the agency's decision because the Due Process Clause does not require the government to provide the farmer with an opportunity for an administrative hearing on her loan application. The farmer had no legitimate claim of entitlement to a loan because the statute gave the agency discretion to decide which applicants were to receive the loans. The agency's denial of the farmer's application, therefore, did not deprive her of a property or liberty interest protected by the Due Process Clause.

A patron ate a spicy dinner at a restaurant on Sunday night. He enjoyed the food and noticed nothing unusual about the dinner. Later that evening, the patron had an upset stomach. He slept well through the night, went to work the next day, and ate three meals. His stomach discomfort persisted, and by Tuesday morning he was too ill to go to work. Eventually, the patron consulted his doctor, who found that the patron was infected with a bacterium that can be contracted from contaminated food. Food can be contaminated when those who prepare it do not adequately wash their hands. The patron sued the restaurant for damages. He introduced testimony from a health department official that various health code violations had been found at the restaurant both before and after the patron's dinner, but that none of the restaurant's employees had signs of bacterial infection when they were tested one month after the incident. The restaurant's best argument in response to the patron's suit would be that A: no one else who ate at the restaurant on Sunday complained about stomach discomfort. B: the restaurant instructs its employees to wash their hands carefully and is not responsible if any employee fails to follow these instructions. C: the patron has failed to establish that the restaurant's food caused his illness. D: the patron assumed the risk of an upset stomach by choosing to eat spicy food.

C is correct. The defense with the best chance of prevailing. Health code violations can only establish duty and breach; they do not establish the restaurant's causal control over the specific instrumentality that caused the actual food poisoning. In addition, res ipsa loquitur does not apply because the patron was unable to show that the restaurant had exclusive control over everything the patron ate within the period leading up to his illness. As a consequence, the restaurant's negligence cannot be inferred by the circumstances.

By warranty deed, a woman conveyed Blackacre to her friend and her neighbor "as joint tenants with right of survivorship." The friend and neighbor are not related. The friend conveyed all her interest to her boyfriend by warranty deed and subsequently died intestate. Thereafter, the neighbor conveyed to his girlfriend by warranty deed. There is no applicable statute, and the jurisdiction recognizes the common law joint tenancy. Title to Blackacre is in A: the girlfriend. B: the woman. C: the girlfriend and the boyfriend. D: the girlfriend and the heirs of the friend.

C is correct. The facts state that the property was held by the friend and neighbor in joint tenancy. Where one joint tenant makes an inter vivos conveyance of their interest in a property, a severance of the joint tenancy occurs and the interest transferred is that of a tenant in common. As soon as the friend made an inter vivos conveyance to her boyfriend, the joint tenancy was severed, and he and the neighbor became tenants in common. As such, the neighbor's conveyance to his girlfriend was likewise a transfer of an interest as a tenant in common. Therefore, the girlfriend and boyfriend hold title as tenants in common. A is incorrect. The neighbor and the friend both made valid conveyances of their interest in the property as tenants in common. Thus, the boyfriend has an interest in the property as well as the girlfriend. B is incorrect. The woman did not retain any interest in the property because she conveyed her entire interest to the friend and the neighbor. D is incorrect. As explained above, the friend's conveyance severed the joint tenancy, so the friend's heirs will receive nothing

An accountant from State A sued a biologist from State A in a federal court in State A. The accountant did not state a claim based on federal law in her complaint. However, the accountant believes the biologist will use a federal law in his defense. Should the court dismiss the accountant's complaint? A: No, because the judge should wait and determine if any pleadings, filed by either side, contain a federal question. B: No, because the judge should wait to see whether the biologist's answer contains a defense based on a federal question. C: Yes, because the complaint does not contain a federal question. D: Yes, because the complaint did not state the anticipated defense based in federal law.

C is correct. The federal question establishing subject-matter jurisdiction must appear as part of the accountant's cause of action as set out in a well-pleaded complaint. The fact that the biologist's defense might be based in federal law is irrelevant and does not establish federal question jurisdiction over the accountant's claim. The court may not look to a defense asserted by the biologist to determine whether the accountant's complaint raises a federal question.

A defendant is on trial for robbing a particular bank with his best friend. The friend has left the country and cannot be found. The prosecutor has called the friend's mother to testify to a conversation she had with her son the day before he left the country. She implored him not to go, but he said, "Mom, I have to go. I was involved in a robbery at [the bank], and I don't want them to catch me." The prosecutor has other evidence indicating that the defendant and the friend were together on the morning of the robbery. Is the statement by the friend to his mother admissible? A: No, because inculpatory statements against penal interest do not satisfy the confrontation clause. B: No, because the friend's statement to his mother was not corroborated. C: Yes, as a statement against penal interest that is not testimonial under the confrontation clause. D: Yes, as a statement of the friend's state of mind that is not testimonial under the confrontation clause.

C is correct. The friend's statement to his mother satisfies the requirements of the statement against interest exception to the hearsay rule, and it does not violate the Confrontation Clause because it was made to a family member, not for the purpose of creating a substitute for in-court testimony.

A seller entered into a written contract to sell a tract of land to an investor. The contract made no mention of the quality of title to be conveyed. The seller and the investor later completed the sale, and the seller delivered a warranty deed to the investor. Soon thereafter, the value of the land increased dramatically. The investor entered into a written contract to sell the land to a buyer. The contract between the investor and the buyer expressly provided that the investor would convey a marketable title. The buyer's attorney discovered that the title to the land was not marketable and had not been marketable when the original seller had conveyed to the investor. The buyer refused to complete the sale. The investor sued the original seller for breach of contract, claiming damages from the seller's failure to convey marketable title, which resulted in the investor's loss of the sale to the subsequent buyer. Who is likely to prevail on this count? A: The investor, because the law implies in such a contract a covenant that the title will be marketable. B: The investor, because the original seller is liable for all reasonably foreseeable damages. C: The original seller, because her contract obligations as to title merged into the deed. D: The original seller, because she did not expressly agree to convey marketable title.

C is correct. The investor's remedy, if there is one, would be based on the deed he received and not on the contract of sale. A is incorrect. The investor's remedy is solely based on the deed he received. The seller will prevail on the breach of contract claim. B is incorrect. The facts do not justify any other claim for damages (e.g., misrepresentation). Accordingly, the seller will prevail on the breach of contract claim. D is incorrect. Even if the contract of sale is silent regarding title matters, the law will imply a requirement of a marketable title in a contract for the sale of land. If the investor has a remedy, it would be based on the deed he received, not on the contract of sale.

A defendant left her car parked on the side of a hill. Two minutes later, the car rolled down the hill and struck and injured the plaintiff. In the plaintiff's negligence action against the defendant, the plaintiff introduced into evidence the facts stated above, which are undisputed. The defendant testified that, when she parked her car, she turned the front wheels into the curb and put on her emergency brakes, which were in good working order. She also introduced evidence that, in the weeks before this incident, juveniles had been tampering with cars in the neighborhood. The jury returned a verdict in favor of the defendant, and the plaintiff properly moved for a judgment notwithstanding the verdict. The plaintiff's motion should be A: granted, because it is more likely than not that the defendant's negligent conduct was the legal cause of the plaintiff's injuries. B: granted, because the evidence does not support the verdict. C: denied, because, given the defendant's evidence, the jury was not required to draw an inference of negligence from the circumstances of the accident. D: denied, because the defendant was in no better position than the plaintiff to explain the accident

C is correct. The jury's verdict was reasonable in light of the defendant's evidence countering the plaintiff's res ipsa loquitur showing, which would have allowed an inference of negligence. The defendant offered plenty of evidence to defeat an inference of her negligence, and the motion should be denied.

A man contacted his lawyer regarding his right to use a path that was on his neighbor's vacant land. Fifteen years ago, after part of a path located on his land and connecting his cabin to the public highway washed out, the man cleared a small part of his neighbor's land and rerouted a section of the path through the neighbor's land. Twelve years ago, the neighbor leased her land to some hunters. For the next 12 years, the hunters and the man who had rerouted the path used the path for access to the highway. A month ago, the neighbor discovered that part of the path was on her land. The neighbor told the man that she had not given him permission to cross her land and that she would be closing the rerouted path after 90 days. The man's land and the neighbor's land have never been in common ownership. The period of time necessary to acquire rights by prescription in the jurisdiction is 10 years. The period of time necessary to acquire title by adverse possession in the jurisdiction is 10 years. What should the lawyer tell the man concerning his right to use the rerouted path on the neighbor's land? A: The man has fee title by adverse possession of the land included in the path. B: The man has an easement by necessity to use the path. C: The man has an easement by prescription to use the path. D: The man has no right to use the path.

C is correct. The man has used the path for the past 15 years without the neighbor's permission, openly and continuously, and has acquired an easement by prescription. A is incorrect. The man is not claiming to own the land, and thus acquired an easement instead of title by adverse possession. B is incorrect. No easement by necessity could be found here because the properties were never under common ownership. D is incorrect. The man has acquired an easement by prescription from his use of the path.

A mother rushed her eight-year-old daughter to the emergency room at a local hospital after the child fell off her bicycle and hit her head on a sharp rock. The wound caused by the fall was extensive and bloody. The mother was permitted to remain in the treatment room, and held the child's hand while the emergency room physician cleaned and sutured the wound. During the procedure, the mother said that she was feeling faint and stood up to leave the room. While leaving the room, the mother fainted and, in falling, struck her head on a metal fixture that protruded from the emergency room wall. She sustained a serious injury as a consequence. If the mother sues the hospital to recover damages for her injury, will she prevail? A: Yes, because the mother was a public invitee of the hospital's. B: Yes, because the fixture was not an obvious, commonly used, and essential part of the hospital's equipment. C: No, because there is no evidence that the hospital's personnel failed to take reasonable steps to anticipate and prevent the mother's injury. D: No, because the hospital's personnel owed the mother no affirmative duty of care.

C is correct. The mother's status as an invitee imposed a duty on the hospital to warn of known dangers, inspect for others, and make safe any that are discovered. There is no evidence that hospital personnel failed to use reasonable care in connection with the fixture, which was not hidden and there was no indication that it posed a foreseeable danger of injury.

A state enacts the Young Adult Marriage Counseling Act, which provides that, before any persons less than 30 years of age may be issued a marriage license, they must receive at least five hours of marriage counseling from a statelicensed social worker. This counseling is designed to assure that applicants for marriage licenses know their legal rights and duties in relation to marriage and parenthood, understand the "true nature" of the marriage relationship, and understand the procedures for obtaining divorces. In a case in which the constitutionality of the Young Adult Marriage Counseling Act is in issue, the burden of persuasion will probably be on the A: person challenging the law, because there is a strong presumption that elected state legislators acted properly. B: person challenging the law, because the Tenth Amendment authorized states to determine the conditions on which they issue marriage licenses. C: state, because there is a substantial impact on the right to marry, and that right is fundamental. D: state, because there is a substantial impact in the discrete and insular class of young adults.

C is correct. The right to marry is fundamental, and this statute substantially impacts that right, so the statute would be subject to strict scrutiny. The state bears the burden to show that the statute will pass strict scrutiny.

On May 1, an uncle mailed a letter to his adult nephew that stated: "I am thinking of selling my pickup truck, which you have seen and ridden in. I would consider taking $7,000 for it." On May 3, the nephew mailed the following response: "I will buy your pickup for $7,000 cash." The uncle received this letter on May 5 and on May 6 mailed a note that stated: "It's a deal." On May 7, before the nephew had received the letter of May 6, he phoned his uncle to report that he no longer wanted to buy the pickup truck because his driver's license had been suspended. Which of the following statements concerning this exchange is accurate? A: There was a contract as of May 3. B: There was a contract as of May 5. C: There was a contract as of May 6. D: There is no contract.

C is correct. The uncle's original letter was not an offer. It was merely a statement indicating a possible interest in selling the truck, and a suggestion as to a price that might be acceptable. It would be regarded, if anything, as a statement soliciting an offer. The nephew's letter, mailed on May 3, constituted an offer to buy the pickup. The uncle's note, mailed on May 6, constituted an acceptance of the nephew's offer, and was effective when mailed. Therefore a contract arose on May 6. A is incorrect. There was no contract formed on May 3, because the uncle's original letter was not an offer. It was merely a statement indicating a possible interest in selling the truck and a suggestion as to a price that might be acceptable. It would be regarded, if anything, as a statement soliciting an offer. The nephew's letter, mailed on May 3, constituted an offer to buy the pickup truck. The uncle's note, mailed on May 6, constituted an acceptance of the nephew's offer, and was effective when mailed. And so a contract arose on May 6. B is incorrect. On May 5 the uncle received the offer from the nephew, but had not yet accepted it. As a result, there was no contract on May 5. The uncle's note, mailed on May 6, constituted an acceptance of the nephew's offer, and was effective when mailed. Therefore a contract arose on May 6. D is incorrect. The nephew's letter, mailed on May 3, constituted an offer to buy the pickup truck. The uncle's note, mailed on May 6, constituted an acceptance of the nephew's offer, and was effective when mailed. And so a contract arose on May 6. The law would treat the nephew's May 3 letter as an offer, even though the nephew might have mistakenly believed it to be an acceptance of an offer. The nephew's May 7 phone call was too late to constitute a revocation of his May 3 offer, since it had already been accepted. The nephew's license revocation would not constitute a defense to the existence of the contract for sale.

A defendant was charged with the sale of narcotics. The federal prosecutor arranged with the defendant's wife for her to testify against her husband in exchange for leniency in her case. At trial, the prosecution calls the wife, who had been granted immunity from prosecution, to testify, among other things, that she saw her husband sell an ounce of heroin. Which of the following statements is most clearly correct in the federal courts? A: The defendant's wife cannot be called as a witness over her husband's objection. B: The defendant's wife can be called as a witness but cannot testify, over the defendant's objection, that she saw him sell heroin. C: The defendant's wife can refuse to be a witness against her husband. D: The defendant's wife can be required to be a witness and to testify that she saw her husband sell heroin.

C is correct. The wife's proposed testimony could lead to her invoking the adverse spousal privilege, a privilege against testifying against her husband. That privilege, however, only exists at the wife's discretion. The witness spouse, in this case the defendant's wife, alone has a privilege to refuse to testify adversely against her husband; she may be neither compelled to testify nor foreclosed from testifying. The choice is hers.

A defendant is charged with mail fraud. At trial, the defendant has not taken the witness stand, but he has called a witness who has testified that the defendant has a reputation for honesty. On cross-examination, the prosecutor seeks to ask the witness, "Didn't you hear that two years ago the defendant was arrested for embezzlement?" Should the court permit the question? A: No, because the defendant has not testified and therefore has not put his character at issue. B: No, because the incident was an arrest, not a conviction. C: Yes, because it seeks to impeach the credibility of the witness. D: Yes, because the earlier arrest for a crime of dishonesty makes the defendant's guilt of the mail fraud more likely.

C is correct. The witness has testified that she knows about the defendant's reputation for honesty, which opened the door. The prosecutor then has the right to test the basis and adequacy of that knowledge, as well as the nature of the community itself, through inquiry into specific acts on cross-examination.

A defendant is charged with aggravated assault on a game warden. The defendant testified that, when he was confronted by the warden, who was armed and out of uniform, the defendant believed the warden was a robber and shot in self-defense. The state calls a witness to testify that a year earlier, he had seen the defendant shoot a man without provocation and thereafter falsely claimed self-defense. The witness's testimony is A: admissible, as evidence of the defendant's untruthfulness. B: admissible, as evidence that the defendant did not act in self-defense on this occasion. C: inadmissible, because it is improper character evidence. D: inadmissible, because it is irrelevant to the defense the defendant raised.

C is correct. The witness's testimony is improper extrinsic character evidence intended to prove the defendant's propensity for violence. The state cannot initiate evidence of the defendant's bad character merely to show that he is more likely to have committed the crime.

A woman wanted to kill a business competitor. She contacted a man who she believed was willing to commit murder for hire and offered him $50,000 to kill the competitor. The man agreed to do so and accepted $25,000 as a down payment. Unbeknownst to the woman, the man was an undercover police officer. In a jurisdiction that has adopted the unilateral theory of conspiracy, is the woman guilty of conspiracy to murder the business competitor? A: No, because the man did not intend to kill the competitor. B: No, because it would have been impossible for the woman to kill the competitor by this method. C: Yes, because the woman believed that she had an agreement with the man that would bring about the competitor's death. D: Yes, because the woman took a substantial step toward bringing about the competitor's death by paying the man $25,000.

C is correct. The woman agreed to commit a crime, and she committed an overt act in furtherance of that agreement when she paid the man $25,000. She, therefore, is guilty of conspiracy in a jurisdiction that recognizes unilateral conspiracies.

A man and a woman planned to hold up a bank. They drove to the bank in the man's car. The man entered while the woman remained as lookout in the car. After a few moments, the woman panicked and drove off. The man looked over the various tellers, approached one and whispered nervously, "Just hand over the cash. Don't look around, don't make a false move--or it's your life." The teller looked at the fidgeting man, laughed, flipped him a dollar bill and said, "Go on, beat it." Flustered, the man grabbed the dollar and left. Soon after leaving the scene, the woman was stopped by the police for speeding. Noting her nervous condition, the police asked the woman if they might search the car. She agreed. The search turned up heroin concealed in the lid of the trunk. The prosecution's best argument to sustain the validity of the search of the man's car would be that A: the search was reasonable under the circumstances, given the woman's nervous condition. B: the search was incident to a valid arrest. C: the woman had, under the circumstances, sufficient standing and authority to consent to the search. D: exigent circumstances, including the inherent mobility of a car, justified the search.

C is correct. The woman, as the driver and sole person in the vehicle, had standing and the apparent authority to consent to the search of the vehicle, even if she did not own the vehicle. By obtaining the woman's consent to search the vehicle, the police acted reasonably, and the evidence should be admitted.

Public schools in a state are financed, in large part, by revenue derived from real estate taxes imposed by each school district on the taxable real property located in that district. Public schools also receive other revenue from private gifts, federal grants, student fees, and local sales taxes. For many years, the state has distributed additional funds, which come from the state treasury, to local school districts in order to equalize the funds available on a perstudent basis for each public school district. These additional funds are distributed on the basis of a state statutory formula that considers only the number of students in each public school district and the real estate tax revenue raised by that district. The formula does not consider other revenue received by a school district from different sources. The school boards of two school districts, together with parents and schoolchildren in those districts, bring suit in federal court to enjoin the state from allocating the additional funds from the state treasury to this formula. They allege that the failure of the state, in allocating this additional money, to take into account a school district's sources of revenue other than revenue derived from taxes levied on real estate located there violates the Fourteenth Amendment. The complaint does not allege that the allocation of the additional state funds based on the current statutory formula has resulted in a failure to provide minimally adequate education to any child. Which of the following best describes the appropriate standard by which the court should review the constitutionality of the state statutory funding formula? A: Because classifications based on wealth are inherently suspect, the state must demonstrate that the statutory formula is necessary to vindicate a compelling state interest. B: Because the statutory funding formula burdens the fundamental right to education, the state must demonstrate that the formula is necessary to vindicate a compelling state interest. C: Because no fundamental right or suspect classification is implicated in this case, the plaintiffs must demonstrate that the funding allocation formula bears no rational relationship to any legitimate state interest. D: Because the funding formula inevitably leads to disparities among the school districts in their levels of total funding, the plaintiffs must only demonstrate that the funding formula is not substantially related to the furtherance of an important state interest.

C is correct. There is no fundamental right to education, and wealth is not a suspect class. Because there is no fundamental right or suspect class at issue, the court will apply rational basis review, which would require the plaintiffs to demonstrate that the funding allocation formula is not rationally related to any conceivable legitimate state interest.

A state legislature recently enacted a statute forbidding public utilities regulated by the state's public service commission to increase their rates more than once every two years. A power company, a public utility regulated by that commission, has just obtained approval of the commission for a general rate increase. The power company has routinely filed for a rate increase every ten to 14 months during the last 20 years. Because of uncertainties about future fuel prices, the power company cannot ascertain with any certainty the date when it will need a further rate increase; but it thinks it may need such an increase sometime within the next 18 months. The power company files an action in the federal district court in the state requesting a declaratory judgment that this new state statute forbidding public utility rate increases more often than once every two years is unconstitutional. Assume no federal statute is relevant. In this case, the court should A: hold the statute unconstitutional, because such a moratorium on rate increases deprives utilities of their property without due process of law. B: hold the statute constitutional, because the judgment of a legislature on a matter involving economic regulation is entitled to great deference. C: dismiss the complaint, because this action is not ripe for decision. D: dismiss the complaint, because controversies over state-regulated utility rates are outside of the jurisdiction conferred on federal courts by Article III of the Constitution.

C is correct. There is no injury yet to the power company. It has not yet been denied a rate increase, and it does not even know whether it will need to seek a rate increase before the statute will allow it to. Therefore, there is not yet an actual or imminent injury, and the claim is not yet ripe for decision.

While driving at a speed in excess of the statutory limit, the defendant negligently collided with another car, and the disabled vehicles blocked two of the highway's three northbound lanes. When the plaintiff approached the scene two minutes later, he slowed his car to see if he could help those involved in the collision. As he slowed, he was rearended by the driver of another vehicle. The plaintiff, who sustained damage to his car and was seriously injured, brought an action against the defendant to recover damages. The jurisdiction adheres to the traditional common law rules pertaining to contributory negligence. If the defendant moves to dismiss the action for failure to state a claim upon which relief may be granted, should the motion be granted? A: Yes, because it was the driver, not the defendant, who collided with the plaintiff's car and caused the plaintiff's injuries. B: Yes, because the plaintiff could have safely passed the disabled vehicles in the traffic lane that remained open. C: No, because a jury could find that the plaintiff's injury arose from a risk that was a continuing consequence of the defendant's negligence. D: No, because the defendant was driving in excess of the statutory limit when he negligently caused the first accident.

C is correct. This question addresses the issue of causation in a negligence action, and causation requires that the defendant's negligence be both the actual and legal cause of the plaintiff's injuries. The defendant is the actual cause of the plaintiff's injuries because but for the defendant's negligent speeding, the plaintiff would not have been injured. Furthermore, it is possible the defendant is the legal cause of the plaintiff's injuries because it is foreseeable that speeding would cause an accident, and that the accident would create a dangerous road condition leading to subsequent accidents.

On June 1, a widget manufacturer entered into a written agreement with a tool maker in which the tool maker agreed to produce and sell to the manufacturer 12 sets of newly designed dies to be delivered August 1 for the price of $50,000, payable 10 days after delivery. Encountering unexpected expenses in the purchase of special alloy steel required for the dies, the tool maker advised the manufacturer that production costs would exceed the contract price; and on July 1, the manufacturer and the tool maker signed a modification to the June 1 agreement increasing the contract price to $60,000. After timely receipt of 12 sets of dies conforming to the contract specifications, the manufacturer paid the tool maker $50,000 but refused to pay more. Which of the following concepts of the Uniform Commercial Code, dealing expressly with the sale of goods, best supports an action by the tool maker to recover $10,000 for breach of the manufacturer's July 1 promise? A: Bargained-for exchange. B: Promissory estoppel. C: Modification of contracts without consideration. D: Unconscionability in the formation of contracts.

C is correct. Under the UCC, a modification to a contract for the sale of goods needs no additional consideration to be binding if the modification was made in good faith. Although the modification increasing the purchase price under the tool maker-manufacturer contract was not supported by consideration, the modification was made in light of unexpected increased production costs, which suggests that it was proposed in good faith rather than to coerce the manufacturer into paying a higher price. A is incorrect. The UCC does not require consideration for good faith modifications. B and D are incorrect. The facts do not show either detrimental reliance or unconscionability.

A famous chef entered into a written agreement with his friend, a well-known interior decorator respected for his unique designs, in which the decorator agreed, for a fixed fee, to design the interior of the chef's new restaurant, and, upon the chef's approval of the design plan, to decorate and furnish the restaurant accordingly. The agreement was silent as to assignment or delegation by either party. Before beginning the work, the decorator sold his decorating business to Newman under an agreement in which the decorator assigned to Newman, and Newman agreed to complete, the chef-decorator contract. Newman, also an experienced decorator of excellent repute, advised the chef of the assignment, and supplied him with information confirming both Newman's financial responsibility and past commercial success. Is the chef obligated to permit Newman to perform the chef-decorator agreement? A: Yes, because the agreement contained no prohibition against assignment or delegation. B: Yes, because the chef received adequate assurances of Newman's ability to complete the job. C: No, because the decorator's duties were of a personal nature, involving his reputation, taste, and skill. D: No, because the decorator's purported delegation to Newman of his obligations to the chef effected a novation.

C is correct. Unless the contract provides otherwise, a contractual duty may be delegated to another unless the other party to the contract has a substantial interest in having the original obligor perform. Typically the other party will have such an interest where the contract is a personal services contract involving fancy, taste and judgment. Although the chef-decorator contract was silent on the issue of assignability, the facts suggest that the chef would have a substantial interest in having the decorator and no other person design the interior of his new restaurant. Therefore, the decorator's delegation of the duty to Newman without the chef's consent amounts to a breach of contract. A is incorrect because delegation is restricted in this case even though the contract is silent on the issue. B is incorrect because the chef need not accept the delegation regardless of assurances of performance from Newman. D is incorrect because a novation would require express agreement from the chef to substitute Newman for the decorator.

A real estate company acquired a large tract of land upon which the company developed a mobile home subdivision. The tract was divided into 60 lots, appropriate utilities were installed, and a plat of the entire tract, including a Declaration of Restrictions, was properly drawn and recorded. The Declaration of Restriction included the following: "3. Ownership and/or occupancy are restricted to persons 21 years of age or over, one family per lot." As the separate lots were sold, the deed to each lot included the following provision: "As shown on recorded plat [properly identified by page and plat book reference] and subject to the restrictions therein contained." One of the lots was purchased by a man, who now resides in a mobile home on the lot together with his wife and two children, aged 11 and 13. Other lot owners in the subdivision brought an action against the man to enjoin further occupancy by the children under 21 years of age. If judgment is entered for the man, the issue that most likely will determine the case will be whether A: the mobile home is treated as personalty or realty. B: the restriction constitutes an unlawful restraint on alienation. C: enforcement of the restriction is considered a violation of the Equal Protection Clause of the Fourteenth Amendment of the United States Constitution. D: the terms of the restriction are expressly repeated verbatim in the man's deed.

C is correct. While the Equal Protection Clause does not normally apply to private conduct, if the government affirmatively facilitates unconstitutional activity, the conduct is subject to review. Therefore, if the lot owners are seeking to enforce an age-based restrictive covenant via the courts, the entanglement exception applies and an equal protection review of the classification is triggere

1. At the defendant's trial for burglary, one of the defendant's friends supported the defendant's alibi that they were fishing together at the time of the crime. On cross-examination, the friend was asked whether his statement on a credit card application that he had worked for his employer for the last five years was false. The friend denied that the statement was false. The prosecutor then calls a witness, the manager of the company for which the friend works, to testify that although the friend had been first employed five years earlier and is now employed by the company, there had been a three-year period during which he had not been so employed. The testimony of the witness is A) Admissible, in the judge's discretion, because the friend's credibility is a fact of major consequence to the case. B) Admissible, as a matter of right, because the friend "opened the door" by his denial on cross-examination. C) Inadmissible, because whether the friend lied in his application is a matter that cannot be proved by extrinsic evidence. D) Inadmissible, because the misstatement by the friend could have been caused by a misunderstanding of the application form.

C) Inadmissible, because whether the friend lied in his application is a matter that cannot be proved by extrinsic evidence.

1. Pike sued Day City Community Church for damages he suffered when Pike crashed his motorcycle in an attempt to avoid a cow that had escaped from its corral. The cow and corral belonged to a farm that had recently been left by will to the church. At trial, Pike seeks to ask Defendant's witness, Winters, whether she is a member of that church. The question is A) Improper, because evidence of a witness's religious beliefs is not admissible to impeach credibility. B) Improper, because it violates First Amendment and privacy rights. C) Proper, for the purpose of ascertaining partiality or bias. D) Proper, for the purpose of showing capacity to appreciate the nature and obligation of an oath.

C) Proper, for the purpose of ascertaining partiality or bias.

1. The police had, over time, accumulated reliable information that Jason operated a large cocaine-distribution network, that he and his accomplices often resorted to violence, and that they kept a small arsenal of weapons in his home. One day, the police received reliable information that a large brown suitcase with leather straps containing a supply of cocaine had been delivered to Jason's home and that it would be moved to distribution point the next morning. The police obtained a valid search warrant to search for and seize the brown suitcase and the cocaine and went to Jason's house. The police knocked on Jason's door and called out, "Police. Open up. We have a search warrant." After a few seconds with no response, the police forced the door open and entered. Nearing noises in the basement, the police ran down there and found Jason with a large brown suitcase and put handcuffs on Jason. A search of his person revealed a switchblade knife and a .45-caliber pistol. Jason cursed the police and said, "You never would have caught me with the stuff if it hadn't been for that lousy snitch Harvey!" The police then fanned out through the house, looking in every room and closet. They found no one else, but one officer found an Uzi automatic weapon in a box on a closet shelf in Jason's bedroom. In addition to charges relating to the cocaine in the suitcase, Jason is charged with unlawful possession of weapons. Jason moves pretrial to suppress the use as evidence of the weapons seized by the police and of the statement he made. As to Jason's statement, his motion to suppress should be A- Granted, because entry by forcing open the door was not reasonable. B- Granted, because the police failed to read Jason his Miranda rights. C- Denied, because the statement was volunteered. D- Denied, because the statement was the product of a lawful public safety se

C- Denied, because the statement was volunteered.

1- Congress passed a bill prohibiting the President from granting a pardon to any person who had not served at least one-third of the sentence imposed by the court which convicted the person. The President vetoed the bill, claiming that it was unconstitutional. Nevertheless, Congress passed it over his veto by a two-thirds vote of each house. This act of Congress is A- Constitutional, because it was enacted over the President's veto by a two-thirds vote of each house. B- Constitutional, because it is a necessary and proper means of carrying out the powers of Congress. C- Unconstitutional, because it interferes with the plenary power of the President to grant pardons. D- Unconstitutional, because a Presidential veto based upon constitutional grounds may be overridden only with the concurrence of three-fourths of the state legislatures.

C- Unconstitutional, because it interferes with the plenary power of the President to grant pardons.

1- A federal statute imposes an excise tax of $100 on each new computer sold in the United States. It also appropriates the entire proceeds of that tax to a special fund, which is required to be used to purchase licenses for computer software that will be made available for use, free of charge, to any resident of the United States. Is this statute constitutional? A- No, because the federal government may not impose any direct taxes on citizens of the United States. B- No, because this statute takes without just compensation the property of persons who hold patents or copyrights on computer software. C- Yes, because it is a reasonable exercise of the power of Congress to tax and spend for the general welfare. D- Yes, because the patent power authorizes Congress to impose reasonable charges on the sale of technology and to spend the proceeds of those charges to advance the use of technology in the United States.

C- Yes, because it is a reasonable exercise of the power of Congress to tax and spend for the general welfare.

1) A four-year-old child sustained serious injuries when a playmate pushed him from [SB1] between two parked cars into the street, where he was struck by a car. The child, by his representative, sued the driver of the car, the playmates parents, and his own parents. At trial, the child's total injuries were determined to be $100,000. The playmate's parents were determined to be 20% at fault because they had failed to adequately supervise her. The driver was found to be 50% at fault. The child's own parents were determined to be 30% at fault for failure to adequately supervise him. The court has adopted the pure comparative negligence doctrine, with joint and several liability, in place of the common-law rules relating to plaintiff's fault. In addition, the common-law doctrines relating to intra-family liability have been abrogated. How much, if anything, is the child's representative entitled to recover from the driver? A) $30,000 B) $50,000 C) $100,000 D) Nothing

C. $100,000 Always start with how much damage does the plaintiff have, then with what type of damage you are applying.

1) Traveler was a passenger on a commercial aircraft owned and operated by Airline. The aircraft crashed into a mountain, killing everyone on board. The flying weather was good. Traveler's legal representative brought a wrongful death action against Airline. At trial, the legal representative offered no expert or other testimony as to the cause of the crash. On Airline's motion to dismiss at the conclusion of the legal representative's case, the court should A) Grant the motion, because the legal representative has offered no evidence as to the cause of the crash. B) Grant the motion, because the legal representative has failed to offer evidence negating the possibility that the crash may have been caused by mechanical failure that Airline could not have prevented. C) Deny the motion, because the jury may infer that the aircraft crashed due to Airline's negligence. D) Deny the motion, because in the circumstances common carriers are strictly liable.

C. Deny the motion, because the jury may infer that the aircraft crashed due to Airline's negligence.

1) While approaching an intersection with the red light against him, Motorist suffered a heart attack that rendered him unconscious. Motorist's car struck Child, who was crossing the street with the green in her favor. Under the state motor vehicle code, it is an offense to drive through red traffic light. Child sued Motorist to recover for her injuries. At trial it was stipulated that (1) immediately prior to suffering the heart attack, Motorist had been driving within speed limit, had seen the red light, and had begun to slow his car; (2) Motorist had no history of heart disease and no warning of this attack; (3) while Motorist was unconscious, his car ran the red light. On cross motions for directed verdicts on the issue of liability at the conclusion of the proofs, the court should A) Grant Child's motion, because Motorist ran a red light in violation of the motor vehicle code. B) Grant Child's motion, because in the circumstances, reasonable persons would infer that Motorist was negligent. C) Grant Motorist's motion, because he had no history of heart disease or warning of the heart attack. D) Deny both motions and submit the case to the jury, to determine whether, in the circumstances, Motorist's conduct was that of a reasonably prudent person.

C. Grant Motorist's motion, because he had no history of heart disease or warning of the heart attack. talking about res ipsa

1) When a mother visited a bowling alley to participate in the weekly bowling league competition held there, she brought her two-year-old son along and left him in a nursery provided by the bowling alley for the convenience of its customers. The children in the nursery were normally supervised by three attendants, but at this particular time, as the mother knew, there was only one attendant present to care for about 20 children of assorted ages. About 30 minutes later, while the attendant was looking the other way, the son suddenly started to cry. The attendant found him lying on his back, picked him up, and called his mother. It was later discovered that the son had suffered a skull fracture. If a claim is asserted against the bowling alley on the son's behalf, will the son prevail? A) Yes, because the bowling alley owed the child the highest degree of care. B) Yes, because a 2-year-old is incapable of contributory negligence. C) No, because the bowling alley and its employees exercised reasonable care to assure the son's safety. D) No, because the mother assumed the risk by leaving her son in the nursery.

C. No, because the bowling alley and its employees exercised reasonable care to assure the son's safety. based off what the facts say because they did not give causation

1- The National AIDS Prevention and Control Act is a new, comprehensive federal statute that was enacted to deal with the public health crisis caused by the AIDS virus. Congress and the President were concerned that inconsistent lower court rulings with respect to the constitutionality, interpretation, and application of the statute might adversely affect or delay its enforcement and, thereby, jeopardize the public health. As a result, they included a provision in the statute providing that all legal challenges concerning those matters may be initiated only by filing suit directly in the United States Supreme Court. The provision authorizing direct review of the constitutionality, interpretation, or application of this statute only in the United States Supreme Court is [SB1] A- Constitutional, because it is authorized by the Article I power of Congress to enact all laws that are "necessary and proper" to implement the general welfare. B- Constitutional, because Article III provides that the jurisdiction of the United States Supreme Court is subject to such exceptions and such regulations as Congress shall make. C- Unconstitutional, because it denies persons who wish to challenge this statute the equal protection of the laws by requiring them to file suit in a court different from that in which persons who wish to challenge other statutes may file. D- Unconstitutional, because it is inconsistent with the specification in Article III of the original jurisdiction of the United States Supreme Court.

D - Unconstitutional, because it is inconsistent with the specification in Article III of the original jurisdiction of the United States Supreme Court.

A wallpaper hanger sent a general contractor, this telegram: Will do all paperhanging on new Doctors' Building, per owner's specs, for $14,000 if you accept within reasonable time after main contract awarded. /s/ the wallpaper hanger Three other competing hangers sent the general contractor similar bids in the respective amounts of $18,000, $19,000, and $20,000. The general contractor used the wallpaper hanger's $14,000 figure in preparing and submitting her own sealed bid on Doctors' Building. Before the bids were opened, the wallpaper hanger truthfully advised the general contractor that the former's telegraphic sub-bid had been based on a $4,000 computational error and was therefore revoked. Shortly thereafter, the general contractor was awarded the Doctors' Building construction contract and subsequently contracted with another paperhanger for a price of $18,000. The general contractor now sues the wallpaper hanger to recover $4,000. Which of the following, if proved, would best support the wallpaper hanger's defense? A: The general contractor gave the wallpaper hanger no consideration for an irrevocable sub-bid. B: The wallpaper hanger's sub-bid expressly requested the general contractor's acceptance after awarding of the main contract. C: Even after paying $18,000 for the paperhanging, the general contractor would make a net profit of $100,000 on the Doctors' Building contract. D: Before submitting her own bid, the general contractor had reason to suspect that the wallpaper hanger had made a computational mistake in figuring his sub-bid.

D is correct. A contract may be rescinded on grounds of unilateral mistake if the mistake relates to a fundamental assumption of the contract that has a material effect on the exchange, if the party asserting mistake should not bear the risk of the mistake and if either (i) the non-mistaken party has reason to know of the mistake; or (ii) the effect of the mistake would make enforcement of the contract unconscionable. The wallpaper hanger may argue that the subcontract should be rescinded on grounds of the $4,000 mistake that he made in computing his sub-bid. If the general contractor had reason to know of the computational error, this would greatly support the wallpaper hanger's argument. A is incorrect. The bid may be found to be irrevocable on a reliance theory even in the absence of an option promise supported by consideration. B is incorrect. This answer choice is irrelevant. It would not be a defense that the wallpaper hanger's sub-bid expressly requested acceptance by the general contractor. An option promise is simply an option, which is defined as a promise which meets the requirements for the formation of a contract and limits the promisor's power to revoke an offer. C is incorrect. The mere fact that the general contractor would make a substantial profit on the contract does not make the effect of the mistake unconscionable.

A state statute provides that only citizens of the United States may be employed by that state. In an action brought in a federal court, a resident alien who was prevented from obtaining state employment as a garbage collector solely because of his alien status challenged the statute's constitutionality as applied to his circumstances. Which of the following statements concerning the burden of persuasion applicable to this suit is correct? A: The alien must demonstrate that there is no rational relationship between the citizenship requirement and any legitimate state interest. B: The alien must demonstrate that the citizenship requirement is not necessary to advance a compelling state interest. C: The state must demonstrate that there is a rational relationship between the citizenship requirement and a legitimate state interest. D: The state must demonstrate that the citizenship requirement is necessary to advance a compelling state interest.

D is correct. Alienage is a suspect class. Therefore, the state may only classify on the basis of alienage if the state can prove that the classification is narrowly tailored to promote a compelling state interest. State laws requiring United States citizenship for welfare benefits, civil service jobs, or a license to practice law will be struck down because there is no compelling state interest. However, there is an exception for alien participation in self-government. If a law discriminates against alien participation in the functioning of the state government, only rational basis review is applied. Certain privileges fall under this exception and have been reserved just for citizens: voting, serving on a jury, acting as a police officer, teacher, or probation officer, or any other position with a direct effect on the functioning of government. Garbage collector falls under a civil service job and is not part of the exception for participation in selfgovernment. Therefore, strict scrutiny is used and not rational basis review.

The owner of a shopping mall hired a construction company to design and construct a new entryway to the mall. The construction company negligently selected an unusually slippery material for the floor covering. A week after the entryway was completed, a customer who had come to the mall to buy cosmetics slipped on the floor of the entryway, sustaining injuries. The customer sued the mall owner for the construction company's negligent design of the mall's entryway. Will the injured customer be likely to recover damages? A: No, because the construction company will likely be considered an independent contractor. B: No, because no other customers had previously slipped on the floor. C: Yes, because the customer intended to make a purchase at the mall. D: Yes, because the mall's duty to maintain safe conditions was nondelegable.

D is correct. Although employers are not usually liable for the negligence of independent contractors, there are limits on the ability of employers to circumvent liability in this way. Work in public places often gives rise to a nondelegable duty on the part of the landowner. Here, the mall's duty to maintain safe conditions is nondelegable, so the mall--not the independent contractors--will be liable for the customer's injuries.

A plaintiff domiciled in State A brought a wrongful death action in a federal court in State A against a State B parent corporation and one of its foreign subsidiaries. The plaintiff alleged that a tire manufactured by the subsidiary in Europe had caused his wife's death in an automobile accident in Europe. The parent corporation does significant business throughout the United States, including in State A. The subsidiary conducts no business and has no employees or bank accounts in State A. The subsidiary manufactures its tires for the European market, but 2% of its tires are distributed in State A by the parent corporation. The subsidiary has moved to dismiss for lack of personal jurisdiction. Should the court grant the subsidiary's motion? A: No, because 2% of the subsidiary's tires entered State A through the stream of commerce. B: No, because of the general personal jurisdiction established over the parent corporation. C: Yes, because the accident did not occur in the United States. D: Yes, because the subsidiary lacks continuous, systematic, and substantial contacts with State A.

D is correct. Although the subsidiary put tires into the stream of commerce, this is not enough to establish the minimum contacts necessary for personal jurisdiction. The cause of action is not related to the activity in the state because the accident occurred in Europe, not in State A.

A college student purchased a large bottle of No-Flake dandruff shampoo, manufactured by a shampoo company. The box containing the bottle stated in part: "CAUTION - Use only one capful at most once a day. Greater use may cause severe damage to the scalp." The college student read the writing on the box, removed the bottle, and threw the box away. The college student's roommate asked to use the No-Flake, and college student said, "Be careful not to use too much." The roommate thereafter used No-Flake twice a day, applying two or three capfuls each time, notwithstanding the label statement that read: "Use no more than one capful per day. See box instructions." The more he used No-Flake, the more inflamed his scalp became, the more it itched, and the more he used. After three weeks of such use, the roommate finally consulted a doctor who diagnosed his problem as a serious and irreversible case of dermatitis caused by excessive exposure to the active ingredients in No-Flake. These ingredients are uniquely effective at controlling dandruff, but there is no way to remove a remote risk to a small percentage of persons who may contract dermatitis as the result of applying, for prolonged periods of time, amounts of No-Flake substantially in excess of the directions. This jurisdiction adheres to the traditional common law rules pertaining to contributory negligence and assumption of risk. Based upon the foregoing facts, if the roommate sues the shampoo company to recover damages for his dermatitis, his most promising theory of liability will be that the No-Flake shampoo A: had an unreasonably dangerous manufacturing defect. B: had an unreasonably dangerous design defect. C: was inherently dangerous. D: was inadequately labeled to warn of its dangers.

D is correct. An inadequate warning can function as a type of design defect claim where the product is in a defective condition or is unreasonably dangerous to the user based on a design or manufacturing defect. A product is unreasonably dangerous when it is dangerous beyond the expectations of the ordinary consumer because of a departure from the intended design. A type of design defect is an inadequate warning which does not provide users with clear and complete warnings of dangers, which may not be apparent.

A landlord owns and operates a 12-story apartment building containing 72 apartments, 70 of which are rented. A pedestrian has brought an action against the landlord alleging that while he was walking along a public sidewalk adjacent to the landlord's apartment building a flower pot fell from above and struck him on the shoulder, causing extensive injuries. The action was to recover damages for those injuries. If the pedestrian proves the foregoing facts and offers no other evidence explaining the accident, will his claim survive a motion for directed verdict offered by the defense? A: Yes, because the pedestrian was injured by an artificial condition of the premises while using an adjacent public way. B: Yes, because such an accident does not ordinarily happen in the absence of negligence. C: No, because the landlord is in no better position than the pedestrian to explain the accident. D: No, because there is no basis for a reasonable inference that the landlord was negligent.

D is correct. Any time negligence must be inferred, there is a res ipsa loquitur issue. For a claim based on res ipsa loquitur to prevail, the pedestrian must show that the landlord had exclusive control of the flowerpot before it fell. In this case, the landlord did not have exclusive control of the flowerpot during the relevant time frame because 70 of the units had tenants, so res ipsa loquitur may not be used to establish negligence. Therefore, the pedestrian will not be able to make his prima facie case, and the landlord's motion for a directed verdict should be granted.

A homeowner met a man, who was known to him to be a burglar, in a bar. The homeowner told the man that he needed money. He promised to pay the man $500 if the man would go to the homeowner's house the following night and take some silverware. The homeowner explained to the man that, although the silverware was legally his, his wife would object to his selling it. The homeowner pointed out his home, one of a group of similar tract houses. He drew a floor plan of the house that showed the location of the silverware. The homeowner said that his wife usually took several sleeping pills before retiring, and that he would make sure that she took them the next night. He promised to leave a window unlocked. Everything went according to the plan except that the man, deceived by the similarity of the tract houses, went to the wrong house. He found a window unlocked, climbed in and found silver where the homeowner had indicated. He took the silver to the cocktail lounge where the payoff was to take place. At that point, police arrested the two men. If the homeowner was charged with burglary, his best argument for acquittal would be that A: there was no breaking. B: he consented to the entry. C: no overt act was committed by him. D: there was no intent to commit a felony.

D is correct. Burglary is defined at common law as the breaking and entering of a dwelling of another, at nighttime, with the intent to commit a felony therein. Neither the homeowner nor the man had the intent to commit a felony, which should lead to an acquittal for the homeowner. The man's mistake regarding which house belonged to the homeowner negates the intent required for burglary. Therefore, this is also the homeowner's best argument for acquittal.

The Federal Automobile Safety Act established certain safety and performance standards for all automobiles manufactured in the United States. The Act creates a five-member "Automobile Commission" to investigate automobile safety, to make recommendations to Congress for new laws, to make further rules establishing safety and performance standards, and to prosecute violations of the act. The chairman is appointed by the President, two members are selected by the President pro tempore of the Senate, and two by the Speaker of the House of Representatives. A minor United States car manufacturer seeks to enjoin enforcement of the Commission's rules. The best argument that the car manufacturer can make is that A: legislative power may not be delegated by Congress to an agency in the absence of clear guidelines. B: the commerce power does not extend to the manufacture of automobiles not used in interstate commerce. C: the car manufacturer is denied due process of law because it is not represented on the Commission. D: the Commission lacks authority to enforce its standards because not all of its members were appointed by the President

D is correct. Congress has broad discretion to delegate its legislative power to executive officers and/or administrative agencies. To be delegable, the power must not be uniquely confined to Congress. Although Congress may appoint its own officers to carry on internal legislative tasks, it may not appoint members of a body with administrative or enforcement powers. Those members are "officers of the United States" and must be appointed by the President. Since enforcement is an executive power, the fact that some of the commission is appointed by Congress voids that power. Thus, the best argument the car manufacturer has is that the appointments by Congress would prohibit the Commission from being able to enforce its standards.

A flour wholesaler contracted to deliver to a producer of fine baked goods her flour requirements for a one-year period. Before delivery of the first scheduled installment, the flour wholesaler sold its business and "assigned" all of its sale contracts to a miller, another reputable and long-time flour wholesaler. The original flour wholesaler informed the baked goods producer of this transaction. Assume that when the miller tendered the first installment to the baked goods producer in compliance with the flour wholesaler-baked goods contract, the baked goods producer refused to accept the goods. Can the baked goods producer legally reject the goods? A: Yes; executory requirements contracts are nonassignable. B: Yes; duties under an executory bilateral contract are assumable only by an express promise to perform on the part of the delegate. C: Yes; language of "assignment" in the transfer for value of a bilateral sale-of-goods contract effects only a transfer of rights, not a delegation of duties. D: No; requirements contracts are assignable as long as the assignee does not disproportionately alter the contemplated quantity.

D is correct. Generally, all contractual rights may be assigned. At common law, requirements and output contracts were not assignable. Under Article 2 of the UCC, however, requirements and output contracts are assignable as long as the assignee does not disproportionately alter the contemplated quantity. Here, this contract is governed by Article 2, as it is for the sale of goods. The miller tendered the first installment in compliance with the original contract between the flour wholesaler and the baked goods producer. Therefore, the baked goods producer must accept the goods. A is incorrect. A party may perform his duty through a delegatee unless the other party has a substantial interest in having the original party perform the contract. Although assignment of a requirements contract may potentially affect the burden of the seller in certain cases (such as where the assignee has substantially different requirements than the assignor), courts will still uphold the assignment of such contracts because the UCC rules on requirements contracts impose reasonably objective standards of good faith in specifying requirements. In any event, the assignment to the miller, will not have such an effect, because the party assigning its contract is the seller. Because the miller is comparable to the flour wholesaler in terms of reputation and experience, the baked goods producer should not have a particular interest in having the flour wholesaler perform the contract. B is incorrect. Performance by the delegatee is sufficient. C is incorrect. Under the UCC, general language of assignment of a contract will be construed as a delegation of performance as well as an assignment of rights.

A truck driver from State A and a bus driver from State B were involved in a collision in State B that injured the truck driver. The truck driver filed a federal diversity action in State B based on negligence, seeking $100,000 in damages from the bus driver. What law of negligence should the court apply? A: The court should apply the federal common law of negligence. B: The court should apply the negligence law of State A, the truck driver's state of citizenship. C: The court should consider the negligence law of both State A and State B and apply the law that the court believes most appropriately governs negligence in this action. D: The court should determine which state's negligence law a state court in State B would apply and apply that law in this action.

D is correct. In Klaxon Co. v. Stentor Elec. Mfg. Co., 313 U.S. 487 (1941), the Court made clear that in a federal diversity action a court must look to the choice-of-law rules of the state in which it sits to determine which of two competing states' laws should be applied to the action before it.

While driving his car, the plaintiff sustained injuries in a three-vehicle collision. The plaintiff sued the drivers of the other two vehicles, a truck and a bus, and each defendant crossclaimed against the other for contribution. The jurisdiction has adopted a rule of pure comparative negligence and allows contribution based upon proportionate fault. The rule of joint and several liability has been retained. The jury has found that the plaintiff sustained damages in the amount of $100,000, and apportioned the causal negligence of the parties as follows: The plaintiff 40%, the truck driver 30%, and the bus driver 30%. How much, if anything, can the plaintiff collect from the truck driver, and how much, if anything, can the truck driver then collect from the bus driver in contribution? A: Nothing, and then the truck driver can collect nothing from the bus driver. B: $30,000, and then the truck driver can collect nothing from the bus driver. C: $40,000, and then the truck driver can collect $10,000 from the bus driver. D: $60,000, and then the truck driver can collect $30,000 from the bus driver.

D is correct. Pure comparative negligence allows the plaintiff to recover all damages not attributed to his own negligence. The plaintiff is therefore entitled to $60,000, which is the $100,000 in damages he suffered minus his 40% of the fault. Since this is a joint and several liability jurisdiction, the defendants are each liable for the entire award. The plaintiff can collect the full amount of his award from either defendant or both, as long as the total only equals the $60,000 he is entitled to. The jurisdiction allows contribution based on proportionate fault. The question asks how much (what is the most) the plaintiff can collect from the truck driver. Under joint and several liability, the plaintiff can collect up to his full award, $60,000, from the truck driver. The truck driver can then collect the bus driver's proportionate amount of the award in contribution, which is 30% of $100,000, or $30,000

A pedestrian started north across the street in a clearly marked north-south crosswalk with the green traffic light in her favor. The pedestrian was in a hurry, and so before reaching the north curb on the street, she cut to her left diagonally across the street to the east-west crosswalk and started across it. Just after reaching the east-west crosswalk, the traffic light turned green in her favor. She proceeded about five steps further across the street to the west in the crosswalk when she was struck by a car approaching from her right that she thought would stop, but did not. The car was driven by a driver, 81 years of age, who failed to stop his car after seeing that the traffic light was red against him. The pedestrian had a bone disease, resulting in very brittle bones, that is prevalent in only 0.02 percent of the population. As a result of the impact, the pedestrian suffered a broken leg and the destruction of her family heirloom, a Picasso original painting that she was taking to her bank for safekeeping. The painting had been purchased by the pedestrian's grandmother for $750 but was valued at $500,000 at the time of the accident. The pedestrian has filed suit against the driver. The driver's attorney has alleged that the pedestrian violated a state statute requiring that pedestrians stay in crosswalks, and that if the pedestrian had not violated the statute, she would have had to walk 25 feet more to reach the impact point and therefore would not have been at a place where she could have been hit by the driver. The pedestrian's attorney ascertains that there is a statute as alleged by the driver, that his measurements are correct, that there is a state statute requiring observance of traffic lights, and that the driver's license expired two years prior to the collision. The pedestrian's violation of the crosswalk statute should not be considered by the jury because A: there is no dispute in the evidence about factual cause. B: as a matter of law the violation of the statute results in liability for all resulting harm. C: as a matter of law the driver's conduct was an independent intervening cause. D: as a matter of law the injury to the pedestrian was not the result of a risk the statute was designed to protect against.

D is correct. Safety statutes are used to establish duty and breach only if the violated statute was designed to protect the particular class of foreseeable victim that the plaintiff belonged to, and the harm the plaintiff suffered was the type that the statute was designed to protect. Here, the pedestrian was hit by the driver when she was already in the other crosswalk, so she was not in the class of people that the statute was designed to protect, namely, pedestrians crossing streets outside of crosswalks. This answer choice appropriately acknowledges the test and distinction and is the correct response

A homeowner owned a large poisonous snake which had been defanged and was kept in a cage. A storm damaged the homeowner's house and the snake's cage, allowing the snake to escape. During the cleanup after the storm, a volunteer worker came across the snake. The worker tried to run away from the snake and fell, breaking his arm. In a suit by the worker against the homeowner based on strict liability in tort to recover for his injury, will the worker prevail? A: No, because the snake's escape was caused by a force of nature. B: No, because the worker should have anticipated an injury during his volunteer work. C: Yes, because the homeowner did not take adequate precautions to secure the snake. D: Yes, because the worker's injury was the result of his fear of the escaped snake.

D is correct. Strict liability applies to the homeowner because it was reasonably foreseeable that a person (i.e., the worker) who encountered his large, poisonous snake, with no reason to believe it had been de-fanged, would try to escape and potentially suffer an injury.

A plaintiff suffered a severe loss when his manufacturing plant, located in a shallow ravine, was flooded during a sustained rainfall. The flooding occurred because the city had failed to maintain its storm drain, which was located on city land above the plaintiff's premises, and because a railroad had failed to maintain its storm drain, which was located on railroad land below the plaintiff's premises. The flooding would not have occurred if either one of the two storm drains had been maintained properly. The plaintiff sued the railroad to recover compensation for his loss. The evidence in the case established that the failures of the two drains were caused by the respective negligence of the city and the railroad. There is no special rule insulating the city from liability. In his action against the railroad, the plaintiff should recover A: nothing, because he should have joined the city, without whose negligence he would have suffered no loss. B: nothing, because he did not introduce evidence that enables the court reasonably to apportion responsibility between the city and the railroad. C: one-half his loss, in the absence of evidence that enables the court to allocate responsibility fairly between the city and the railroad. D: all of his loss, because but for the railroad's negligence none of the flooding would have occurred.

D is correct. The "but for" test is satisfied where several acts combine to cause the injury, but none of the acts standing alone would have been sufficient. But for any of the individual acts, the injury would not have occurred. Here, although the plaintiff's damages resulted from the negligence of two independent entities, neither of which alone would have been sufficient to flood the plaintiff's manufacturing plant, the combined negligence satisfies the but for test. Therefore, the plaintiff may recover all of his losses.

A buyer mailed a signed order to a seller that read: "Please ship us 10,000 widgets at your current price." The seller received the order on January 7 and that same day mailed to the buyer a properly stamped, addressed, and signed letter stating that the order was accepted at the seller's current price of $10 per widget. On January 8, before receipt of the seller's letter, the buyer telephoned the seller and said, "I hereby revoke my order." The seller protested to no avail. The buyer received the seller's letter on January 9. Because of the buyer's January 8 telephone message, the seller never shipped the goods. Under the relevant and prevailing rules, is there a contract between the buyer and the seller as of January 10? A: No, because the order was an offer that could be accepted only by shipping the goods; and the offer was effectively revoked before shipment. B: No, because the buyer never effectively agreed to the $10 price term. C: Yes, because the order was, for a reasonable time, an irrevocable offer. D: Yes, because the order was an offer that seller effectively accepted before the buyer attempted to revoke it.

D is correct. The buyer's order was mailed, which invited acceptance also by mail, which the seller properly did, thus establishing a valid contract when it was dispatched on January 7. The buyer's January 8 attempt at revocation was therefore ineffective because a valid contract had already been formed. A is incorrect. Although under the UCC an offer "for prompt shipment" MAY be accepted by prompt shipment of the goods, that was not the only available means of acceptance here. B is incorrect. The buyer's offer manifested an agreement to purchase the widgets at the seller's "current price;" unless $10 was not the seller's current price, the price term was agreed to. C is incorrect. The offer did not state by its terms that it was irrevocable. However, the revocation was still ineffective because a valid contract had already been formed when the seller mailed the acceptance.

A statute in a jurisdiction makes it a crime to sell ammunition to a minor (defined as a person under the age of eighteen). The courts have interpreted this statute as creating a strict liability offense that does not require knowledge of the age of the purchaser and as creating vicarious liability. A minor, who was sixteen years old, but looked four or five years older, entered a store and asked a clerk for a box of .22 caliber shells. The store owner had instructed her employees not to sell ammunition to minors. The clerk asked the minor his age. The minor said he was twenty. The clerk then placed a box of shells on the counter and asked, "Anything else?" The minor said that was all he wanted but then discovered he did not have enough money to pay for the shells, so the clerk put the box back onto the shelf. If the owner of the store is charged with attempting to violate the statute, her best argument would be that A: it was impossible for the sale to have occurred. B: she had strictly instructed her employees not to sell ammunition to minors. C: the minor lied about his age. D: the clerk did not have the mental state needed for attempt.

D is correct. The clerk did not have the intent to sell ammunition to minors, and the store owner, as his accessory, should be found not guilty of attempting to sell ammunition to minors.

After several well-publicized deaths caused by fires in products made from highly flammable fabrics, a state enacted a statute prohibiting "the manufacture or assembly of any product in this state which contains any fabric that has not been tested and approved for flame retardancy by the Zetest Testing Company." The Zetest Testing Company is a privately owned and operated business located in the state. For many years, a fabric mill, located in the state has had its fabrics tested for flame retardancy by a competing testing company, located in a different state. The competitor is a reliable organization that uses a process for testing and approving fabrics for flame retardancy identical in all respects to that used by the Zetest Testing Company. Because the fabric mill wishes to continue to have its fabric tested solely by the competing testing company, the fabric mill files an action in its state court challenging the constitutionality of the statute as applied to its circumstances. In this suit, the court should hold the statute to be A: constitutional, because it is reasonably related to the protection of the reputation of the fabric industry located in the state. B: constitutional, because it is a legitimate means of protecting the safety of the public. C: unconstitutional, because it denies to the fabric mill the equal protection of the laws. D: unconstitutional, because it imposes an unreasonable burden on interstate commerce.

D is correct. The court should hold the statute to be unconstitutional in this case because requiring that all fabric be tested by a single company imposes an unreasonable burden on commerce where companies in neighboring states use the same testing methods.

An ophthalmologist from State A sued a manufacturer from State B in State A state court. The manufacturer makes widgets in a small factory and sells them throughout the country through his website. The ophthalmologist purchased a widget from the manufacturer through the manufacturer's website after seeing an advertisement for the widget in a State A newspaper. The ophthalmologist was injured when the widget malfunctioned. The ophthalmologist is seeking $85,000 in damages under State A products liability law. State A's long-arm statute authorizes the state to have the full range of constitutionally-valid personal jurisdiction. The manufacturer filed a motion to dismiss for lack of personal jurisdiction because the manufacturer has never been to State A. Should the court grant the manufacturer's motion to dismiss? A: Yes, because the manufacturer does not have minimum contacts with State A. B: Yes, because the manufacturer has the right to remove the case to federal court. C: No, because state courts are courts of general jurisdiction. D: No, because the manufacturer has purposely availed himself of State A law

D is correct. The court should not grant the manufacturer's motion because by advertising and selling his goods in State A, the manufacturer targeted citizens of State A, amounting to purposefully availing himself of the benefits of conducting business within State A. Based on this activity, it was foreseeable to the manufacturer that he could be called into court to defend a lawsuit. Because he has established minimum contacts with State A, the court may exercise personal jurisdiction over the manufacturer and his motion should be denied.

A pilot from State A sued a librarian from State B in a federal district court in State A. The librarian sold the pilot an expensive collectible watch through a website that generated a valid sales contract. The pilot's complaint alleged that the librarian breached the contract's terms because the librarian sold the pilot a watch that materially varied from the description the librarian had provided. The pilot is seeking $175,000 in damages. Shortly after selling the watch, the librarian went to State A on vacation with her family. This was the librarian's first time ever in State A. While visiting State A, the librarian was served with the pilot's complaint and a summons in accordance with State A law. The librarian filed a motion to dismiss for lack of jurisdiction. Should the court grant the librarian's motion to dismiss? A: Yes, because only federal courts have jurisdiction to hear disputes between citizens of different states with an amount in controversy over $75,000. B: Yes, because the librarian does not have sufficient minimum contacts with State A to establish personal jurisdiction. C: No, because personal jurisdiction requirements only apply to federal courts, not state courts. \ D: No, because the librarian's physical presence in State A when she was served is sufficient to establish personal jurisdiction.

D is correct. The court should not grant the motion because both subject-matter and personal jurisdiction are satisfied. The facts support diversity jurisdiction based on the citizenship of the parties and amount in controversy. Furthermore, under Pennoyer v. Neff, 95 U.S. 714 (1878), a court may exercise personal jurisdiction over a non-resident defendant if she is present in the forum state when personally served with process. This occurred here, when the librarian was in State A on vacation and personally served with the complaint and summons in accordance with State A law.

A defendant is charged with murder. The evidence shows that she pointed a gun at the victim and pulled the trigger. The gun discharged, killing the victim. The gun belonged to the victim. The defendant testifies that the victim told her, and she believed, that the "gun" was a stage prop that could fire only blanks, and that she fired the gun as part of rehearsing a play with the victim at his house. If the jury believes the defendant's testimony and finds that her mistaken belief that the gun was a prop was reasonable, they should find her A: guilty of murder. B: guilty of manslaughter. C: guilty of either murder or manslaughter. D: not guilty of murder or manslaughter.

D is correct. The defendant is using a mistake of fact defense. She incorrectly believed that the gun was a prop, not a real gun, and that it would fire blanks. Mistake of fact may be a defense only if it negates the state of mind required for the crime. Here, a mistake of fact would negate the mental states needed for both murder and manslaughter. However, for malice and general intent crimes, there is an extra requirement that the mistake of fact must be reasonable (i.e., the type of mistake that a reasonable person would have made under the circumstances). In this case, murder and manslaughter are general intent or malice crimes. Therefore, if the jury believes that the mistake of fact made by the defendant is reasonable, it is a valid defense to both crimes.

A defendant is on trial for bribing a government procurement officer by providing the officer with free vacation facilities. When the defendant was approached by an FBI investigator, the defendant stated that her invitation to the procurement officer to spend his vacation in the defendant's mountain cabin was a favor to a friend, unrelated to his government office. The defendant also said to the investigator that she would reveal some "hot" information on a large-scale fraud in exchange for the investigator's promise to "stop worrying about a little vacation." Is the investigator's testimony about the defendant's offer to give information admissible? A: No, because it is hearsay not within any exception. B: No, because the defendant made the offer in a negotiation for settlement of a criminal investigation. C: Yes, as a matter observed and reported by the investigator pursuant to a duty imposed by law. D: Yes, as a statement of an opposing party.

D is correct. The defendant's offer to the investigator is a statement made by an opposing party and can be admitted against the defendant as an admission.

A plaintiff sued a defendant, alleging that she was seriously injured when the defendant ran a red light and struck her while she was walking in a crosswalk. During the defendant's case, a witness testified that the plaintiff had told him that she was "barely touched" by the defendant's car. On cross-examination, should the court allow the plaintiff to elicit from the witness the fact that he is an adjuster for the defendant's insurance company? A: No, because testimony about liability insurance is barred by the rules of evidence. B: No, because the reference to insurance raises a collateral issue. C: Yes, for both substantive and impeachment purposes. D: Yes, for impeachment purposes only.

D is correct. The fact that the witness is an adjuster for the defendant's insurance company is a legitimate ground for impeachment for bias, but is inadmissible as substantive evidence to prove negligence or improper conduct. Evidence that a witness is biased or has an interest in the outcome of a suit tends to show that the witness has a motive to lie. A witness may always be impeached by extrinsic evidence of bias or interest, provided a proper foundation is laid. Federal Rule of Evidence (FRE) 411 on liability insurance states: "Evidence that a person was or was not insured against liability is not admissible to prove whether the person acted negligently or otherwise wrongfully. But the court may admit this evidence for another purpose, such as proving a witness's bias or prejudice or proving agency, ownership, or control." The FRE do not contain any explicit "collateral issue" rule. However, the trial judge has general discretion under FRE 403 to exclude evidence whose probative value is substantially outweighed by "a danger...of confusing the issues...undue delay [or] wasting time[.]" This discretion allows the judge to keep out evidence that would, under the common law approach, be banned as extrinsic evidence of a collateral issue. A witness's bad character for truthfulness is not considered a collateral matter, regardless of whether that witness has affirmatively asserted his good character for truthfulness. D is correct. The court should allow the plaintiff to cross-examine the witness on his employment as an adjuster with the defendant's insurance company, but only for impeachment purposes. This is because FRE 411 does not permit evidence of liability insurance to prove

Suffering from painful and terminal cancer, a wife persuaded her husband to kill her to end her misery. As they reminisced about their life together and reaffirmed their love for each other, the husband tried to discourage the wife from giving up. The wife insisted, however, and finally her husband held a gun to her head and killed her. The most serious degree of criminal homicide of which the husband can be legally convicted is A: no degree of criminal homicide. B: involuntary manslaughter. C: voluntary manslaughter. D: murder.

D is correct. The husband, with deliberation and premeditation, and with the intent to kill his wife, put a gun to her head and shot her. The husband is guilty of murder. The motive (which is not the same thing as intent) behind the husband's actions does not excuse this murder and does not negate his premeditation, deliberation, and malice aforethought.

To improve the quality of rental housing within its boundaries, a city proposed an ordinance requiring all new and existing rental housing units to provide at least one full bathroom for each bedroom, plumbing and electrical hookups for a washer and dryer, and a covered parking space. A majority of the owners of existing rental housing in the city opposed the ordinance. They argued that it would dramatically decrease the number of low-income rental housing units because owners would be unable to raise rents enough to recoup the investment required to comply. Without denying these contentions, the city enacted the ordinance. A plaintiff who owns low-income rental housing has sued the city, claiming only that the ordinance is unconstitutional on its face. Which of the following best states the burden of persuasion in this action? A: The city must demonstrate that the ordinance is necessary to serve a compelling state interest, because it adversely affects the fundamental right of rental housing owners to use their property in the manner they deem most economically efficient. B: The city must demonstrate that the ordinance is necessary to serve a compelling state interest, because it will have a substantial and disproportionate negative impact on low-income persons. C: The plaintiff must demonstrate that the ordinance is not substantially related to an important state interest, because it requires some owners of rental housing to invest money that they will not be able to recoup from increased rents. D: The plaintiff must demonstrate that there is no rational relationship between the ordinance and any legitimate state interest, because the ordinance regulates economic activity of a type normally presumed to be within state regulatory authority.

D is correct. The issue presented by this question is to determine which standard of review applies and who has the burden of persuasion. Strict scrutiny applies to regulations involving either suspect classifications or fundamental rights, with the burden of persuasion on the government. Intermediate scrutiny applies when there is a classification based on gender or legitimacy, with the burden of persuasion likely on the government. When the other two standards are inapplicable, then rational basis applies, with the burden of persuasion on the challenger. Here, the lawsuit concerns a city ordinance requiring various accommodations added to rental housing, which does not implicate suspect classifications, classifications based on gender or legitimacy, or fundamental rights. Therefore, the standard of review is rational basis with the burden of persuasion on the owners. A is incorrect. This is the standard for strict scrutiny. Courts use a strict scrutiny standard when a suspect classification or fundamental right is involved. Fundamental rights include the right to travel, privacy, voting, and all First Amendment rights. There is no fundamental right to rental income, so strict scrutiny does not apply here. B is incorrect. This is also the standard for strict scrutiny. Under this standard, a law will only be upheld if it is necessary to achieve a compelling or overriding government purpose. As stated above, strict scrutiny does not apply here. C is incorrect. This is the standard for intermediate scrutiny. Courts use intermediate scrutiny when a classification is based on gender or legitimacy is involved. Under this standard, a law will be upheld if it is substantially related to an important government purpose. Here, neither gender or legitimacy is at issue, and thus intermediate scrutiny does not apply.

While the defendant was in jail on a pending charge, his landlord called the police because rent had not been paid and because he detected a disagreeable odor coming from the defendant's apartment into the hallways. The police officer who responded to the call knew that the defendant was in jail. He recognized the stench coming from the defendant's apartment as that of decomposing flesh and, without waiting to obtain a warrant and using the landlord's passkey, entered the apartment with the landlord's consent. The lease to these premises gave the landlord a right of entry, at any reasonable hour, for the purpose of making repairs. The police officer found a large trunk in the bedroom which seemed to be the source of the odor. Upon breaking it open, he found the remains of the defendant's former mistress. The landlord's consent to the police officer's search of the defendant's apartment is A: a waiver of the defendant's Fourth Amendment rights, because a landlord has implied consent to enter a tenant's apartment. B: a waiver of the defendant's Fourth Amendment rights, because the lease gave the landlord express authority to enter the premises. C: not a waiver of the defendant's Fourth Amendment rights, because the landlord lacked probable cause to believe a crime was then in the process of commission. D: not a waiver of the defendant's Fourth Amendment rights, because the landlord had neither actual nor apparent authority to permit the entry.

D is correct. The landlord had no actual or apparent authority to permit the entry because landlords generally do not have such authority over the premises of their lessees, and the provision in the lease only allowed limited rights of entry to make repairs, not a general authority to control the premises. In fact, that the lease had to contain a provision to allow the landlord such a limited right of entry is an indication that the premises were controlled by the lessee alone. Without authority, the landlord could not waive the defendant's Fourth Amendment rights.

A man owned property that he used as his residence. The man received a loan, secured by a mortgage on the property, from a bank. Later, the man defaulted on the loan. The bank then brought an appropriate action to foreclose the mortgage, was the sole bidder at the judicial sale, and received title to the property as a result of the foreclosure sale. Shortly after the foreclosure sale, the man received a substantial inheritance. He approached the bank to repurchase the property, but the bank had decided to build a branch office on the property and declined to sell. If the man prevails in an appropriate action to recover title to the property, what will be the most likely reason? A: He had used the property as his residence. B: He timely exercised an equitable right of redemption. C: The court applied the doctrine of exoneration. D: The jurisdiction provides a statutory right of redemption

D is correct. The man will recover title if the jurisdiction has a statutory right of redemption giving him the option to pay and redeem title to the foreclosed property within a designated period of time. A is incorrect. Whether the man resided at the property is irrelevant unless the jurisdiction's statutory right of redemption requires that it was used in a certain manner for the right to apply (i.e., residentially). B is incorrect. The man only had the equitable right of redemption until the foreclosure sale, at which point this doctrine no longer applies. C is incorrect. Exoneration does not apply here because the man did not die; the property was sold by way of foreclosure.

The police received an anonymous tip that the defendant was growing marijuana in her backyard, which was surrounded by a 15-foot high, solid wooden fence. A police officer was unable to view the yard from the street, so he used a police helicopter to fly over the defendant's house. The officer identified a large patch of marijuana plants growing right next to the house and used this observation to obtain a search warrant. The defendant is prosecuted for possession of marijuana and moves to suppress use of the marijuana in evidence. The court should A: grant the motion, because the only purpose of the officer's flight was to observe the yard. B: grant the motion, because the defendant had a reasonable expectation of privacy in the curtilage around her house and the police did not have a warrant. C: deny the motion, because a warrant is not required for a search of a residential yard. D: deny the motion, because the defendant had no reasonable expectation of privacy from aerial observation

D is correct. The officer's action of looking into the yard from the police helicopter is not considered a search within the meaning of the Fourth Amendment. The defendant did not have an objective, reasonable expectation of privacy from aerial observation because the observations were made from the public airspace and were no more than any private citizen could have seen from the same location. Because the marijuana was properly seen in plain view, and the actions of the officers did not violate the Fourth Amendment, the defendant's motion to suppress should be denied.

A landowner and a contractor entered into a written contract under which the contractor agreed to build a building and pave an adjacent sidewalk for the landowner for $200,000. Later, while construction was proceeding, the landowner and the contractor entered into an oral modification under which the contractor was not obligated to pave the sidewalk but still would be entitled to $200,000 upon completion of the building. The contractor completed the building. The landowner, after discussions with his landscaper, demanded that the contractor pave the adjacent sidewalk. The contractor refused. Has the contractor breached the contract? A: No, because the oral modification was in good faith and therefore enforceable. B: Yes, because a discharge of a contractual obligation must be in writing. C: Yes, because the parol evidence rule bars proof of the oral modification. D: Yes, because there was no consideration for the discharge of the contractor's duty to pave the sidewalk.

D is correct. The oral modification that removed the contractor's duty to pave the sidewalk will not be enforced because it was not supported by new consideration, nor did it fall under an exception that allows modification without new consideration. The contractor is bound to the original agreement and by refusing to pave the sidewalk, he is in breach. A is incorrect. This choice is referring to the UCC rule for modification that requires only good faith and no new consideration. However, this question does not involve the sale of goods, but rather, a services contract, which means common law will apply and new consideration was necessary for the modification to be enforceable. B is incorrect. To the contrary, the discharge of a party's contractual duty may be oral (rather than written) in many circumstances. Nevertheless, the contractor is in breach because the modification lacked new consideration, as explained above. C is incorrect. Parol evidence is admissible when examining an oral modification of a pre-existing written contract.

A contractor agreed to remodel a homeowner's garage for $5,000. Just before the parties signed the one-page written contract, the homeowner called to the contractor's attention the fact that the contract did not specify a time of completion. The parties orally agreed but did not specify in the contract that the contractor would complete the work in 60 days, and then they both signed the contract. The contract did not contain a merger clause. The contractor failed to finish the work in 60 days. The homeowner has sued the contractor for breach of contract. Is the court likely to admit evidence concerning the parties' oral agreement that the work would be completed in 60 days? A: No, because the court must ascertain the meaning of the agreement from the terms of the written contract. B: No, because the oral agreement was merely part of the parties' negotiations. C: Yes, because the contract is ambiguous. D: Yes, because the time limit is an additional term that does not contradict the partially integrated written contract.

D is correct. The parol evidence rule states that evidence of earlier agreements cannot be considered for the purpose of contradicting terms in the written contract. Here, the contract did not contain a merger clause, meaning that this written agreement was a partial integration. A partial integration may be supplemented (but not contradicted) to prove consistent additional terms. The written agreement did not contain terms that stipulated by when the work must be completed. The oral evidence would provide additional terms that do not contradict the original writing, and would, therefore, be allowed. A is incorrect. Because the written agreement lacked a merger clause, it is a partial integration, meaning the court may consider other evidence. B is incorrect. Although the oral expressions were made contemporaneously with the writing, it does not vary nor contradict the terms of the writing. The time in which the work must be completed is a missing term in the written agreement and is not merely part of the negotiations. C is incorrect. The timeframe is not ambiguous in the written agreement, but rather, it is missing.

In an action brought against a defendant by a pedestrian's legal representative, the only proof that the legal representative offered on liability were that: (1) the pedestrian was killed instantly while walking on the shoulder of the highway; (2) the defendant was driving the car that struck the pedestrian; and (3) there were no living witnesses to the accident other than the defendant, who denied negligence. The jurisdiction has adopted a rule of pure comparative negligence. If, at the end of the plaintiff's case, the defendant moves for directed verdict, the trial judge should A: grant the motion, because the legal representative has offered no specific evidence from which reasonable jurors may conclude that the defendant was negligent. B: grant the motion, because it is just as likely that the pedestrian was negligent as that the defendant was negligent. C: deny the motion, because the pedestrian was in violation of the state highway code. D: deny the motion, because, in the circumstances, negligence on the part of the defendant may be inferred.

D is correct. The pedestrian's representative introduced sufficient evidence for a res ipsa loquitur claim, which has the effect of allowing the jury to decide whether to infer the defendant's negligence. A directed verdict for the defendant would thus be improper because the jury must decide whether it will infer the defendant's negligence or find that his denial of negligence overcomes the res ipsa showing.

The owner of a house told his neighbor that he was going away for two weeks and asked the neighbor to keep an eye on his house. The neighbor agreed. The owner gave the neighbor a key to use to enter the house. The neighbor decided to have a party in the owner's house. He invited a number of friends. One friend, a pickpocket, went into the owner's bedroom, took some of the owner's rings, and put them in his pocket. Which of the following is true? A: The neighbor and the pickpocket are guilty of burglary. B: The neighbor is guilty of burglary and the pickpocket is guilty of larceny. C: The neighbor is guilty of trespass and the pickpocket is guilty of larceny. D: The pickpocket is guilty of larceny and the neighbor is not guilty of any crime.

D is correct. The pickpocket took the owner's property, without the owner's consent, and put it in his pocket. The pickpocket is thus guilty of larceny. The neighbor is not criminally responsible for the pickpocket's theft. Additionally, the neighbor had permission from the owner to be in the residence. Since the neighbor was given permission to watch the house, he did not commit a trespass by being in the house that evening.

Two police officers in uniform were on foot patrol in a neighborhood frequented by drug sellers. They saw the defendant, who, when she saw them, turned around and started to walk quickly away. The police ran after her and shouted, "Stop and don't take another step, lady!" The defendant turned, looked at the police, and stopped. She put her arms up in the air. As the police approached, she threw a small object into the nearby bushes. The police retrieved the object, which turned out to be a small bag of cocaine, and then arrested the defendant. The defendant is charged with possession of the cocaine. She moves pretrial to suppress its use as evidence on the ground that it was obtained as a result of an illegal search and seizure. Her motion should be A: granted, because the police did not know the item was cocaine until after they had seized it. B: granted, because the police acquired the cocaine as the result of an unlawful seizure. C: denied, because the police had probable cause to seize the package. D: denied, because the defendant voluntarily discarded the contraband.

D is correct. The police made a valid Terry stop based on the circumstances; the defendant was in a high-crime area and ran as soon as she saw the police. The defendant then voluntarily discarded the cocaine, at which point she relinquished any privacy rights to it.

Two companies each manufacture pesticide. Their plants are located along the same river. During a specific 24-hour period, each plant discharged pesticide into the river. Both plants were operated negligently and such negligence caused the discharge of pesticide into the river. A rancher operated a cattle ranch downstream from the companies' plants. The rancher's cattle drank from the river and were poisoned by the pesticide. The amount of the discharge from either plant alone would not have been sufficient to cause any harm to the rancher's cattle. If the rancher asserts a claim against the two companies, what, if anything, will the rancher recover? A: Nothing, because neither company discharged enough pesticide to cause harm to the rancher's cattle. B: Nothing, because the rancher cannot establish how much pesticide each plant discharged. C: One-half of the rancher's damages from each company. D: The entire amount of the rancher's damages, jointly and severally, from the two companies

D is correct. The rancher's damages resulted from the acts of two independent companies, neither of which alone would have been sufficient to poison the cattle, and in circumstances that make it impossible to determine if either defendant alone caused the injury. This is a concurrent cause of harm, and majority courts will hold that where the defendant's negligent act unites with another event, the acts of both defendants will be considered the cause of at least part of the harm, and the defendants will be held jointly and severally liable. The effect is that both companies are liable for the entire amount of the rancher's damages.

A scientist properly filed a diversity action against an engineer in federal court in State A, alleging that the engineer failed to pay a debt due on a valid contract for services rendered. The engineer did not dispute that there was a contract between herself and the scientist. The engineer argued that she paid the debt and has records of the checks made payable to the scientist to prove it, although has not produced any evidence in support of this assertion. In response, the scientist filed: (i) an affidavit, stating that the engineer never paid the debt on the contract, and that the checks from the engineer are payment for a used automobile that the scientist sold to the engineer and not for any services rendered; and (ii) copies of the engineer's checks with the words "for used car" in the memo line. The trial is set to begin next week. Based only on evidence in the record, which party is likely to prevail at this point in the lawsuit? A: The engineer, on a motion for summary judgment. B: The engineer, on a motion to dismiss for failure to state a claim upon which relief can be granted. C: The scientist, on a motion for a judgment as a matter of law. D: The scientist, on a motion for summary judgment.

D is correct. The scientist would likely prevail on a motion for summary judgment because the engineer has not disputed the contract's validity, and the engineer has merely stated that she paid the debt and that she has the checks, but has not offered any evidence into the record. Based on what is contained in the record, including the affidavit and checks the scientist offered to prove that the engineer did not pay the debt owed for the breach of contract, the scientist would likely prevail on a motion for summary judgment

A man and a woman planned to hold up a bank. They drove to the bank in the man's car. The man entered while the woman remained as lookout in the car. After a few moments, the woman panicked and drove off. The man looked over the various tellers, approached one and whispered nervously, "Just hand over the cash. Don't look around, don't make a false move--or it's your life." The teller looked at the fidgeting man, laughed, flipped him a dollar bill and said, "Go on, beat it." Flustered, the man grabbed the dollar and left. Soon after leaving the scene, the woman was stopped by the police for speeding. Noting her nervous condition, the police asked the woman if they might search the car. She agreed. The search turned up heroin concealed in the lid of the trunk. The woman's best defense to a charge of robbery would be that A: the man alone entered the bank. B: the woman withdrew before commission of the crime when she fled the scene. C: the woman had no knowledge of what the man whispered to the teller. D: the teller was not placed in fear by the man.

D is correct. The woman and the man had planned on holding up the bank, and the woman aided in the planning and drove the vehicle to the bank. She will bear criminal responsibility for the man's actions. The woman's best defense to the charge of robbery would be that the man did not commit a robbery because the teller was not placed in fear by the man's threat. Thus, although she had the requisite intent and did aid and abet the man, an actual robbery did not occur.

A painter filed a diversity action against a doctor in federal court in State A. The painter alleged that the doctor failed to pay $95,000 due on a valid service contract. The doctor does not dispute there was a contract, but testified in a deposition that he paid the debt. The painter stated in a deposition that the doctor never paid the debt due pursuant to the contract and that the $95,000 the doctor paid was for a rare sports car he sold the doctor. The trial is set to begin next week. Based only on evidence in the record, which party is likely to prevail at this point in the lawsuit? A: The painter, on a motion for summary judgment. B: The doctor, on a motion for summary judgment. C: The doctor, on a motion to dismiss for failure to state a claim upon which relief can be granted. D: Neither party, on motions for summary judgment.

D is correct. There is a genuine issue of material fact with regard to both the painter's claim and the doctor's defense regarding whether the $95,000 payment was for the contractual debt or something else. Therefore, because there is an issue regarding a genuine issue of material fact, this is in dispute this case will probably have to go to trial.

A college student purchased a large bottle of No-Flake dandruff shampoo, manufactured by a shampoo company. The box containing the bottle stated in part: "CAUTION - Use only one capful at most once a day. Greater use may cause severe damage to the scalp." The college student read the writing on the box, removed the bottle, and threw the box away. The college student's roommate asked to use the No-Flake, and college student said, "Be careful not to use too much." The roommate thereafter used No-Flake twice a day, applying two or three capfuls each time, notwithstanding the label statement that read: "Use no more than one capful per day. See box instructions." The more he used No-Flake, the more inflamed his scalp became, the more it itched, and the more he used. After three weeks of such use, the roommate finally consulted a doctor who diagnosed his problem as a serious and irreversible case of dermatitis caused by excessive exposure to the active ingredients by No-Flake. These ingredients are uniquely effective at controlling dandruff, but there is no way to remove a remote risk to a small percentage of persons who may contract dermatitis as the result of applying, for prolonged periods of time, amounts of No-Flake substantially in excess of the directions. This jurisdiction adheres to the traditional common law rules pertaining to contributory negligence and assumption of risk. The roommate asserts a claim for his injuries against the shampoo company based on strict liability in tort. Three important facts were established at trial: The roommate misused the No-Flake shampoo, the roommate was contributorily negligent in continuing to use No-Flake shampoo when his scalp began to hurt and itch, and the roommate was a remote user and not in privity with the shampoo company. Which of the following would constitute a defense for the shampoo company? A: The roommate misused the No-Flake shampoo. B: The roommate misused the shampoo and was contributorily negligent in continuing to use No-Flake shampoo when his scalp began to hurt and itch. C: The roommate was not in privity with the shampoo company. D: The product was substantially changed from the condition in which it was sold.

D is correct. This question is asking for which answer choice would be the BEST defense against a products liability claim. If the shampoo company could show that the product was substantially changed from the condition in which it was sold, then there would not be adequate causation to sustain the roommate's claim against the company. Thus, this would be a valid defense to the strict liability claim.

A train conductor brought an action against her railroad employer under a federal statute providing liability for workrelated injuries occurring on railroads. The employer denied liability, claiming that the conductor's injuries pre-dated her employment and were outside the scope of the statute. At the close of the evidence at trial, the employer moved for judgment as a matter of law (JMOL), which the court denied. The jury returned a verdict for the conductor. The employer has renewed its JMOL motion. What standard should the court apply in ruling on the motion? A: Whether a preponderance of the evidence supports the verdict. B: Whether the verdict is against the weight of the evidence. C: Whether there is a scintilla of evidence to support the verdict. D: Whether there is legally sufficient evidence to support the verdict.

D is correct. Under Federal Rule of Civil Procedure (FRCP) 50, a court may grant judgment as a matter of law against a party only if the court finds that a reasonable jury would not have a legally sufficient evidentiary basis to find for that party.

A state requires licenses of persons "who are engaged in the trade of barbering." It will grant such licenses only to those who are graduates of barber schools located in the state, have resided in the state for two years, and are citizens of the United States. The requirement that candidates for licenses must be citizens is A: constitutional as an effort to ensure that barbers speak English adequately. B: constitutional as an exercise of the state police power. C: unconstitutional as a bill of attainder. D: unconstitutional as a denial of equal protection.

D is correct. Under the Equal Protection Clause, a state statute that uses alienage as a classification must pass strict scrutiny. Here, the classification fails strict scrutiny because the state cannot prove that limiting barber licenses to citizens is necessary to achieve a compelling state interest.

A seller owned a single-family house. A buyer gave the seller a signed handwritten offer to purchase the house. The offer was unconditional and sufficient to satisfy the statute of frauds, and when the seller signed an acceptance, an enforceable contract resulted. The house had been the seller's home, but he had moved to an apartment, so the house was vacant at all times relevant to the proposed transaction. Two weeks after the parties had entered into their contract, one week after the buyer had obtained a written mortgage lending commitment from a lender, and one week before the agreed-upon closing date, the house was struck by lightning and burned to the ground. The loss was not insured, because three years earlier, the seller had let his homeowner's insurance policy lapse after he had paid his mortgage debt in full. The handwritten contract was wholly silent as to matters of financing, risk of loss, and insurance. The buyer declared the contract voided by the fire, but the seller asserted a right to enforce the contract despite the loss. There is no applicable statute. If a court finds for the seller, what will be the likely reason? A: The contract was construed against the buyer, who drafted it. B: The lender's written commitment to make a mortgage loan to the buyer made the contract of sale fully binding on the buyer. C: The risk of loss falls on the party in possession, and constructive possession passed to the buyer on the contract date. D: The risk of loss passed to the buyer on the contract date under the doctrine of equitable conversion.

D is correct. Under the doctrine of equitable conversion, if the contract is silent regarding the risk of loss, that risk goes to the party with the equitable title. Equitable conversion occurs when the contract is capable of specific performance. This contract was silent regarding the risk of loss, and there were no conditions to be met. The buyer thus had the equitable title at the time of the loss. Because the facts state that there is no statute in the jurisdiction, the Uniform Vendor and Purchaser Act, which places the risk of loss on the one in possession, is not applicable. The court found for the seller, and thus the minority common law rule, which places the risk on the seller under these facts, is inapplicable. A is incorrect. It is not relevant who drafted the contract. The contract was silent regarding any risk of loss. A minority of states have adopted the Uniform Vendor and Purchaser Act, which places the risk of loss on the seller instead of the buyer. The court found for the seller, and thus the minority rule, which places the risk on the seller under these facts, is inapplicable. Under the majority common law rule of equitable conversion, the individual with the equitable interest bears the risk of loss. The equitable title passed to the buyer when the seller signed the contract, because the contract was unconditional and was silent regarding the risk of loss. B is incorrect. There were no conditions in the contract of sale. The contract became binding when the seller signed the acceptance. The buyer would have been obligated to purchase even if the buyer had not received a loan commitment. Since the jurisdiction has no statute, the Uniform Vendor and Purchaser Act, which places the risk of loss on the one in possession, is not applicable. The court found for the seller, and thus the minority common law rule, which places the risk on the seller under these facts, is inapplicable. Under the majority common law rule of equitable conversion, the individual with the equitable interest bears the risk of loss. The equitable title passed to the buyer when the seller signed the contract, because the contract was unconditional and was silent regarding the risk of loss. C is incorrect. Possession does not pass to the buyer until closing absent a contrary provision in the contract of sale. © Copyright 2003 - 2023 AdaptiGroup LLC. This product is covered by U.S. Patent No. 8,340,568. All rights reserved. 19 In a jurisdiction that has adopted the Uniform Vendor and Purchaser Act, if a contract is silent regarding the risk of loss, the risk of loss is on the party in possession. The question notes, however, that the jurisdiction has no applicable statute. The court found for the seller and thus the minority common law rule, which places the risk on the seller under these facts, is inapplicable. Under the majority common law rule of equitable conversion, the individual with the equitable interest bears the risk of loss. The equitable title passed to the buyer when the seller signed the contract, because the contract was unconditional and was silent regarding the risk of loss.

A husband and wife acquired land as common law joint tenants with right of survivorship. One year later, without his wife's knowledge, the husband executed a will devising the land to his best friend. The husband subsequently died. Is the wife now the sole owner of the land? A: No, because a joint tenant has the unilateral right to end a joint tenancy without the consent of the other joint tenant. B: No, because the wife's interest in the husband's undivided 50% ownership in the land adeemed. C: Yes, because of the doctrine of after-acquired title, or estoppel by deed. D: Yes, because the devise to the friend did not sever the joint tenancy.

D is correct. When the husband died, the wife's interest in the joint tenancy immediately swelled and she became the sole owner of the land as the surviving joint tenant. A is incorrect. A joint tenant's interest cannot be devised in a will, which means this will not sever the joint tenancy. B is incorrect. Ademption does not apply because the husband did not lose his property rights and then devise them to another party. His death activated the wife's right to sole ownership of the land as the surviving joint tenant. C is incorrect. The doctrine of after-acquired title is inapplicable here because this is not a case where an individual attempted to convey title without ownership and then later gets title

An actress, who played the lead role in a television soap opera, was seriously injured in an automobile accident caused by the defendant's negligent driving. As a consequence of the actress's injury, the television series was canceled, and a supporting actor was laid off. Although the supporting actor looked for other work, he remained unemployed. In an action against the defendant, can the supporting actor recover for his loss of income attributable to the accident? A: Yes, because the defendant's negligence was the cause in fact of the supporting actor's loss. B: Yes, because the supporting actor took reasonable measures to mitigate his loss. C: No, because the defendant had no reason to foresee that by injuring the lead actress he would cause harm to the supporting actor. D: No, because the defendant's liability does not extend to economic loss to the supporting actor that arises solely from physical harm to the lead actress.

D is correct. With the exception of a wrongful death claim allowed by statute, a negligence action for pure economic loss to a plaintiff as the result of an injury suffered by a third party is generally not recoverable. As a matter of public policy, courts will not find liability for purely economic losses in tort law. Typically, that is what contract law is for. Here, because the supporting actor is seeking to recover damages as a result of the television series being canceled and being laid off, these damages constitute economic damages and therefore are not recoverable. Courts, as a policy matter, will refuse to find proximate cause in such cases.

1. Decker, charged with armed robbery of a store, denied that he was the person who had robbed the store. In presenting the state's case, the prosecutor seeks to introduce evidence that Decker had robbed two other stores in the past year. This evidence is A) Admissible, to prove a pertinent trait of Decker's character and Decker's action in conformity therewith. B) Admissible, to prove Decker's intent and identity. C) Inadmissible, because character must be proved by reputation or opinion and may not be proved by specific acts. D) Inadmissible, because its probative value is substantially outweighed by the danger of unfair prejudice.

D) Inadmissible, because its probative value is substantially outweighed by the danger of unfair prejudice.

1. Janet has an established dental practice. She ordered three new dental chairs from a supply company for a total of $9,000, fully payable 60 days after delivery. Before the chairs were delivered, Janet decided to sell her practice to Lori, a friend from dental school. They included in their sale agreement a provision that the dental chair purchase was assignable to Lori. Lori notified the dental supply company of the assignment. The dental supply company shipped the three chairs. Lori accepted and used them in the practice but refused to pay for them for four months. Who is liable for the payment to the dental supply company? A) Lori is liable, but Janet is not, because there was a novation. B) Janet is liable, but Lori is not, because Lori is not, because Lori never promised the dental supply company she would pay for the chairs. C) Janet is liable, but Lori is not, because this was not an effective delegation of duties. D) Janet and Lori are both liable.

D) Janet and Lori are both liable.

1. A defendant was charged with aggravated assault. At trial, the victim testified that the defendant beat her savagely, but she was not asked about anything said during the incident. The prosecutor then called a witness to testify that when the beating stopped, the victim screamed: "I'm dying-don't let [the defendant] get away with it!" Is the testimony of the witness concerning the victim's statement admissible? A) No, because it is hearsay not within any exception. B) No, because the victim was not asked about the statement. C) Yes, as a statement under belief of impending death, even though the victim did not die. D) Yes, as an excited utterance.

D) Yes, as an excited utterance.

The defendant was on trial for murder. Following a recess, the defendant was in the crowded hallway outside the courtroom speaking with his attorney. During their conversation, the defendant said, "So what if I killed him, big deal." The defendant's statement was overheard by a witness as he was walking down the corridor. The witness then informed the prosecuting attorney of the defendant's comment. After trial is reconvened, the prosecutor calls the witness to testify to what he heard the defendant tell his attorney. The defendant's attorney objects, arguing that this would violate the attorney-client privilege. May the witness be called to testify? A) No, because the defendant's statement was a confidential communication and protected under the attorney-client privilege. B) No, because the statement is inadmissible as hearsay not within any recognized exception. C) Yes, because the witness has a duty to disclose incriminating evidence to the prosecution. D) Yes, because the defendant's statement was not a confidential communication.

D) Yes, because the defendant's statement was not a confidential communication.

1. Beth wanted to make some money, so she decided to sell cocaine. She asked Albert, who was reputed to have access to illegal drugs, to supply her with cocaine so she could resell it. Albert agreed and sold Beth a bag of white powder. Beth then repackaged the white powder into smaller containers and sold one to Carol, an undercover police officer, who promptly arrested Beth. Beth immediately confessed and said Albert was her supplier. Upon examination, the white powder was found not to be cocaine or any type of illegal substance. If Albert knew the white powder was not cocaine but Beth believed it was, which of the following is correct? A- Both Albert and Beth are guilty of attempting to sell cocaine. B- Neither Albert nor Beth is guilty of attempting to sell cocaine. C- Albert is guilty of attempting to sell cocaine, but Beth is not. D- Albert is not guilty of attempting to sell cocaine, but Beth is.

D- Albert is not guilty of attempting to sell cocaine, but Beth is.

1- A federal state appropriated $7 million for a nationwide essay contest on "How the United States Can Best Stop Drug Abuse." The statute indicates that its purpose is to generate new, practical ideas for eliminating drug abuse in the United States. Contest rules set forth in the statute provide that winning essays are to be selected on the basis of the "originality, aptness and feasibility of their ideas." The statute expressly authorizes a first prize of $1 million, 50 second prizes of $10,00 each. It also states that judges for the contest are to be appointed by the President of the United States with the advice and consent of the Senate, and that all residents of the United States who are not employees of the federal government are eligible to enter and win the contest. A provision of the statute authorizes any taxpayer of the United States to challenge its constitutionality. In a suit by a federal taxpayer to challenge the constitutionality of the statute, the court should A- Refuse to decide its merits, because the suit involves policy questions that are inherently political and, therefore, nonjusticiable. B- Hold the statute unconstitutional, because it does not provide sufficient guidelines for awarding the prize money appropriated by Congress and, therefore, unconstitutionally delegates legislative power to the contest judges. C- Hold the statute unconstitutional, because its relationship to legitimate purposes of tenuous and conjectural to satisfy the necessary and proper clause of Article I. D- Hold the statute constitutional, because it is reasonably related to the general welfare, it states concrete objectives, and it provides adequate criteria for conducting the essay contest and awarding the prize money.

D- Hold the statute constitutional, because it is reasonably related to the general welfare, it states concrete objectives, and it provides adequate criteria for conducting the essay contest and awarding the prize money. Spending issue look for general welfare

1. Sally told Michael she would like to have sexual intercourse with him and that he should come to her apartment that night at 7 P.M. After Michael arrived, he and Sally went into the bedroom. As Michael started to remove Sally's blouse, Sally said she had changed her mind. Michael tried to convince her to have intercourse with him, but after ten minutes of her sustained refusals, Michael left the apartment. Unknown to Michael, Sally was 15 years old. Because she appeared to be older, Michael believed her to be about 18 years old. A statute in the jurisdiction provides: "A person commits rape in the second degree if he has sexual intercourse with a girl, not his wife, who is under the age of 16 years." If Michael is charged with attempting to violate this statute, he is A- Guilty, because no mental state is required as to the element of age. B- Guilty, because he persisted after she told him she had changed her mind. C- Not guilty, because he reasonably believed she had consented and voluntarily withdrew after she told him she had changed her mind. D- Not guilty, because he did not intend to have intercourse with a girl under the age of 16.

D- Not guilty, because he did not intend to have intercourse with a girl under the age of 16.

1- A federal statute required the National Bureau of Standards to establish minimum quality standards for all beer sold in the United States. The statute also provided that public proceedings must precede adoption of the standards, and that once they were adopted, the standards would be subject to judicial review. No standards have yet been adopted. Several officials of the National Bureau of Standards have indicated their personal preference for beer produced by a special brewing process commonly referred to as pasteurization. However, these officials have not indicated whether they intend to include a requirement for pasteurization in the minimum beer quality standards to be adopted by the Bureau. A brewery that produces an unpasteurized beer believes that its brewing process is as safe as pasteurization. The brewery is concerned that, after the appropriate proceedings, the Bureau may adopt quality standards that will prohibit the sale of any unpasteurized beer. As a result, the brewery sued in federal district court to enjoin the Bureau from adopting any standards that would prohibit the sale of unpasteurized beer in this country. How should the district court dispose of the suit? A- Determine whether the Bureau could reasonably believe that pasteurization is the safest process by which to brew beer, and if the Bureau could reasonably believe that, refuse to issue the injunction against the Bureau B- Determine whether the process used by the brewery is as safe as pasteurization and, if it is, issue the injunction against the Bureau C- Refuse to adjudicate the merits of the suit at this time and stay the action until the Bureau has actually issued beer quality standards. D- Refuse to adjudicate the merits of the suit, because it does not involve a justiciable case or controversy.

D- Refuse to adjudicate the merits of the suit, because it does not involve a justiciable case or controversy. (ripeness or mootness = no case or controversy)

1) In an action brought against Driver by Walker's legal representative, the only proofs that the legal representative offered on liability were that: (1) Walker, a pedestrian, was killed instantly while walking on the shoulder of the highway; (2) Driver was driving the car that struck Walker; and (3) there were no living witnesses to the accident other than Driver, who denied negligence. Assume the jurisdiction has adopted a rule of pure comparative negligence. If, at the end of the plaintiff's case, Driver moves for a directed verdict, the trial judge should A) Grant the motion, because the legal representative has offered no specific evidence from which reasonable jurors may conclude that Driver was negligent. B) Grant the motion, because it is just as likely that Walker was negligent as that Driver was negligent. C) Deny the motion, unless Walker was walking with his back to traffic, in violation of the state highway code. D) Deny the motion, negligence on the part of Driver may be inferred.

D. Deny the motion, negligence on the part of Driver may be inferred.

1) Dieter parked her car in violation of a city ordinance that prohibits parking within ten feet of a fire hydrant. Because Grove was driving negligently, his car was sideswiped Dieter's parked car. Plaintiff, a passenger in Grove's car, was injured in the collision. If plaintiff asserts a claim against Dieter to recover damages for his injuries, basing his claim on Dieter's violation of the parking ordinance, will Plaintiff prevail? A) Yes, because Dieter was guilty of negligence per se. B) Yes, if Plaintiff would not have been injured had Dieter's car not been parked where it was. C) No, because Dieter's parked car was not an active or efficient cause of Plaintiff's injury. D) No, if prevention of traffic accidents was not a purpose of the ordinance.

D. No, if prevention of traffic accidents was not a purpose of the ordinance. Rule for neg. per se.

1. Defendant is charged with murder. The evidence shows that she pointed a gun at Victim and pulled the trigger. The gun discharged, killing Victim. The gun belonged to Victim. Defendant testifies that Victim told her, and she believed, that the "gun" was a stage prop that could fire only blanks, and that she fired the gun as part of rehearsing a play with Victim at his house. If the jury believes Defendant's testimony and finds that her mistaken belief that the gun was a prop was reasonable, they should find her A- Guilty of murder B- Guilty of manslaughter C- Guilty of either murder or manslaughter D- Not guilty of murder or manslaughter

D. Not guilty of murder or manslaughter can use mistake as a defense to a general intent crime, where the mistake was reasonable. Any kind of mistake will be a defense to murder whether it was reasonable or not.

1) A 16-year-old boy purchased an educational chemistry set manufactured by Chemo. The teenager invited his friend and classmate, the plaintiff, to assist him in a chemistry project. Referring to a library chemistry book on explosives and finding chemistry set contained all of the necessary chemicals, the teenager and the plaintiff agreed to make a bomb. During the course of the project, the teenager carelessly knocked a lighted Bunsen burner into a bowl of chemicals from the chemistry set. The chemicals burst into flames, injuring the plaintiff. Although the chemistry set was as safe as possible, and its educational benefits exceeded its risks, the set did not contain warning that it could be used to make dangerous explosives. In a suit by the plaintiff against Chemco, based on strict liability, the plaintiff will A) Prevail, because the chemistry set did not contain a warning that its contents could be combined to form dangerous explosives. B) Prevail, because manufacturers of chemistry sets are engaged in an abnormally dangerous activity. C) Not prevail, because the teenager's negligence was the cause in fact of the plaintiff's injury. D) Not prevail, because the chemistry set was safe as possible, consistent with its educational purposes, and its benefits exceeded its risks.

D. Not prevail, because the chemistry set was safe as possible, consistent with its educational purposes, and its benefits exceeded its risks. Is using the words from the fact because in plain English is the most specific answer to what the fact pattern is telling you

1) Oscar purchased a large bottle of No-Flake dandruff shampoo, manufactured by Shampoo Company. The box containing the stated in part: "CAUTION—use only 1 capful at most once a day. Greater use may cause severe damage to the scalp." Oscar read the writing on the box, removed the bottle, and threw the box away. Oscar's roommate, Paul, asked to use the No-Flake, and Oscar said, "Be careful not to use too much." Paul thereafter used No-Flake twice a day, applying two or three capfuls each time, notwithstanding the label statement that read: "Use no more than one capful per day. See box instructions." The more he used No-Flake, the more inflamed his scalp became, the more it itched, and the more he used. After three weeks of such use, Paul finally consulted a doctor who diagnosed his problem as a serious and irreversible case of dermatitis caused by excessive exposure to the active ingredients in No-Flake. These ingredients are uniquely effective at controlling dandruff, but there is no way to remove a remote risk to a small percentage of persons who may contract dermatitis as the result of applying for prolonged periods of time amounts of No-Flake substantially in excess of the directions. This jurisdiction adheres to the traditional common-law rules pertaining to contributory negligence and assumption of risk. Based upon the foregoing facts, if Paul sues Shampoo Company to recover damages for his dermatitis, his most promising theory of liability will be that the No-Flake shampoo A) Had an unreasonably dangerous manufacturing defect. B) Had an unreasonably dangerous design defect. C) Was inherently dangerous. Talking about strict liability D) Was inadequately labeled to warn of its dangers.

D. Was inadequately labeled to warn of its dangers. Talking about the lack of warning


Related study sets

Chapter 24 The origin of species

View Set

Modern World History Patterns Final

View Set

Hospitalized Adult Assessment PrepU

View Set

Need a New Drug? Exam 2 Study Guide

View Set

Chapter 3: Collecting Objective Data: The Physical Examination PrepU

View Set